इंदौर शाखा: IAS और MPPSC फाउंडेशन बैच-शुरुआत क्रमशः 6 मई और 13 मई   अभी कॉल करें
ध्यान दें:

प्रारंभिक परीक्षा


प्रारंभिक परीक्षा
Switch To English

प्रारंभिक परीक्षा, 2023: एक विश्लेषणात्मक अध्ययन

  • 02 Aug 2023
  • 234 min read

1. धान्यकटक, जो महासांघिकों के अधीन एक प्रमुख बौद्ध केंद्र के रूप में समृद्ध हुआ, निम्नलिखित में से किस एक क्षेत्र में अवस्थित था?

(a) आंध्र 
(b) 
गांधार
(c) कलिंग
(d) मगध

उत्तर:(a)

व्याख्या:

  • धान्यकटक वर्तमान में दक्षिण पूर्वी भारत के आंध्र प्रदेश में अमरावती के निकट स्थित एक छोटा-सा नगर है, जहाँ शाक्यमुनि बुद्ध ने शंभला के राजाओं को कालचक्र धर्म के सार स्वरूप की शिक्षा दी थी।
  • अमरावती धरणीकोटा/धरनीकोटा, इतिहास में तीसरी बार (12वीं शताब्दी में) कोटा शासकों की राजधानी बना था। गुण्टूर के वेलपुरु में एक मंदिर में पाए गए एक शिलालेख के अनुसार, अमरावती को निम्नलिखित प्रकार से वर्णित किया गया था:
    • "श्री धान्यकटक नाम का एक नगर है, जो देवताओं के नगर से श्रेष्ठ है, जहाँ अमरेश्वर नाम के शंभू मंदिर की पूजा देवताओं के राजा (इंद्र) द्वारा की जाती है, यहाँ भगवान बुद्ध को भी पूजा जाता है। यहाँ एक बहुत ऊँचा चैत्य है, जो विभिन्न मूर्तियों से सुशोभित है।” जिससे यह पता चलता है कि यह स्तूप अपने काल में चरमोत्कर्ष पर था।

दृष्टि आईएएस इनपुट

■     धान्यकटक मूर्तिकला शैली का विकास अमरावती में होने के कारण इसे अमरावती शैली भी कहा गया। अमरावती दक्षिण भारत में कृष्णा नदी के निचले हिस्से में गुण्टूर ज़िले (आंध्र प्रदेश) के पास स्थित है।

अतः विकल्प (a) सही है।

स्रोत:

2. प्राचीन भारत के संदर्भ में, निम्नलिखित कथनों पर विचार कीजिये:

  1. स्तूप की संकल्पना मूलत: बौद्ध संकल्पना है।
  2. स्तूप आमतौर पर अवशेषों का निक्षेपागार होता था।
  3. बौद्ध परंपरा में स्तूप एक संकल्प-अर्पित या स्मारक संरचना होती थी।

उपर्युक्त में से कितने कथन सही हैं?

(a) केवल एक 
(b) 
केवल दो
(c) सभी तीन                         
(d) 
कोई भी नहीं

उत्तर:(b)

व्याख्या:

स्तूप की संकल्पना:

  • स्तूप वैदिक काल से भारत में प्रचलित शवाधान टीले थे। स्तूप शब्द का उल्लेख ऋग्वेद, अथर्ववेद, वाजसनेयी संहिता, तैत्तिरीय संहिता, पंचविंश ब्राह्मण में मिलता है। अतः कथन 1 सही नहीं है।
    • ऋग्वेद में वन में देवता वरुण द्वारा बनाए गए स्तूप का उल्लेख मिलता है, जिसकी कोई नींव (आधार) नहीं होती थी। ऋग्वेद में 'एस्तुका' शब्द का उपयोग भी इसी अर्थ में किया गया है वस्तुतः उस समय भूमि पर निर्मित ढेर/टीले को स्तूप के नाम से जाना जाता था।
  • सामान्यतः यह अवशेषों का निक्षेपागार था जिसमें मृतकों के अवशेष एवं राख को रखा जाता था। वैदिक परंपरा के स्तूपों को बौद्धों द्वारा लोकप्रिय बनाया गया। अतः कथन 2 सही है।
  • अवदान सतक, महावदान एवं स्तूपवदान जैसे बौद्ध ग्रंथों से स्तूपों के स्मारक पहलूओं का उल्लेख किया गया है। यहीं तक कि रायपसेणिय सुत्त (Raya Pasenaiya Sutta) जैसे जैन साहित्य में भी इसका उल्लेख मिलता है। मोक्ष प्राप्ति के उद्देश्य से भगवान की पूजा करने की सामान्य जन की गहरी आस्था के परिणामस्वरूप स्तूप ने आगे चलकर अपना संकल्प-अर्पित संरचना स्वरुप प्राप्त किया। अतः कथन 3 सही है।
  • अतः विकल्प (b) सही है।

स्रोत:

3. प्राचीन दक्षिण भारत के संदर्भ में, कोरकई, पूमपुहार और मुचीरि किस रूप में सुविख्यात थे?

(a)  राजधानी नगर       
(b)  पत्तन
(c)  लोह और इस्पात निर्माण के केंद्र
(d)  जैन तीर्थंकरों के चैत्य

उत्तर:(b)

व्याख्या:

कोरकई:

  • कोरकई बंगाल की खाड़ी के तट पर थमिराबरानी नदी के मुहाने के समीप स्थित पांड्यों का पत्तन था।
  • इस पत्तन के समीप गंगा घाटी के साथ-साथ प्राचीन रोमन सभ्यताओं के साथ विकसित व्यापार के प्रमाण प्राप्त हुए हैं। प्रथम शताब्दी ईस्वी में लिखी गई "पेरिप्लस ऑफ द एरिथ्रियन सी" नामक पुस्तक में तमिलनाडु के अन्य पत्तनों के साथ कोरकई का भी उल्लेख है।

पूमपुहार:

  • पूमपुहार, तमिलनाडु के मयिलादुत्रयी/माइलादुत्रयी/माइलादुत्रयी ज़िले का एक कस्बा है।
  • यह एक समृद्ध प्राचीन पत्तन नगर था जिसे कावेरी पूमपट्टिनम के नाम से जाना जाता था, जो कुछ समय के लिये तमिलकम में प्रारंभिक चोल राजाओं की राजधानी भी थी।
  • पुहार कावेरी नदी के मुहाने के समीप समुद्र तट पर स्थित है।

मुज़िरिस /मुचीरि:

  • मुचीरि, केरल का एक पत्तन नगर था।
  • यह विश्व के सबसे प्राचीन पत्तनों में से एक था।
  • संगम साहित्य में मुचीरि पत्तन पर काली मिर्च के लिये आने वाले रोमन जहाज़ों का वर्णन मिलता है।
  • वर्ष 1341 में मालाबार तट पर पेरियार नदी बेसिन में बाढ़ आने के कारण मुचीरि पत्तन विनष्ट हो गया था।
  • इस पत्तन के अवशेषों को भारत की सबसे बड़ी संरक्षण परियोजनाओं में से एक- मुज़िरिस विरासत परियोजना, के तहत संरक्षित किया जा रहा है।

अतः विकल्प (b) सही है।

स्रोत:

4. निम्नलिखित में से कौन-सा एक संगम कविता यथावर्णित ‘वटकिरुतल’ की प्रथा को स्पष्ट करता है।

(a)  राजाओं द्वारा महिला अंगरक्षिकाओं को नियुक्त करना         
(b) 
राजदरबारों में विद्वानों का, धर्म और दर्शन के विषयों पर विचार-विमर्श करने हेतु, एकत्र होना
(c)  किशोरियों द्वारा कृषि क्षेत्रों की निगरानी करना और चिड़ियों तथा पशुओं को भगाना
(d)  युद्ध में पराजित राजा का आमरण अनशन आनुष्ठानिक मृत्युवरण करना

उत्तर:(d)

व्याख्या:

  • वटकिरूतल, आमरण अनशन करने का एक तमिल अनुष्ठान था। यह विशेष रूप से संगम काल में व्यापक रूप से प्रचलित था। तमिल राजा अपने सम्मान एवं प्रतिष्ठा को बचाने के लिये उत्तर दिशा के सन्मुख यह प्रतिज्ञा करते थे कि वे युद्ध में कभी भी अपनी मृत्यु का सामना करने से (वटकिरूतल) पीछे नहीं हटेंगे।
  • यह अनुष्ठान या तो अकेले अथवा बंदी राजा के समर्थकों द्वारा एक समूह के रूप में किया जाता था।

अतः विकल्प (d) सही है।

स्रोत:

5. निम्नलिखित साम्राज्यों पर विचार कीजिये:

  1. होयसल
  2. गहड़वाल
  3. काकतीय
  4. यादव

उपर्युक्त में से कितने साम्राज्यों ने अपने राज आद्य आठवीं शताब्दी ईस्वी में स्थापित किये?

(a) केवल एक         
(b) केवल दो
(c) केवल तीन        
(d) किसी ने नहीं

उत्तर: (d)

व्याख्या:

होयसल:

  • होयसल साम्राज्य उन शक्तिशाली साम्राज्यों में से एक था जिसने 10वीं से  14वीं शताब्दी के मध्य दक्षिण भारत में शासन किया था।
  • होयसल साम्राज्य की राजधानी आरंभ में बेलूर में अवस्थित थी जिसे बाद में हलेबिडू में स्थानांतरित किया गया था।
  • होयसल साम्राज्य के शासनकाल से दक्षिण भारतीय कला, वास्तुकला एवं धर्म का विकास हुआ तथा इसकी विरासत मुख्य रूप से होयसल वास्तुकला में निहित है।

गहड़वाल:

  • गहड़वाल राजवंश, 12वीं-13वीं शताब्दी में हुए मुस्लिम आक्रमण से पहले उत्तर भारत का एक साम्राज्य था।
  • इसकी कालावधि 11वीं शताब्दी के उत्तरार्द्ध से लेकर 13वीं शताब्दी के मध्य तक थी जिसमें प्रारंभिक मध्यकालीन उत्तर भारतीय राजनीति की सभी विशेषताओं जैसे- वंशवादी प्रतिद्वंद्विता और गठबंधन, सामंती राज्य संरचना, ब्राह्मणवादी सामाजिक विचारधारा पर पूर्ण निर्भरता तथा बाह्य आक्रमण के प्रति सुभेद्यता आदि शामिल हैं।

काकतीय और यादव:

  • 13वीं से 15वीं शताब्दी तक का दक्षिण भारतीय इतिहास दो अलग-अलग चरणों में विभाजित है: i) 13वीं शताब्दी की शुरुआत को चोल एवं चालुक्य साम्राज्यों के विघटन के रूप में चिह्नित किया जाता है। चोलों के पतन के बाद दक्षिण में पांड्य एवं होयसल का उदय हुआ तथा इस क्षेत्र के उत्तर में काकतीय एवं यादवों का उदय, चालुक्य शक्ति के पतन के परिणामस्वरूप हुआ। ये राज्य एक शताब्दी से भी अधिक तक अस्तित्व में रहे।
  • अतः विकल्प (d) सही है।

स्रोत:

6. प्राचीन भारतीय इतिहास के संदर्भ में, निम्नलिखित युग्मों पर विचार कीजिये:

   साहित्यिक कृति         रचनाकार

  1. देवीचंद्रगुप्त          :  बिल्हण
  2. हम्मीर-महाकाव्य    :  नयचंद्र सूरि
  3. मिलिंद-पन्ह          :  नागार्जुन
  4. नीतिवाक्यामृत       :  सोमदेव सूरि

उपर्युक्त युग्मों में से कितने सही सुमेलित है?

(a)  केवल एक                        
(b)  केवल दो
(c)  केवल तीन 
(d)  सभी चार

उत्तर:(c)

व्याख्या:

साहित्यिक कृति

रचनाकार

देवीचंद्रगुप्त

विशाखदत्त

हम्मीर-महाकाव्य

नयचंद्र सूरि

मिलिंद-पन्ह

नागार्जुन

नीतिवाक्यामृत

सोमदेव सूरि

अत: विकल्प (C) सही है।

स्रोत:

7. ‘‘आत्मा केवल जंतु और पादप जीवन की संपदा नहीं है, बल्कि शिलाओं, प्रवाहित जलधाराओं और अन्य अनेक प्राकृतिक वस्तुओं की भी है, जिन्हें अन्य धार्मिक संप्रदाय जीवित नहीं मानते।’’

उपर्युक्त कथन प्राचीन भारत के निम्नलिखित में से किस एक धार्मिक संप्रदाय के एक मूलभूत विश्वास को प्रतिबिंबित करता है?

(a)  बौद्ध परंपरा                   
(b)  जैन परंपरा
(c)  शैव परंपरा                       
(d)  वैष्णव परंपरा

उत्तर: (b)

व्याख्या:

  • जैन परंपरा के अंतर्गत माना गया है कि सभी प्राणियों में एक तत्त्व निहित है जिसे आत्मा कहते हैं। जैसा कि आधुनिक जैन आचार्य, गुरुदेव चित्रभानु लिखते हैं, “ब्रह्मांड केवल मानवता के लिये नहीं है; यह सभी जीवित प्राणियों के विकास का क्षेत्र है। जाति, रंग, पंथ, या राष्ट्रीयता के आधार पर बिना किसी भेदभाव के सभी का जीवन पवित्र है छोटी चींटी या सूक्ष्मतम जीव सहित सभी स्तरों पर मौजूद जीवों का जीवन पवित्र है”।
  • पृथ्वी, जल, अग्नि, वायु और पौधों जैसी स्थिर इकाइयों में भी आत्माएँ होती हैं - जिनमें से सभी का एक ही अर्थ है, स्पर्श की भावना। इसके साथ ही गतिशील इकाइयों में आत्माओं के संवेदी अंगों में भिन्नताएँ,जैसे: कीट में दो इंद्रियाँ (स्पर्श और स्वाद), चींटी में तीन इंद्रियाँ (स्पर्श, स्वाद और गंध), मधुमक्खी में चार इंद्रियाँ (स्पर्श, स्वाद, गंध और दृष्टि) तथा जानवरों और मानव में पाँच इंद्रियाँ (स्पर्श, स्वाद, गंध, दृष्टि और श्रवण) होती हैं।
  • अतः विकल्प (b) सही है।

स्रोत:

8. विजयनगर साम्राज्य के इनमें से किस एक शासक ने तुंगभद्रा नदी पर एक विशाल बाँध निर्मित किया और नदी से राजधानी नगर तक कई किलोमीटर लंबी एक नहर और कुल्या का निर्माण किया?

(a) देवराय प्रथम                   
(b) 
मल्लिकार्जुन
(c) वीर विजय                       
(d) 
विरूपाक्ष

उत्तर:(a)

व्याख्या:

देवराय ने तुंगभद्रा नदी पर एक बाँध का निर्माण करवाया तथा इस बाँध से नहरों के माध्यम से नगरों एवं गाँवों को पेय/सिंचाई हेतु जलापूर्ति होती थी। अतः विकल्प (a) सही है।

स्रोत:

9. मध्यकालीन गुजरात के इनमें से किस एक शासक ने द्वीव को पुर्तगालियों के सुपुर्द कर दिया?

(a) अहमद शाह                     
(b) महमूद बेगढ़ा
(c) बहादुर शाह
(d) मुहम्मद शाह

उत्तर: (c)

व्याख्या:

  • 16वीं शताब्दी के आरंभ में मुगल सम्राट हुमायूँ द्वारा के आक्रमण से गुजरात का सुल्तान बहादुर शाह अत्यधिक दबाव में आ गया।
  • तत्कालीन पारिस्थितियों में उसने पुर्तगालियों के साथ मैत्रीपूर्ण संबंध बनाने का निर्णय लिया, जिनके पास उस समय सक्रिय समुद्री शक्ति थी जिसके जरिये 15वीं शताब्दी के अंत में वे भारत पहुँचे थे।
  • वर्ष 1534 में, शाह ने पुर्तगालियों के साथ बसीन की संधि पर हस्ताक्षर किया, आगे चलकर उसने दियु (Diu) के साथ-साथ साम्राज्य के अन्य क्षेत्रों जैसे कि वसई तथा वर्तमान मुंबई स्थित द्वीपों को भी पुर्तगालियों को सौंप दिया। वर्ष 1559 में पुर्तगालियों ने शाह से दमन को प्राप्त किया।

स्रोत:

10. निम्नलिखित में से किस एक एक्ट द्वारा बंगाल के गवर्नर जनरल को भारत के गवर्नर जनरल के रूप में नामित किया गया था?

(a) रेगुलेटिंग ऐक्ट 
(b) पिट्स इंडिया ऐक्ट
(c) 1793 का चार्टर ऐक्ट
(d) 1833 का चार्टर ऐक्ट

उत्तर:(d)

व्याख्या:

चार्टर अधिनियम, 1833

  • यह अधिनियम ब्रिटिश भारत में केंद्रीकरण की दिशा में अंतिम निर्णायक कदम था।
  • इसके द्वारा बंगाल के गवर्नर-जनरल को भारत का गवर्नर जनरल बनाने के साथ उसे सभी नागरिक और सैन्य शक्तियाँ प्रदान की गईं। इस प्रकार इस अधिनियम द्वारा पहली बार तत्कालीन भारत में ब्रिटिशों के अधीन संपूर्ण क्षेत्र को भारत सरकार के अधिकार क्षेत्र में लाया गया। इस अधिनियम के तहत लॉर्ड विलियम बेंटिक, भारत के पहले गवर्नर-जनरल बना। अतः विकल्प (d) सही है।

स्रोत: Laxmikanth

11. सारभूत रूप में, ‘विधि की सम्यक प्रक्रिया’ का अभिप्राय क्या है?

(a)  नैसर्गिक न्याय का सिद्धांत         
(b) 
विधि द्वारा स्थापित पद्धति
(c)  विधि की निष्पक्ष प्रयुक्ति
(d)  विधि के समक्ष समता

उत्तर:(c)

व्याख्या:

  • कानून की सम्यक प्रक्रिया का सिद्धांत न केवल इस बात पर विचार करता है कि क्या कोई कानून किसी व्यक्ति को उसके जीवन और व्यक्तिगत स्वतंत्रता से वंचित करने हेतु मौजूद है, बल्कि यह भी कि क्या कानून उचित, न्यायपूर्ण एवं मनमाना तो नहीं है।
  • कानून की उचित प्रक्रिया एक संवैधानिक गारंटी है जो सरकारों को नागरिकों को अपमानजनक तरीके से प्रभावित करने से रोकती है। उचित प्रक्रिया की आधुनिक परिभाषाओं में स्वतंत्रता के हितों की विविधता शामिल है, जो कानूनों एवं विनियमों का उल्लंघन नहीं कर सकते हैं, साथ ही प्रक्रियात्मक नियम जो न्यायालयों को लोगों की व्यक्तिगत स्वतंत्रता की रक्षा हेतु बनाए रखना चाहिये।
  • इसकी उत्पत्ति किंग जॉन के मैग्ना कार्टा के अध्याय 39 में देखी जा सकती है, जिसमें कहा गया है कि किसी भी फ्रीमैन को उसकी संपत्ति से वंचित नहीं किया जाएगा या नुकसान नहीं पहुँचाया जाएगा जब तक कि "देश के कानून द्वारा" एक वाक्यांश जो न्यायालय के पारंपरिक प्रथाओं को संदर्भित करता है। यह वाक्यांश "कानून की उचित प्रक्रिया" मूल रूप से किंग एडवर्ड III के 1354 के कानून में मैग्ना कार्टा के "देश के कानून" के प्रतिस्थापन के रूप में उभरा, जिसने मैग्ना कार्टा की विषय स्वतंत्रता की गारंटी को दोहराया है।
  • अतः विकल्प (c) सही है।

स्रोत…

12. निम्नलिखित कथनों पर विचार कीजिये:

    कथन-I: भारत में, दैनंदिन जेल प्रशासन के लिये, जेलों का अनुरक्षण राज्य सरकारें अपने नियमों और विनियमों द्वारा करती हैं।

    कथन-II: भारत में, जेल का नियंत्रण जेल अधिनियम, 1894 द्वारा किया जाता है, जिससे जेल का विषय स्पष्ट रूप से प्रांतीय सरकारों के नियंत्रण में रहा।

उपर्युक्त कथनों के संबंध में निम्नलिखित में से कौन-सा एक सही है?

(a)  कथन-I और कथन-II दोनों सही हैं तथा कथन-II, कथन-I की सही व्याख्या है
(b)  कथन-I और कथन-II दोनों सही है तथा कथन-II, कथन-1 की सही व्याख्या नहीं है
(c)  कथन-I सही है किन्तु कथन-II गलत है
(d)  कथन-I गलत है किन्तु कथन-II सही है

उत्तर:(a)

व्याख्या:

  • भारतीय संविधान की सातवीं अनुसूची की सूची II की प्रविष्टि संख्या 4 के तहत 'जेल'/'उसमें बंद व्यक्ति' "राज्य-सूची" का एक विषय है। जेलों और कैदियों का प्रशासन एवं प्रबंधन संबंधित राज्य सरकारों की ज़िम्मेदारी है जो इस संबंध में उचित कार्रवाई करने में सक्षम हैं। अतः कथन 1 सही है।
  • हालाँकि आपराधिक न्याय प्रणाली में जेलों के महत्त्व को देखते हुए, गृह मंत्रालय जेल प्रशासन से संबंधित विभिन्न मुद्दों पर राज्यों तथा केंद्रशासित प्रदेशों को नियमित मार्गदर्शन और सहायता प्रदान करता रहा है।
  • कारागार अधिनियम 1894, जो कारागारों/ज़ेलों को नियंत्रित करता है, के तहत जेलों का प्रबंधन एवं प्रशासन राज्य सरकारों के क्षेत्राधिकार में आता है। अतः कथन 2 सही है।
  • इसलिए, कथन-1 और कथन-2 दोनों सही हैं और कथन-2, कथन-1 की सही व्याख्या है

स्रोत:

13. निम्नलिखित में से कौन-सा एक कथन किसी देश के ‘संविधान’के मुख्य प्रयोजन को सर्वोत्तम रूप से प्रतिबिंबित करता है?

(a)  यह आवश्यक विधियों के निर्माण के उद्देश्य का निर्धारण करता है।
(b)  यह राजनीतिक पदों और सरकार के सृजन को सुकर बनाता है।
(c)  यह सरकार की शक्तियों को परिभाषित और सीमाबद्ध करता है।
(d)  यह सामाजिक न्याय, सामाजिक समता और सामाजिक सुरक्षा को प्रतिभूत करता है।

उत्तर:(c)

व्याख्या:

  • किसी भी संविधान का मुख्य उद्देश्य एक सरकार के मौलिक सिद्धांतों, संरचना एवं कार्यों को स्थापित करना और एक देश के भीतर व्यक्तियों के अधिकारों एवं स्वतंत्रता को परिभाषित करना है। संविधान भूमि के सर्वोच्च कानून (supreme law of the land) के रूप में कार्य करता है और शासन हेतु एक रूपरेखा प्रदान करता है, शक्ति संतुलन सुनिश्चित करता है, व्यक्तिगत अधिकारों की रक्षा करता है तथा राज्य के कामकाज या कार्यप्रणाली का मार्गदर्शन करता है

अतः विकल्प (c) सही है।

स्रोत:

14. भारत में, निम्नलिखित में से किस संविधान संशोधन के लिये व्यापक रूप से यह माना गया है कि उसे मूल अधिकारों की न्यायिक व्याख्या को अधिभूत करने के लिये अधिनियमित किया गया?

(a) पहला संशोधन 
(b) 42वाँ संशोधन
(c) 44वाँ संशोधन
(d) 86वाँ संशोधन

उत्तर: a/b

व्याख्या:

  • प्रथम संविधान संशोधन अधिनियम, 1951:
    • मामलों में शामिल मुद्दों में भाषण/वाक् की स्वतंत्रता, ज़मींदारी भूमि का अधिग्रहण, व्यापार का राज्य एकाधिकार आदि शामिल थे।
    • वाक् एवं अभिव्यक्ति की स्वतंत्रता पर प्रतिबंध के तीन और आधार जोड़े गए: लोक व्यवस्था, विदेशी राज्यों के साथ मैत्रीपूर्ण संबंध और अपराध के लिये उकसाना। साथ ही, इसने प्रतिबंधों को ‘युक्ति-युक्त’ और इस प्रकार, प्रकृति में न्यायसंगत बना दिया।
  • 42वाँ संविधान संशोधन अधिनियम, 1976:
    • इसने संवैधानिक संशोधनों को न्यायिक जाँच के दायरे से बाहर कर दिया।
    • सर्वोच्च न्यायालय और उच्च न्यायालयों की न्यायिक समीक्षा और रिट क्षेत्राधिकार की शक्ति को कम कर दिया।
    • यह प्रावधान किया गया कि निदेशक सिद्धांतों के कार्यान्वयन के लिये बनाए गए कानूनों को कुछ मौलिक अधिकारों के उल्लंघन के आधार पर न्यायालयों द्वारा अमान्य घोषित नहीं किया जा सकता है।

अतः विकल्प a/b सही है।

स्रोत:

15. भारत के निम्नलिखित संगठनों / निकायों पर विचार कीजिये :

  1. राष्ट्रीय पिछड़ा वर्ग आयोग
  2. राष्ट्रीय मानव अधिकार आयोग
  3. राष्ट्रीय विधि आयोग
  4. राष्ट्रीय उपभोक्ता विवाद निवारण आयोग

उपर्युक्त में से कितने सांविधानिक निकाय हैं?

(a) केवल एक 
(b) केवल दो
(c) केवल तीन 
(d) सभी चार

उत्तर:(a)

व्याख्या:

  • राष्ट्रीय पिछड़ा वर्ग आयोग (National Commission for Backward Classes- NCBC) का गठन शुरू में केंद्र सरकार द्वारा राष्ट्रीय पिछड़ा वर्ग आयोग अधिनियम, 1993 द्वारा किया गया था और अब तक आयोग को वर्ष 2016 तक 7 बार पुनर्गठित किया गया था। राष्ट्रीय पिछड़ा वर्ग आयोग अधिनियम, 1993 को राष्ट्रीय पिछड़ा वर्ग आयोग (निरसन) अधिनियम, 2018 के माध्यम से निरस्त कर दिया गया है। आयोग को संवैधानिक दर्जा दिया गया है एवं "संविधान ( 102वाँ संशोधन) अधिनियम, 2018" अधिनियम के माध्यम से गठित किया गया है।
  • भारत के राष्ट्रीय मानवाधिकार आयोग (National Human Rights Commission- NHRC) की स्थापना 12 अक्तूबर, 1993 को हुई थी। मानवाधिकार संरक्षण अधिनियम (PHRA), 1993 की स्थापना संविधि के तहत की गई है, जिसे मानव अधिकार संरक्षण (संशोधन) अधिनियम, 2006 द्वारा संशोधित किया गया है।
  • राष्ट्रीय उपभोक्ता विवाद निवारण आयोग (NCDRC), भारत में एक अर्द्ध-न्यायिक आयोग है जिसे वर्ष 1988 में उपभोक्ता संरक्षण अधिनियम, 1986 के तहत स्थापित किया गया था।
  • भारत का विधि आयोग एक गैर-सांविधिक निकाय है और इसका गठन भारत सरकार, विधि एवं न्याय मंत्रालय, विधि कार्य विभाग की एक अधिसूचना द्वारा विधि के क्षेत्र में अनुसंधान करने हेतु एक निश्चित विचारार्थ विषयों के साथ किया जाता है, साथ ही आयोग अपने विचारार्थ विषयों के अनुसार सरकार (रिपोर्ट के रूप में) को सिफारिशें करता है।

अतः विकल्प (a) सही है।

स्रोत…

16. निम्नलिखित कथनों पर विचार कीजिये:

  1. यदि भारत के राष्ट्रपति का निर्वाचन भारत के उच्चतम न्यायालय द्वारा शून्य घोषित कर दिया जाता है, तो ऐसे विनिश्चय की तिथि से पूर्व राष्ट्रपति के पद के कर्तव्यों के निष्पादन में राष्ट्रपति के द्वारा किये गए सभी कृत्य अविधिमान्य हो जाते हैं।
  2. भारत के राष्ट्रपति के पद के लिये निर्वाचन इस आधार पर मुल्तवी किया जा सकता है कि कुछ विधान-सभाएँ विघटित हो गई हैं और उनके निर्वाचन अभी होने शेष हैं।
  3. कोई विधेयक भारत के राष्ट्रपति को प्रस्तुत किये जाने पर, संविधान द्वारा विहित की गई समय-सीमा के अंदर राष्ट्रपति को अपनी अनुमति देनी होती है।

उपर्युक्त में से कितने कथन सही हैं?

(a) केवल एक 
(b) केवल दो
(c) सभी तीन                         
(d) कोई भी नहीं

उत्तर:(d)

व्याख्या

  • यदि राष्ट्रपति या उपराष्ट्रपति के रूप में किसी व्यक्ति का निर्वाचन सर्वोच्च न्यायालय द्वारा शून्य घोषित किया जाता है तो उसके द्वारा राष्ट्रपति या उपराष्ट्रपति के कार्यालय की शक्तियों और कर्तव्यों के प्रयोग एवं किए गए कार्य, सर्वोच्च न्यायालय के निर्णय की तारीख से पूर्व उस घोषणा के कारण अमान्य नहीं किया जाएगा। अतः कथन 1 सही नहीं है।
  • जब विधानसभा भंग हो जाती है तो सदस्य राष्ट्रपति निर्वाचन में मतदान करने के योग्य नहीं रह जाते हैं, भले ही राष्ट्रपति निर्वाचन से पूर्व विघटित विधानसभा के नवीन निर्वाचन न हुए हों। इस प्रकार राष्ट्रपति का निर्वाचन इस आधार पर स्थगित नहीं किया जाएगा कि कुछ विधानसभाएँ भंग कर दी गई हैं। अतः कथन 2 सही नहीं है।
  • विधेयकों पर सहमतिः जब कोई विधेयक संसद के सदनों द्वारा पारित कर दिया जाता है, तो उसे राष्ट्रपति के समक्ष प्रस्तुत किया जाता है ऐसे में राष्ट्रपति या तो यह घोषणा करेगा कि वह विधेयक पर अपनी सहमति देता है या वह उस पर सहमति रोक लेता है (बशर्ते कि यह विधेयक धन विधेयक न हो)। राष्ट्रपति, विधेयक पेश करने के बाद सदनों को इस अनुरोध के साथ लौटा सकता है कि वे विधेयक या उसके किसी निर्दिष्ट प्रावधान पर पुनर्विचार करेंगे एवं विशेष रूप से विधेयक पर विचार करेंगे। जब कोई विधेयक इस प्रकार लौटाया जाता है तो सदन तदनुसार विधेयक पर पुनर्विचार करेगा एवं यदि विधेयक सदनों द्वारा संशोधन के साथ या बिना संशोधन के फिर से पारित किया जाता तो राष्ट्रपति वित्तीय मामलों से संबंधित मामलों में अपनी सहमति नहीं रोकेंगे। अतः कथन 3 सही नहीं है।

स्रोत:

17. भारतीय संसद में वित्त विधेयक और धन विधेयक के संदर्भ में, निम्नलिखित कथनों पर विचार कीजिये:

  1. जब लोक सभा वित्त विधेयक को राज्य सभा में भेजती है, तो राज्य सभा उस विधेयक को संशोधित या अस्वीकृत कर सकती है।
  2. जब लोक सभा धन विधेयक को राज्य सभा में भेजती है, तो राज्य सभा उस विधेयक को संशोधित या अस्वीकृत नहीं कर सकती, वह केवल अनुशंसाएँ कर सकती है।
  3. लोक सभा और राज्य सभा के बीच असहमति होने पर, धन विधेयक के लिये कोई संयुक्त बैठक नहीं होती, किन्तु वित्त विधेयक के लिए, संयुक्त बैठक आवश्यक होती है।

उपर्युक्त में से कितने कथन सही है?

(a) केवल एक                        
(b) 
केवल दो
(c) सभी तीन                         
(d) कोई भी नहीं

उत्तर:(a)

व्याख्या:

  • वित्त विधेयक संविधान के अनुच्छेद 110 (a) के तहत परिभाषित धन विधेयक है। इसे अनुच्छेद 112 के तहत वार्षिक वित्तीय विवरण (बजट) के एक भाग के रूप में प्रस्तुत किया जाता है।
  • राज्यसभा के पास धन विधेयक के संबंध में सीमित शक्तियाँ हैं। लोकसभा द्वारा पारित होने और राज्य सभा को प्रेषित होने के बाद यह धन विधेयक को अस्वीकार या संशोधित नहीं कर सकता है। इसे 14 दिनों के भीतर सिफारिशों के साथ या बिना सिफारिश के विधेयक को वापस करना होगा। राज्यसभा द्वारा की गई किसी या सभी सिफारिशों को स्वीकार या अस्वीकार करना लोकसभा के विवेक पर है। अतः कथन 2 सही है।
  • वित्त विधेयक के धन विधेयक की सभी शर्तों के अधीन होने के कारण, राज्यसभा केवल इस पर सिफारिशें कर सकती है। राज्यसभा वित्त विधेयक में न तो संशोधन कर सकती है और न ही इसे अस्वीकार नहीं कर सकती है (जैसा कि धन विधेयक के मामले में भी लागू होता है)। अतः कथन 1 सही नहीं है।
  • संयुक्त बैठक का प्रावधान केवल सामान्य विधेयकों या वित्तीय विधेयकों पर लागू होता है न कि धन विधेयकों (वित्त विधेयकों सहित) या संविधान संशोधन विधेयकों पर। अतः कथन 3 सही नहीं है।

स्रोत…

18. निम्नलिखित कथनों पर विचार कीजिये:

एक बार यदि केंद्र सरकार किसी क्षेत्र को ‘समुदाय प्रारक्षित’ अधिसूचित कर देती है, तो

  1. राज्य का मुख्य वन्यजीव वार्डन ऐसे वन का नियंत्रक प्राधिकारी बन जाता है।
  2. ऐसे क्षेत्र में शिकार की अनुमति नहीं होती है।
  3. ऐसे क्षेत्र के लोगों को गैर-इमारती लकड़ी वनोत्पाद को संग्रह करने की अनुमति होती है।
  4. ऐसे क्षेत्र के लोगों को पारंपरिक कृषि प्रथाओं की अनुमति होती है।

उपर्युक्त में से कितने कथन सही हैं?

(a) केवल एक                        
(b) केवल दो
(c) केवल तीन                       
(d) 
सभी चार

उत्तर:(b)

व्याख्या:

  • WLPA अधिनियम की धारा 33 के अनुसार केंद्र द्वारा सामुदायिक आरक्षित क्षेत्र के रूप में एक क्षेत्र को अधिसूचित करने के बाद राज्य का मुख्य वन्यजीव संरक्षक वन का शासकीय प्राधिकारी होता है, जिसकी सहमति क्षेत्र से संबंधित सभी निर्णयों के लिये आवश्यक होती है। अतः कथन 1 सही है।
  • एक वन को एक सामुदायिक रिज़र्व घोषित करने के बाद:
    • वहाँ लोगों को शिकार करने की अनुमति नहीं होती है। अतः कथन 2 सही है।
    • लोग गैर-इमारती वनोपज एकत्र नहीं कर सकते हैं। अतः कथन 3 सही नहीं है।
    • झूम खेती जैसी कृषि पद्धतियों के लिये लोगों को इसका उपयोग करने की अनुमति नहीं होती है। अतः कथन 4 सही नहीं है।

स्रोत…

19. भारत में ‘अनुसूचित क्षेत्र’ के संदर्भ में, निम्नलिखित कथनों पर विचार कीजिये:

  1. किसी राज्य के भीतर, किसी क्षेत्र की अनुसूचित क्षेत्र के रूप में अधिसूचना राष्ट्रपति के एक आदेश के माध्यम से होती है।
  2. अनुसूचित क्षेत्र के रूप में बनने वाली सबसे बड़ी प्रशासकीय इकाई ज़िला होता है और सबसे छोटी इकाई ब्लॉक में गाँवों का समूह होता है।
  3. संबंधित राज्यों के मुख्यमंत्रियों से अपेक्षित है कि वे राज्यों के अनुसूचित क्षेत्रों के प्रशासन के विषय में केंद्रीय गृह मंत्रालय को वार्षिक प्रतिवेदन दें।

उपर्युक्त में से कितने कथन सही हैं?

(a) केवल एक                       
(b) केवल दो
(c) सभी तीन                         
(d) 
कोई भी नहीं

उत्तर:(b)

व्याख्या:

  • "अनुसूचित क्षेत्र" वे हैं जो पाँचवीं अनुसूची के पैरा 6 (1) के तहत एक राष्ट्रपति के आदेश द्वारा निर्धारित किये गए हैं। इसमें कहा गया है "इस संविधान में, अभिव्यक्ति 'अनुसूचित क्षेत्र' ऐसे क्षेत्र अभिप्रेत हैं जिन्हें राष्ट्रपति आदेश द्वारा अनुसूचित क्षेत्र घोषित करें”।
  • किसी राज्य के संबंध में "अनुसूचित क्षेत्रों" का विनिर्देश संबंधित राज्य सरकार के परामर्श के बाद राष्ट्रपति के अधिसूचित आदेश द्वारा होता है। किसी भी परिवर्तन, वृद्धि, कमी, नए क्षेत्रों को शामिल करने या "अनुसूचित क्षेत्रों" से संबंधित किसी भी आदेश को रद्द करने के मामले में भी यही लागू होता है। अतः कथन 1 सही है।
  • अनुसूचित क्षेत्रों का गठन करने वाली सबसे बड़ी प्रशासनिक इकाई ज़िला और ब्लॉक स्तर पर ग्रामीण समूह है। अतः कथन 2 सही है।
  • संघ की कार्यकारी शक्ति उक्त क्षेत्रों के प्रशासन के संबंध में राज्य को निर्देश देने तक विस्तृत है। अनुसूचित क्षेत्रों के साथ प्रत्येक राज्य के राज्यपाल, राष्ट्रपति को एक वार्षिक प्रतिवेदन पर प्रस्तुत करेंगे, जो उस राज्य के भीतर अनुसूचित क्षेत्रों के प्रशासन की रूपरेखा तैयार करेंगे। अतः कथन 3 सही नहीं है।

स्रोत…

20. निम्नलिखित कथनों पर विचार कीजिये :

कथन-I: भारत के उच्चतम न्यायालय ने अपने कतिपय निर्णयों में यह विचारण किया है कि भारत के संविधान के अनुच्छेद 16(4) के अधीन बनाई गई आरक्षण नीतियाँ प्रशासन की दक्षता को बनाए रखने के लिये अनुच्छेद 335 द्वारा सीमित होगी।

कथन-II : भारत के संविधान का अनुच्छेद 335 ‘प्रशासन की दक्षतापद को परिभाषित करता है।

उपर्युक्त कथनों के बारे में, निम्नलिखित में से कौन-सा एक सही है?

(a)  कथन-I और कथन-II दोनों सही हैं तथा कथन-II, कथन-I की सही व्याख्या है।
(b)  कथन-I और कथन-II दोनों सही हैं तथा कथन-II, कथन-I की सही व्याख्या नहीं है।
(c)  कथन-I सही है किन्तु कथन-II गलत है।
(d)  कथन-I गलत है किन्तु कथन-II सही है।

उत्तर:(c)

व्याख्या:

  • भारत के सर्वोच्च न्यायालय ने आरक्षण पर पिछले सात दशकों के संवैधानिक फैसलों में विभिन्न आरक्षण नीतियों की वैधता का निर्णय करते हुए लगातार "दक्षता" और "योग्यता" की धारणाओं का उल्लेख किया है।
  • न्यायालय ने कई निर्णय दिये हैं (इंदिरा साहनी और अन्य बनाम भारत संघ एवं अन्य 1993; एम नागराज और अन्य बनाम भारत संघ एवं अन्य 2006) कि संविधान के अनुच्छेद 16(4)1 के तहत बनाई गई आरक्षण नीतियाँ अनुच्छेद 335 (2) द्वारा सीमित होंगी जो "प्रशासन की दक्षता बनाए रखने" का प्रावधान करती हैं। अतः कथन 1 सही है।
  • यह तब किया गया जब संविधान "प्रशासन की दक्षता" शब्द को परिभाषित नहीं करता है। अतः कथन 2 सही नहीं है।
  • सार्वजनिक सेवाओं और पदों पर नियुक्तियाँ करने में अनुसूचित जाति (Scheduled Castes- SC) और अनुसूचित जनजाति (Scheduled Tribes -ST) के दावों पर विचार करते हुए। यह तब किया गया जब संविधान "प्रशासन की दक्षता" शब्द को परिभाषित नहीं करता है।
  • जबकि सार्वजनिक सेवाओं और पदों पर नियुक्तियों में अनुसूचित जातियों (Scheduled Castes- SC) और अनुसूचित जनजातियों (Scheduled Tribes -ST) के दावों पर विचार किया गया है। यह तब किया गया जब संविधान "प्रशासन की दक्षता" शब्द को परिभाषित नहीं करता है।
  • अतः विकल्प (c) सही है।

स्रोत…

21. निम्नलिखित कथनों पर विचार कीजिये:

कथन-I भारत, अपने पास यूरेनियम निक्षेप (डिपॉजिट) होने के बावजूद, अपने अधिकांश विद्युत् उत्पादन के लिये कोयले पर निर्भर करता है।

कथन-II: विद्युत् उत्पादन के लिये कम से कम 60% तक समृद्ध (एन्रिच्ड) यूरेनियम का होना आवश्यक है।

उपर्युक्त कथनों के बारे में, निम्नलिखित में से कौन-सा एक सही है?

(a)  कथन-I और कथन-II दोनों सही है कथन-II, कथन-I की सही व्याख्या है।
(b)  कथन-I और कथन-II दोनों सही तथा तथा कथन-II, कथन-I की सही व्याख्या नहीं है।
(c)  कथन-I सही है किन्तु कथन-II गलत है।
(d)  कथन-I गलत है किन्तु कथन-II सही है।

उत्तर:(c)

व्याख्या:

  • भारत में विद्युत का उत्पादन परंपरागत (तापीय, परमाणु और जलीय) एवं नवीकरणीय स्रोतों (पवन, सौर, जैव आदि) से होता है।
  • हालाँकि, कोयला आधारित विद्युत सयंत्र विद्युत उत्पादन का सबसे प्रमुख प्रमुख स्रोत हैं, जो कुल विद्युत उत्पादन में लगभग 75% की हिस्सेदारी रखते हैं। अतः कथन- I सही है।
  • हालाँकि, विद्युत के उत्पादन के लिये कम से कम 60% तक समृद्ध यूरेनियम की आवश्यकता नहीं है। यूरेनियम संवर्धन यूरेनियम-235 की सांद्रता बढ़ाने की प्रक्रिया है, जो यूरेनियम का विखंडनीय समस्थानिक है जो कि परमाणु शृंखला प्रतिक्रिया को बनाए रख सकता है।
  • असैन्य परमाणु ऊर्जा संयंत्रों के लिये, यूरेनियम को आमतौर पर यूरेनियम-235 के लगभग 3-5% तक समृद्ध किया जाता है, जो हल्के जल रिएक्टरों के लिये पर्याप्त है जो आमतौर पर विद्युत उत्पादन के लिये उपयोग किया जाता है। अतः कथन-II सही नहीं है।

अत: विकल्प (c) सही है।

स्रोत:

22. निम्नलिखित कथनों पर विचार कीजिये:

कथन-I: शिशुधानी-स्तनी (मार्सूपियल) प्राकृतिक रूप से भारत में नहीं होते।

कथन-II : शिशुधानी-स्तनी केवल परभक्षी-रहित पर्वतीय घास स्थलों में ही पनप सकते हैं।

उपर्युक्त कथनों के बारे में, निम्नलिखित में से कौन-सा एक सही है?

(a)  कथन-I और कथन-II दोनों सही है तथा कथन-II, कथन-I की सही व्याख्या है।
(b)  कथन-I और कथन-II दोनों सही हैं तथा कथन-II, कथन-I की सही व्याख्या नहीं है।
(c)  कथन-I सही है किन्तु कथन-II गलत है
(d)  कथन-I गलत है किन्तु कथन-II सही है

उत्तर:(c)

व्याख्या:

  • शिशुधानी-स्तनी (मार्सुपियल) स्तनधारी मार्सुपियालिया वर्ग के सदस्य हैं, जो समय से पहले जन्म और माँ के निचले पेट पर स्तन से जुड़े हुए नवजात शिशु के निरंतर विकास की विशेषता रखते हैं।
  • उन्हें शिशुधानी वाले स्तनधारियों के रूप में भी जाना जाता है, क्योंकि वयस्क मादाओं में शिशुधानी होती है।
    • युवा शिशुधानी-स्तनी (जिन्हें जॉयज़ कहा जाता है) अपना अधिकांश प्रारंभिक विकास अपनी माँ के शरीर के बाहर एक थैली में करते हैं।
  • शिशुधानी-स्तनी भारत में प्राक्रतिक रूप से नहीं पाए जाते हैं। धानी की 330 से अधिक प्रजातियाँ हैं। उनमें से लगभग दो-तिहाई ऑस्ट्रेलिया में निवास करते हैं। जबकि अन्य मुख्यतः दक्षिण अमेरिका में पाए हैं। अतः कथन-I सही है।
  • हालाँकि,शिशुधानी-स्तनी विभिन्न आवासों में पनप सकते हैं,जो कि न केवल परभक्षी रहित पर्वतीय घास के मैदानों में रह सकते बल्कि यह वनीय क्षेत्रों में भी रह सकते हैं।
    • उदाहरण के लिये, कुछ शिशुधानी-स्तनी की कुछ आबादी वर्षावनों, रेगिस्तानों, वुडलैंड्स और सवाना में भी पाई जाती है। अतः कथन- II सही नहीं है।
  • अतः विकल्प (C) सही है।

स्रोत:

23. ‘संक्रामक (इन्वेसिव) जीव-जाति (स्पीशीज़) विशेषज्ञ समूह’(जो वैश्विक संक्रामक जीव-जाति डेटाबेस विकसित करता है) निम्नलिखित में से किस एक संगठन से संबंधित है?

(a)  अंतर्राष्ट्रीय प्रकृति संरक्षण संघ (इंटरनैशनल यूनियन फॉर कंज़र्वेशन ऑफ नेचर)
(b)  संयुक्त राष्ट्र पर्यावरण कार्यक्रम (यूनाइटेड नेशंस एन्वाइरनमेंट प्रोग्राम)
(c)  संयुक्त राष्ट्र का पर्यावरण एवं विकास पर विश्व आयोग (यूनाइटेड नेशंस वर्ल्ड कमीशन फॉर एन्वाइरनमेंट एंड  डेवेलप्मेंट)
(d)  प्रकृति के लिये विश्वव्यापी निधि (वर्ल्डवाइड फंड फॉर नेचर)

उत्तर: (a)

व्याख्या:

  • संक्रामक जीव-जाति विशेषज्ञ समूह (ISSG) आक्रामक प्रजातियों पर वैज्ञानिक और नीति विशेषज्ञों का एक वैश्विक नेटवर्क है, जिसका गठन अंतर्राष्ट्रीय प्रकृति संरक्षण संघ (IUCN) के प्रजाति उत्तरजीविता आयोग (SSC) के तत्त्वाधान में आयोजित किया गया है। अतः विकल्प (a) सही है।
  • इसकी स्थापना वर्ष 1994 में हुई थी।
  • ISSG ग्लोबल इनवेसिव स्पीशीज़ डेटाबेस (GISD) का प्रबंधन करता है, जो विश्व भर में आक्रामक विदेशी प्रजातियों के बारे में जानकारी प्रदान करता है। ISSG अन्य ऑनलाइन संसाधनों जैसे कि एलियंस-एल लिस्टसर्व, द इनवेसिव स्पीशीज़ कम्पेंडियम, द ग्लोबल रजिस्टर ऑफ इंट्रोड्यूस्ड एंड इनवेसिव स्पीशीज़ का भी रखरखाव करता है।

स्रोत:

24. निम्नलिखित प्राणिजात पर विचार कीजिये:

  1. सिंह-पुच्छी मकाॅक
  2. मालाबार सिवेट
  3. सांभर हिरण

उपर्युक्त में से कितने आमतौर पर रात्रिचर हैं या सूर्यास्त के बाद अधिक सक्रिय होते हैं?

(a) केवल एक                        
(b) केवल दो
(c) सभी तीन                         
(d) 
कोई भी नहीं

उत्तर:(b)

व्याख्या:

  • सिंह पुच्छी मकाॅक कोई निशाचर जानवर नहीं है। यह एक वृक्षवासी और दिनचर प्राणी है, जो रात में पेड़ों पर सोते हैं (आमतौर पर,उच्च वर्षावन कैनोपी में)। ये मकाक प्रादेशिक होने के साथ-साथ संचरण शाली जानवर हैं। अतः 1 सही नहीं है।
  • मालाबार सिवेट मुख्य रूप से निशाचर जानवर है। यह एक छोटा, माँसाहारी स्तनपायी है जो मूलतः भारत के पश्चिमी घाट के क्षेत्र में पाए जाते हैं।
    • इसकी प्रकृति एकांत और गोपनीय अर्थात् इन्हें वनों में खोजना एक चुनौतीपूर्ण कार्य है। इसका रात्रिचर व्यवहार शिकारियों से बचने में मदद करता है। अत: 2 सही है।
  • सांभर हिरण निशाचर होते हैं। वे आमतौर पर सुगंध और फुट स्टैम्पिंग द्वारा संवाद करते हैं। ये पर्णपाती झाड़ियों और घास के सघन आवरण को पसंद करते हैं। अत: 3 सही है।

अतः विकल्प (b) सही है।

स्रोत:

25. निम्नलिखित में से कौन-सा जीव अपने सगे-संबंधियों को अपने खाद्य के स्रोत की दिशा और दूरी इंगित करने के लिये दोलन नृत्य (वैगल डांस) करता है?

(a) तितली
(b) 
व्याध पतंग (ड्रैगनफ्लाई)
(c) मधुमक्खी                         
(d) बर्र

उत्तर:(c)

व्याख्या:

  • मधुमक्खियाँ अपनी कॉलोनी के साथियों को खाद्य स्रोतों या नए घोंसले के स्थान के बारे में बताने के लिये दोलन नृत्य करती हैं। अतः विकल्प (c) सही है।
  • इस नृत्य में मधुमक्खियाँ आठ संख्या का पैटर्न बनाती हैं। गुरुत्व के सापेक्ष इनकी गति का कोण सूर्य के सापेक्ष खाद्य स्रोत की दिशा को इंगित करता है और सीधी गति की अवधि छत्ते से भोजन स्रोत की दूरी को इंगित करती है।
  • वृद्ध मधुमक्खियों की तुलना में युवा मधुमक्खियों द्वारा दोलन नृत्य में अधिक त्रुटियों की संभावना रहती है।
    • इससे पता चलता है कि युवा मधुमक्खियाँ वृद्ध मधुमक्खियों से सीखती हैं। इस प्रकार मधुमक्खियों का दोलन नृत्य मानव भाषा या सोंगबर्ड संचार की तरह सीखा हुआ होता है।

स्रोत:

26. निम्नलिखित कथनों पर विचार कीजिये:

  1. कुछ छत्रको (मशरूमों) में औषधीय है।
  2. कुछ छत्रकों में मन:प्रभावी (साइकोऐक्टिव) गुण होते हैं।
  3. कुछ छत्रकों में कीटनाशी गुण होते हैं।
  4. कुछ छत्रको मैं जैव संदीप्तिशील (बायोल्युमिनिसेंट) गुण होते हैं।

उपर्युक्त में से कितने कथन सही है?

(a) केवल एक                       
(b) केवल दो
(c) केवल तीन                       
(d) सभी चार

उत्तर: (d)

व्याख्या:

  • मशरूम जीवाणुरोधी, प्रतिरक्षा प्रणाली में वृद्धि करने और कोलेस्ट्रॉल को कम करने का कार्य करता है; ये जैव सक्रिय यौगिकों के महत्त्वपूर्ण स्रोत हैं। इन गुणों के परिणामस्वरूप मशरूम के अर्क का उपयोग मानव स्वास्थ्य में व्रद्धि करने और आहार पूरक के रूप में किया जाता है। अतः कथन 1 सही है।
  • मैजिक मशरूम (साइकोएक्टिव कवक) जो संयुक्त राज्य अमेरिका, मैक्सिको, दक्षिण अमेरिका एवं विश्व के कई अन्य हिस्सों में उगते हैं, इनमें साइलोसाइबिन और साइलोसिन पाए जाते हैं, जो मतिभ्रम नियंत्रित पदार्थ होते हैं। अतः कथन 2 सही है।
  • कवक कीट की तरह विकसित होने और अनुकूलन करने में सक्षम जीव होते हैं। विगत कुछ वर्षों में कई सिंथेटिक कीटनाशकों के प्रति कीटों ने प्रतिरोधकता विकसित की है। परजीवी और मेजबान, परभक्षी के बीच किसी भी अन्य संबंध की तरह कवक-आधारित बायोपेस्टीसाइड्स/जैव कीटनाशक में किसी भी अनुकूलन के साथ विकसित होने की क्षमता होती है। ये मनुष्यों और अन्य वन्यजीवों के लिये भी गैर विषैले होते हैं। अतः कथन 3 सही है।
  • पूर्वोत्तर भारतीय वनों में एक मशरूम प्रलेखन परियोजना ने न केवल कवक की 600 किस्मों के बारे में सूचित किया है, बल्कि एक नई एक बायोल्यूमिनिसेंट - या प्रकाश उत्सर्जक - मशरूम की किस्म की खोज भी की है। अतः कथन 4 सही है।

स्रोत:

27. भारतीय गिलहरियों के बारे में, निम्नलिखित कथनों पर विचार कीजिये:

  1. वे भूमि में बिल बनाकर अपने नीड़ का निर्माण करती हैं।
  2. वे अपने भोज्य पदार्थों, जैसे कि दृढ़फलों (नट) और बीजों को भूमि के अंदर जमा करती हैं।
  3. वे सर्वभक्षी होती हैं।

उपर्युक्त में से कितने कथन सही हैं ?

(a)  केवल एक                        
(b)   
केवल दो
(c)  सभी तीन                         
(d)   
कोई भी नहीं

उत्तर:(a)

व्याख्या:

  • भारतीय गिलहरियाँ पेड़ों के शीर्ष पर घोंसला बनाती है। अतः कथन 1 सही नहीं है।
  • भारतीय गिलहरियाँ अपने घोंसले में भोजन संग्रहण करती हैं (जमीन पर रहने वाली गिलहरियाँ अपने भोजन को संग्रहण करने के लिए अपने भूमिगत बिलों के अंदर अलग-अलग जगह बनाती हैं।) अतः कथन 2 सही नहीं है।
  • भारतीय गिलहरियाँ सर्वाहारी होती हैं। ये मुख्य रूप से मेवे और फल खाती हैं लेकिन यह कभी-कभी बीज, कीट, पक्षियों के अंडे भी खाती हैं। अतः कथन 3 सही है।

स्रोत:

28. निम्नलिखित कथनों पर विचार कीजिये:

  1. कुछ सूक्ष्मजीव जल के क्वथनांक से ऊपर के तापमान वाले वातावरण में पनप सकते हैं।
  2. कुछ सूक्ष्मजीव जल के हिमांक से नीचे के तापमान वाले वातावरण में पनप सकते हैं।
  3. कुछ सूक्ष्मजीव 3 से कम pH वाले अत्यंत अम्लीय वातावरण में पनप सकते हैं।

उपर्युक्त में से कितने कथन सही हैं?

(a) केवल एक                        
(b) केवल दो
(c) सभी तीन                         
(d) 
कोई भी नहीं

उत्तर:(c)

व्याख्या:

  • हाइपरथर्मोफिलिक ('सुपरहीट-लविंग') बैक्टीरिया और आर्चिया उच्च तापमान वाले वातावरण में पाए जाते हैं। अतः कथन 1 सही है।
  • सूक्ष्मजीव जीवमंडल के हर भाग में रहते हैं और उनमें से कुछ कम तापमान (हिमांक बिंदु से नीचे भी) पर भी विकास करने में सक्षम होते हैं। ये सूक्ष्मजीव समुद्र या ऊँचे पहाड़ों में रहते हैं। अतः कथन 2 सही है।
  • एसिडोफिल्स ऐसे सूक्ष्मजीव हैं जो अत्यधिक अम्लीय वातावरण में इष्टतम वृद्धि दिखाते हैं। ये दो प्रकार के होते हैं। अत्यधिक एसिडोफिल्स जीव पीएच मान <3 वाले वातावरण में रहते हैं और मध्यम एसिडोफिल्स वाले जीव 3 से 5 के बीच पीएच मान वाली स्थितियों में इष्टतम रूप से बढ़ते हैं। अतः कथन 3 सही है।

29. निम्नलिखित में से कौन, किसी वृक्ष या लकड़ी के लठ्ठे में बने छिद्र में से कीटों को खुरचने के लिये लकड़ी का औज़ार बनाता है?

(a) मत्स्यन मार्जार (फिशिंग कैट)
(b) औरंगउटैन
(c) ऊदबिलाव
(d) स्लॉथ बियर

उत्तर:(b)

व्याख्या:

  • शोधकर्त्ताओं ने ओरिंगउटैन को वृक्ष या लकड़ी के लठ्ठे में बने छिद्र में से कीटों को खुरचने के लिये लकड़ी को औजार के रूप में उपयोग करते हुए देखा है। अत: विकल्प (B) सही है।

स्रोत:

30. निम्नलिखित पर विचार कीजिये:

  1. ऐरोसॉल  
  2. फोम एजेंट
  3. अग्निमंदक
  4. स्नेहक (लूब्रिकेंट)

उपर्युक्त में से कितनों को बनाने में हाइड्रोफ्लोरोकार्बन प्रयुक्त होते हैं?

(a) केवल एक                        
(b) केवल दो
(c) केवल तीन                       
(d) सभी चार

उत्तर:(c)

व्याख्या:

HFC पूर्ण रूप से मानव निर्मित है। ये मुख्य रूप से रेफ्रिजरेशन, एयर-कंडीशनिंग, इंसुलेटिंग फोम और एयरोसोल प्रोपेलेंट के उपयोग से उत्पादित होते हैं, इनका उत्पादन कुछ हद तक अग्नि सुरक्षा के रूप में सॉल्वैंट्स के उपयोग से होता है। अतः विकल्प (c) सही है।

स्रोत:

31. निम्नलिखित कथनों पर विचार कीजिये:

  1. झेलम नदी, वुलर झील से होकर जाती है।
  2. कृष्णा नदी सीधे कोल्लेरू झील का भरण करती है।
  3. गंडक नदी के विसर्पण से काँवर झील निर्मित हुई है।

उपर्युक्त में से कितने कथन सही है?

(a) केवल एक                        
(b) केवल दो
(c) सभी तीन                         
(d) 
कोई भी नहीं

उत्तर:(a)

व्याख्या:

  • वुलर झील एशिया की दूसरी सबसे बड़ी स्वच्छ जल की झील है। यह भारत के जम्मू और कश्मीर में बांदीपोरा ज़िले में अवस्थित है। वुलर झील के जल का मुख्य स्रोत झेलम नदी है। इस झील के केंद्र में एक छोटा सा द्वीप भी है जिसे 'ज़ाइना लंक' कहा जाता है। इस द्वीप का निर्माण जैनुल-अबी-दीन द्वारा करवाया था। अतः कथन 1 सही है।
    • वुलर झील को प्राचीन काल में मौजूद सतीसर झील का अवशेष भी कहा जा ताहै। इस झील का परिसर एक लोकप्रिय सूर्यास्त बिंदु भी है।
  • कोलेरू, भारत की सबसे बड़ी ताजे जल की झीलों में से एक है, (इसे अक्तूबर 1999 में एक अभयारण्य नामित किया गया था) राज्य के कृष्णा और पश्चिम गोदावरी ज़िलों के मध्य स्थित है।
    • कृष्णा नदी प्रत्यक्ष रूप से कोलेरू झील को भरण नहीं करती है। कोलेरू झील को दो मौसमी नदियों, बुडमेरू और तमिलेरू से जल प्राप्त होता है, जो कृष्णा नदी की सहायक नदियाँ हैं। इसलिये, कृष्णा नदी अप्रत्यक्ष रूप से अपनी सहायक नदियों के माध्यम से कोलेरू झील को भरण करती हैअतः कथन 2 सही नहीं है।
  • गंडक नदी के विसर्पण से काँवर झील निर्मित नहीं हुई है, जो भारत में बिहार के बेगूसराय ज़िले में स्थित ताज़े जल की गोखुर झील है
    • बूढ़ी गंडक नदी के एक पुराने वितिरिका के विसर्प द्वारा काँवर झील का निर्माण किया गया था। बूढ़ी गंडक नदी एक पुराने वितिरिका के माध्यम से गंडक नदी के पूर्वी हिस्से के समानांतर बहती है। अतः कथन 3 सही नहीं है।

स्रोत:

32. निम्नलिखित युग्मों पर विचार कीजिये:

पत्तन (पोर्ट)       जिस रूप में सुविख्यात है

  1. कामराजर पोर्ट : भारत में एक कम्पनी के रूप में पंजीकृत सबसे पहला प्रमुख पत्तन
  2. मुंद्रा पोर्ट      : भारत में निजी स्वामित्व वाला सबसे बड़ा पत्तन
  3. विशाखापत्तनम पोर्ट : भारत में सबसे बड़ा आधान पत्तन (कंटेनर पोर्ट)

उपर्युक्त में से कितने युग्म सही सुमेलित हैं?

(a) केवल एक युग्म             
(b) केवल दो युग्म
(c) सभी तीन युग्म              
(d) 
कोई भी युग्म नहीं

उत्तर: (b)

व्याख्या:

  • कामराजर पत्तन (जिसे पहले एन्नोर पोर्ट के नाम से जाना जाता था) भारत में एक कंपनी के रूप में पंजीकृत पहला प्रमुख पत्तन है तथा यह भारत का एकमात्र प्रमुख कॉर्पोरेट/निगम पत्तन है। अतः युग्म 1 सही सुमेलित है।
    • इसे मार्च 1999 में भारत के 12वें प्रमुख पत्तन के रूप में घोषित किया गया था और अक्टूबर 1999 में कंपनी अधिनियम, 1956 के तहत एन्नोर पोर्ट लिमिटेड के रूप में शामिल किया गया था।
    • यह तमिलनाडु के चेन्नई बंदरगाह से लगभग 24 किमी. उत्तर में कोरोमंडल तट पर स्थित है।
  • मुंद्रा पत्तन, गुजरात के कच्छ ज़िले के मुंद्रा के पास कच्छ की खाड़ी के उत्तरी किनारे पर स्थित भारत का सबसे बड़ा वाणिज्यिक पत्तन है। यह अदानी पोर्ट्स एंड स्पेशल इकोनॉमिक ज़ोन लिमिटेड (APSEZ) के स्वामित्व और संचालन में है, जो अडानी समूह का एक हिस्सा है।
    • यह वर्ष 1998 में एक निजी क्षेत्र के बंदरगाह के रूप में स्थापित किया गया था, जो अक्टूबर 2001 में शुरू हुआ। यहाँ से विभिन्न प्रकार के कार्गो जैसे कंटेनर, बल्क (ढेर), ब्रेक-बल्क, तरल, रसायन, ऑटोमोबाइल आदि का आवागमन होता है। अतः युग्म 2 सही सुमेलित है।
  • आंध्र प्रदेश में भारत के पूर्वी तट पर स्थित विशाखापत्तनम बंदरगाह भारत का सबसे बड़ा कंटेनर पोर्ट नहीं है। यह भारत के सबसे पुराने और सबसे बड़े बंदरगाहों में से एक है, जहाँ लौह अयस्क, कोयला, पेट्रोलियम उत्पाद, उर्वरक, कंटेनर आदि जैसे विभिन्न प्रकार के कार्गो का संचालन होता है।
    • भारत में सबसे बड़ा कंटेनर पोर्ट जवाहरलाल नेहरू पोर्ट ट्रस्ट (JNPT) है, जो महाराष्ट्र में मुंबई के पास स्थित है। अतः युग्म 3 सही सुमेलित नहीं है।

स्रोत:

33. निम्नलिखित वृक्षों पर विचार कीजिये:

  1. कटहल (आर्टोकार्पस हेटेरोफाइलस)
  2. महुआ (मधुका इंडिका)
  3. सागौन (टेक्टोना ग्रैन्डिस)

उपर्युक्त में से कितने पर्णपाती वृक्ष हैं?

(a) केवल एक                        
(b) 
केवल दो
(c) सभी तीन                         
(d) 
कोई भी नहीं

उत्तर: (b)

व्याख्या:

  • कटहल सदाबहार, लेटेक्स-उत्पादक 25 मीटर ऊँचाई वाले वृक्ष हैं जो भारत और मलेशिया के स्थानिक वृक्ष हैं, जो श्रीलंका, चीन, दक्षिण-पूर्व एशिया एवं उष्णकटिबंधीय अफ्रीका में पाए जाते हैं। उनकी खेती बड़े फलों व लकड़ी हेतु की जाती है जो आकार एवं स्वरुप में भिन्न हो सकते हैं।
  • महुआ एक उष्णकटिबंधीय पर्णपाती तेज़ी से बढ़ने वाला वृक्ष है और मध्य प्रदेश, झारखंड, छत्तीसगढ़, ओडिशा, महाराष्ट्र एवं बिहार में पाया जाता है।
  • सागौन एक उष्णकटिबंधीय दृढ़ लकड़ी वाला पर्णपाती वृक्ष है। यह दक्षिण एवं दक्षिण पूर्व एशिया का स्थानिक पेड़ है, लेकिन इसकी खेती कई अन्य क्षेत्रों में भी की जाती है।
    • सागौन की लकड़ी को इसके स्थायित्त्व और जल प्रतिरोध हेतु महत्त्वपूर्ण माना जाता है और इसका उपयोग नाव निर्माण, फर्नीचर, नक्काशी एवं लिबास जैसे विभिन्न उद्देश्यों हेतु किया जाता है।

अतः विकल्प (b) सही है।

स्रोत…

34. निम्नलिखित कथनों पर विचार कीजिये:

  1. चीन की तुलना में भारत के पास अधिक कृषियोग्य क्षेत्र है।
  2. चीन की तुलना में भारत में सिंचित क्षेत्र का अनुपात अधिक है।
  3. चीन की तुलना में भारत की कृषि में प्रति हेक्टेयर औसत उत्पादकता अधिक है।

उपर्युक्त में से कितने कथन सही है?

(a) केवल एक                        
(b) 
केवल दो
(c) सभी तीन                         
(d) 
कोई भी नहीं

उत्तर:(b)

व्याख्या:

  • कृषि योग्य भूमि वह भूमि है जिसका उपयोग फसलों को उगाने हेतु किया जा सकता है। कृषि योग्य भूमि देश की कृषि क्षमता एवं खाद्य सुरक्षा का महत्त्वपूर्ण संकेतक है।
    • विश्व जनसंख्या समीक्षा के अनुसार, भारत के पास 156.1 मिलियन हेक्टेयर कृषि योग्य भूमि है, जो इसके कुल भूमि क्षेत्र का लगभग 47% है।
    • चीन के पास 119.5 मिलियन हेक्टेयर कृषि योग्य भूमि है, जो उसके कुल भूमि क्षेत्र का लगभग 12% है। भारत के पास चीन की तुलना में अधिक कृषि योग्य भूमि है क्योंकि इसमें मैदानी एवं नदी घाटियों का एक बड़ा हिस्सा शामिल है, जो खेती हेतु उपयुक्त हैं। चीन में अधिक पहाड़ी तथा रेगिस्तानी क्षेत्र हैं, जो खेती के लिये उपयुक्त नहीं हैं। अतः कथन 1 सही है।
  • सिंचित क्षेत्र का अनुपात भारत में लगभग 48% और चीन में 40-41% है। इसका मतलब यह है कि चीन की तुलना में भारत में सिंचाई के तहत कृषि योग्य भूमि का प्रतिशत अधिक है।
    • हालाँकि चीन ने जल की कमी से निपटने और कृषि उत्पादकता बढ़ाने हेतु सिंचाई के बुनियादी ढाँचे एवं प्रौद्योगिकी जैसे बाँध, नहर, पंप, स्प्रिंकलर तथा ड्रिप सिस्टम में अधिक निवेश किया है। भारत की सिंचाई के लिये वर्षा व भूजल पर बड़ी निर्भरता है, जो परिवर्तनशीलता एवं कमी से प्रभावित हैं। अतः कथन 2 सही है।
  • कृषि में उत्पादकता को भूमि के प्रति इकाई क्षेत्र की उपज से मापा जा सकता है, जो कि प्रति हेक्टेयर या एकड़ में उत्पादित फसल की मात्रा है।
    • भारतीय कृषि में प्रति हेक्टेयर औसत उत्पादकता चावल हेतु 2.4 टन और गेहूँ के लिये 3 टन है, जबकि चीन में यह चावल हेतु 6.7 टन और गेहूँ के लिये 5 टन है। अतः कथन 3 सही नहीं है।

स्रोत:

35. निम्नलिखित में से कौन-सा एक, समुद्र स्तर में पुनरावर्ती गिरावट का सर्वोत्तम उदाहरण है, जिससे वर्तमान समय की सुविस्तृत कच्छभूमि उत्पन्न हुई है?

(a)  भीतरकनिका गरान (मैन्ग्रोव)
(b)  मरक्कनम लवण बेसिन
(c)  नौपाड़ा अनूप                       
(d)  कच्छ का रण

उत्तर:(d)

व्याख्या:

  • कच्छ का रण गुजरात, भारत में थार रेगिस्तान में स्थित वृहद् लवणीय दलदल है।
  • कच्छ में वर्तमान समुद्र तट और बालू के टीलों का निर्माण लगभग 5,000 वर्ष पहले शुरू हुआ, जो वर्षारहित सैकड़ों वर्षों का परिणाम है।
    • इन गंभीर जलवायु परिस्थितियों के कारण समुद्र का स्तर कम हो रहा है परिणामस्वरूप समुद्र तट के किनारे टीलों का निर्माण हुआ।

स्रोत…

36. भारत के कतिपय तटीय क्षेत्रों में, प्रचुर मात्रा में उपलब्ध इल्मेनाइट और रूटाइल निम्नलिखित में से किसके समृद्ध स्रोत हैं?

(a) ऐलुमिनियम                     
(b) 
ताम्र
(c) लौह                                    
(d) 
टाइटेनियम

उत्तर:(d)

व्याख्या:

  • भारत भारी खनिज संसाधनों से संपन्न है जो मुख्य रूप से देश के तटीय हिस्सों में पाए जाते हैं। भारी खनिज रेत में सात खनिजों जैसे, इल्मेनाइट, ल्यूकॉक्सीन (ब्राउन इल्मेनाइट), रूटाइल, जिरकोन, सिलिमेनाइट, गार्नेट और मोनाज़ाइट का एक समूह है। इल्मेनाइट (FeO.TiO2) एवं रूटाइल (TiO2) टाइटेनियम के दो प्रमुख खनिज स्रोत हैं।

अतः विकल्प (d) सही है।

37. संसार का लगभग तीन-चौथाई कोबाल्ट, जो विद्युत् मोटर वाहनों की बैटरी के निर्माण के लिये आवश्यक धातु है, किस देश द्वारा उत्पादित किया जाता है?

(a)  अर्जेन्टीना
(b)  बोत्सवाना
(c)  कांगो लोकतांत्रिक गणराज्य (डेमोक्रेटिक रिपब्लिक ऑफ द कांगो)
(d)  कज़ाख़स्तान

उत्तर: (c)

व्याख्या:

  • लोकतांत्रिक गणराज्य कांगो (DRC) वर्तमान में कोबाल्ट का विश्व में सबसे बड़ा उत्पादक है, जो वैश्विक उत्पादन का लगभग 70 प्रतिशत है। अतः विकल्प (c) सही है।

38. निम्नलिखित में से कौन-सा एक, कांगो बेसिन का भाग है?

(a) कैमरून
(b) नाइज़ीरिया
(c) दक्षिण सूडान
(d) युगांडा

उत्तर: (a)

व्याख्या:

  • कांगो बेसिन छह देशों में विस्तारित है- कैमरून, मध्य अफ्रीकी गणराज्य, लोकतांत्रिक गणराज्य कांगो, कांगो गणराज्य, भूमध्यरेखीय गिनी और गैबॉन। अतः विकल्प (a) सही है।

39. निम्नलिखित कथनों पर विचार कीजिये:

  1. अमरकंटक पहाड़ियाँ विन्ध्य और सह्याद्रि श्रेणियों के संगम पर हैं।
  2. बिलीगिरिरंगन पहाड़ियाँ सतपुड़ा श्रेणी के सबसे पूर्वी भाग हैं।
  3. शेषाचलम पहाड़ियाँ पश्चिमी घाट के सबसे दक्षिणी भाग हैं।

उपर्युक्त में से कितने कथन सही है?

(a) केवल एक
(b) केवल दो
(c) सभी तीन
(d) कोई भी नहीं

उत्तर: (d)

व्याख्या:

  • अमरकंटक पहाड़ी अद्वितीय प्राकृतिक विरासत क्षेत्र है और विंध्य एवं सतपुड़ा पर्वत शृंखला का मिलन बिंदु है। अतः कथन 1 सही नहीं है।
  • बिलीगिरिरंगन पहाड़ी दक्षिण-पूर्वी कर्नाटक में स्थित है, जबकि सतपुड़ा पर्वत शृंखला पूर्वी गुजरात में महाराष्ट्र एवं पूर्वी मध्य प्रदेश से होते हुए छत्तीसगढ़ में समाप्त होती है। अतः कथन 2 सही नहीं है।
  • शेषचलम पहाड़ियाँ दक्षिणी आंध्र प्रदेश में पूर्वी घाट का हिस्सा हैं, जबकि पश्चिमी घाट केरल, तमिलनाडु, कर्नाटक, गोवा, महाराष्ट्र एवं गुजरात राज्यों तक विस्तृत हैं। अतः कथन 3 सही नहीं है।

अतः विकल्प (d) सही है।

40. भारत की कनेक्टिविटी परियोजनाओं के संदर्भ निम्नलिखित कथनों पर विचार कीजिये:

  1. स्वर्णिम चतुर्भुज परियोजना के अधीन पूर्व-पश्चिम गलियारा (कॉरिडोर), डिब्रूगढ़ और सूरत को जोड़ता है।
  2. त्रिपक्षीय राजमार्ग मणिपुर में मोरेह को, म्यांमार से होते हुए, थाईलैंड में चियांग माई से जोड़ता है।
  3. बांग्लादेश चीन-भारत-म्यांमार आर्थिक गलियारा (कॉरिडोर) उत्तर प्रदेश में वाराणसी को चीन में कुनमिंग से जोड़ता है।

उपर्युक्त में से कितने कथन सही हैं?

(a) केवल एक                        
(b) 
केवल दो
(c) सभी तीन                         
(d) 
कोई भी नहीं

उत्तर:(d)

व्याख्या:

  • पूर्व-पश्चिम गलियारा सिलचर को पोरबंदर से जोड़ता है। अतः कथन 1 सही नहीं है।
  • भारत-म्याँमार-थाईलैंड त्रिपक्षीय राजमार्ग तीन देशों को भारत में मोरेह से म्याँमार में बागान के माध्यम से थाईलैंड में माई सॉत को जोड़ता है। अतः कथन 2 सही नहीं है।
  • BCIM कॉरिडोर (2800 किलोमीटर लंबा) कोलकाता से पहले म्याँमारमें मांडले और बांग्लादेश में ढाका जैसे प्रमुख शहरों से गुज़रते हुए चीन के युन्नान प्रांत में कुनमिंग को कोलकाता से जोड़ने का प्रस्ताव करता है। अतः कथन 3 सही नहीं है।

अतः विकल्प (d) सही है।

स्रोत…

41. निम्नलिखित कथनों पर विचार कीजिये :

    कथन-I: आधारिक संरचना निवेश न्यासों (InvITs) में जमा (डिपॉजिट) से हुई ब्याज की आय, जो उनके निवेशकों में वितरित की जाती है, कर से छूट प्राप्त है, किन्तु लाभांश करयोग्य है।

    कथन-II: ‘वित्तीय परिसंपत्तियों का प्रतिभूतिकरण और पुनर्गठन तथा प्रतिभूति हित का प्रवर्तन अधिनियम, 2002’ के अधीन InvITs को ऋणी के रूप में मान्यता प्राप्त है।

उपर्युक्त कथनों के बारे में, निम्नलिखित में से कौन-सा एक सही है?

(a)  कथन-I और कथन-II दोनों सही हैं तथा कथन-II, कथन-I की सही व्याख्या है।
(b)  कथन- I और कथन-II दोनों सही हैं तथा कथन-II, कथन-I की सही व्याख्या नहीं है
(c)  कथन- I सही है किन्तु कथन-II गलत है
(d)  कथन-I गलत है किन्तु कथन-II सही है

उत्तर:(d)

व्याख्या:

  • ब्याज आय जो InvIT अपने अंतर्निहित SPVs से प्राप्त करती है तथा यूनिटधारकों को हस्तांतरित करती है, उस पर कर लगाया जाता है। InvIT जो लाभांश प्रदान करता है उस पर भी टैक्स लगता है। ब्याज और लाभांश दोनों पर आयकर स्लैब के अनुसार कर लगाया जाता है। यह वहाँ लागू होता है जहाँ InvIT ने अधिनियम की धारा 115BAA के तहत कराधान का विकल्प चुना है। अतः कथन 1 सही नहीं है।
  • InvITs को सरफेसी (SARFAESI) अधिनियम 2002 के तहत उधारकर्त्ताओं के रूप में मान्यता प्राप्त है। सरफेसी अधिनियम और ऋण वसूली अधिनियम में संशोधन किया गया है। अब, इन कानूनों के तहत एक पूल निवेश वाहन को उधारकर्त्ता माना जा सकता है। इसका मतलब है कि InvIT या REIT द्वारा जारी सूचीबद्ध सुरक्षित ऋण प्रतिभूतियों हेतु एक डिबेंचर ट्रस्टी SARFAESI अधिनियम के तहत सुरक्षा और प्रवर्तन तंत्र का उपयोग कर सकता है। अतः कथन 2 सही है।

स्रोत…

42. निम्नलिखित कथनों पर विचार कीजिये:

    कथन-I: कोविड विश्वमारी के बाद के हाल के विगत काल में, पूरे विश्व में अनेक केंद्रीय बैंकों ने ब्याज दरें बढ़ा दी थीं।

    कथन-II : आमतौर पर केंद्रीय बैंक मानते हैं कि उनके पास बढ़ती हुई उपभोक्ता कीमतों का, मौद्रिक नीति के उपायों से, प्रतिकार करने की सामर्थ्य है।

उपर्युक्त कथनों के बारे में, निम्नलिखित में से कौन-सा एक सही है?

(a)  कथन-I और कथन-II दोनों सही हैं तथा कथन-II, कथन-I की सही व्याख्या है।
(b)  कथन-I और कथन-II दोनों सही हैं तथा कथन-II, कथन-I की सही व्याख्या नहीं है
(c)  कथन-I सही है किन्तु कथन-II गलत है
(d)  कथन-I गलत है किन्तु कथन-II सही है

उत्तर: (a)

  • कोविड विश्वमारी के बाद के हाल के विगत काल में, विश्व भर में अनेकों केंद्रीय बैंकों ने महामारी के उपरांत की मुद्रास्फीति को रोकने के लिये ब्याज दरों में बढ़ोतरी की थी। उदाहरण के लिये, RBI की मौद्रिक नीति समिति ने मई 2022 से कई बार दरों में बढ़ोतरी कर चुकी है। अतः कथन 1 सही है।
  • केंद्रीय बैंकों को आमतौर पर वस्तुओं की बढ़ती कीमतों को नियंत्रित करने का कार्य सौंपा जाता है। केंद्रीय बैंक द्वारा आर्थिक उतार-चढ़ाव का प्रबंधन करने तथा मूल्य स्थिरता प्राप्त करने के लिये मौद्रिक नीति का उपयोग किया जाता है। अतः कथन 2 सही है।

स्रोत:

43. निम्नलिखित कथनों पर विचार कीजिये:

कथन-I: ऐसी संभावना है कि कार्बन बाज़ार, जलवायु परिवर्तन से निपटने का एक सबसे व्यापक साधन हो जाए।

कथन-II : कार्बन बाज़ार संसाधनों को प्राइवेट सेक्टर से राज्य को हस्तांतरित कर देते हैं।

उपर्युक्त कथनों के बारे में, निम्नलिखित में से कौन-सा एक सही है?

(a)  कथन-I और कथन-II दोनों सही हैं तथा कथन-II, कथन-I की सही व्याख्या है।
(b)  कथन-I और कथन-II दोनों सही हैं तथा कथन-II, कथन-I की सही व्याख्या नहीं है।
(c)  कथन-I सही है किन्तु कथन-II गलत है।
(d)  कथन-I गलत है किन्तु कथन-II सही है।

उत्तर:(c)

  • कार्बन बाज़ार, जलवायु परिवर्तन के विरुद्ध वर्षों से चल रहे प्रयासों में सबसे व्यापक उपकरणों में से एक बन चुका है। वर्ष 2021 के अंत तक इसे विश्व के 21% से अधिक उत्सर्जन हेतु कार्बन मूल्य के निर्धारण में किसी न किसी रूप में आवृत किया गया था, जो वर्ष 2020 में 15% था। अतः कथन 1 सही है। 
  • कार्बन बाज़ारों को एक ऐसे वित्तीय तंत्र के सृजन हेतु डिज़ाइन किया गया है जिससे ग्रीनहाउस गैस उत्सर्जन में कमी को प्रोत्साहित किया जा सके। कार्बन बाज़ारों में विभिन्न कंपनियाँ एवं संगठन कार्बन क्रेडिट को खरीद या बेच सकते हैं। हालाँकि कार्बन बाज़ार से कार्बन क्रेडिट की बिक्री के माध्यम से सरकार के लिये राजस्व उत्पन्न हो सकता है लेकिन यह पूरी तरह से कहना सही नहीं होगा कि इसमें निजी क्षेत्र से सरकार को संसाधनों का हस्तांतरण होता है। अतः कथन 2 सही नहीं है।

स्रोत:

44. भारतीय रिज़र्व बैंक की निम्नलिखित में से किस एक गतिविधि को ‘बंध्यकरण (स्टेरिलाइजेशन)’ के एक के रूप में माना जाता है?

(a)  ‘खुला बाज़ार कार्रवाईका संचालन
(b)  निपटारा और भुगतान प्रणालियों की निगरानी
(c)  केंद्र सरकार और राज्य सरकारों के लिये ऋण एवं रोकड़ प्रबंधन
(d)  गैर-बैंक वित्तीय संस्थानों के कार्यों का विनियमन

उत्तर:(a)

व्याख्या:

  • बंध्यकरण (स्टेरिलाइजेशन) उस प्रक्रिया को संदर्भित करता है जिसके द्वारा भारतीय रिज़र्व बैंक (Reserve Bank of India- RBI) अर्थव्यवस्था में प्रवेशित नवीन मुद्रा को प्रभावहीन करने हेतु बैंकिंग प्रणाली से मुद्रा का निर्गमन करता है। क्लासिकी बंध्यकरण में केंद्रीय बैंक खुला बाज़ार कार्रवाई (खुला बाज़ार परिचालन) जैसे मौद्रिक उपकरणों का प्रयोग करता है। अतः विकल्प (a) सही है।
  • सामान्यतः RBI बंध्यकरण के लिये बाज़ार स्थिरीकरण योजना (Market Stabilisation Scheme- MSS) को अपनाता है।
    • बाज़ार स्थिरीकरण योजना (MSS): इसमें बड़ी पूंजी (लार्ज कैपिटल) के प्रवाह से उत्पन्न अधिशेष तरलता को अल्प-दिनांकित सरकारी प्रतिभूतियों और ट्रेज़री बिलों की बिक्री के माध्यम से कम किया जाता है।

स्रोत:

45. निम्नलिखित बाज़ारों पर विचार कीजिये:

  1. सरकारी बॉन्ड बाज़ार
  2. शीघ्रावधि द्रव्य बाज़ार (कॉल मनी मार्केट)
  3. कोषपत्र बाज़ार (ट्रेजरी बिल मार्केट)
  4. स्टॉक बाज़ार

पूँजी बाज़ार में, उपर्युक्त में से कितने शामिल हैं?

(a) केवल एक                        
(b) 
केवल दो
(c) केवल तीन                       
(d) 
सभी चार

उत्तर:(b)

व्याख्या:

  • सरकारी बॉन्ड बाज़ार, पूंजी बाज़ार का एक हिस्सा है। अतः विकल्प 1 सही है।
  • शीघ्रावधि दृव्य (कॉल मनी) दर, वह दर है जिस पर धन को अल्पकालिक ऋण के रूप में लेकर बाज़ार में दिया जाता है। अतः विकल्प 2 सही नहीं है।

स्रोत:

  • कोषपत्र (ट्रेज़री बिल) सरकार द्वारा जारी अल्पकालिक ऋण प्रतिभूतियाँ हैं, जिनकी परिपक्वता अवधि एक वर्ष की होती है। इन्हें सबसे सुरक्षित प्रकार का दृव्य बाज़ार साधन माना जाता है। अतः विकल्प 3 सही नहीं है।

स्रोत:.

  • स्टॉक मार्केट, पूंजी बाज़ार का हिस्सा है। अतः विकल्प 4 सही है।

46. निम्नलिखित में से कौन-सा एक, ‘लघु कृषक बड़े खेत’ की संकल्पना का सर्वोत्तम वर्णन है?

(a)  युद्ध के कारण अपने देशों से बड़ी संख्या में विस्थापित लोगों का, एक बड़ी खेतीयोग्य ज़मीन देकर, जिसमें वे सामूहिक खेती कर उपज को आपस में बाँटते हैं, पुन:स्थापन करना
(b)  किसी क्षेत्र के अनेक सीमांत कृषक अपने आपको समूहों में व्यवस्थित कर चुनिन्दा कृषि संक्रियाओं में समकालन और संगति लाते हैं
(c)  किसी क्षेत्र के अनेक सीमांत कृषक मिलकर किसी निगमित निकाय के साथ संविदा कर अपनी ज़मीन उस निगमित निकाय को किसी नियत अवधि के लिये दे देते हैं, जिसके लिये वह निगमित निकाय कृषकों को एक सहमत राशि का भुगतान करता है
(d)  कोई कम्पनी किसी क्षेत्र के कुछ संख्या में लघु कृषकों को ऋण, तकनीकी जानकारियाँ और सामग्री की निविष्टियाँ प्रदान करती है, ताकि वे कंपनी की विनिर्माण प्रक्रिया और वाणिज्यिक उत्पादन के लिये उसकी ज़रूरत के कृषि-पण्य का उत्पादन करें

उत्तर:(b)

व्याख्या:

  • "लघु कृषक बड़े खेत (SFLF)" एक ऐसा कृषि मॉडल है जिसे लाखों लघु एवं सीमांत किसानों को आपूर्ति शृंखला में होने वाली विसंगतियों एवं सौदेबाजी की शक्ति की कमी के कारण होने वाली क्षति को दूर करने के लिये लाया गया है। यह मॉडल सहभागी और लचीला है जिसके तहत किसी क्षेत्र के अनेक सीमांत कृषक स्वयं को समूहों में व्यवस्थित कर चुनिंदा कृषि कार्यों में समकालन और संगति लाते हैं।

स्रोत:

47. निम्नलिखित कथनों पर विचार कीजिये:

  1. भारत सरकार काले तिल [नाइजर (गुइजोटिया एबिसिनिका)] के बीजों के लिये न्यूनतम समर्थन कीमत उपलब्ध कराती है।
  2. काले तिल की खेती खरीफ की फसल के रूप में की जाती है।
  3. भारत के कुछ जनजातीय लोग काले तिल के बीजों का तेल भोजन पकाने के लिये प्रयोग में लाते हैं।

उपर्युक्त में से कितने कथन सही हैं?

(a) केवल एक                        
(b) 
केवल दो
(c) सभी तीन                         
(d) 
कोई भी नहीं

उत्तर:(c)

व्याख्या:

  • भारत सरकार काले तिल [नाइजर (गुइजोटिया एबिसिनिका)] के बीजों के लिये न्यूनतम समर्थन कीमत प्रदान करती है। अतः कथन 1 सही है।
  • काले तिल की खेती खरीफ की फसल के रूप में की जाती है। अतः कथन 2 सही है।
  • भारत में कुछ जनजातियाँ काले तिल/रामतिल के तेल का उपयोग खाना पकाने के लिये करती हैं। तेल निकालने के उपरांत इसके खल का उपयोग पशुओं के चारे के लिये करने के साथ ही इन बीजों का उपयोग मसाले के रूप में भी किया जाता है। अतः कथन 3 सही है।

स्रोत:  

48. निम्नलिखित परिसंपत्तियों में निवेशों पर विचार कीजिये :

  1. ब्रांड पहचान
  2. माल-सूची
  3. बौद्धिक संपदा
  4. ग्राहकों की डाक सूची

उपर्युक्त में से कितने अमूर्त निवेश माने जाते हैं?

(a) केवल एक                       
(b) 
केवल दो
(c) केवल तीन                       
(d) सभी चार

उत्तर:(c)

व्याख्या:

  • जिन संपत्तियों को भौतिक रूप से स्पर्श नहीं किया जा सकता उन्हें अमूर्त संपत्ति के रूप में जाना जाता है। वे प्रकृति में गैर-भौतिक हैं और एक वर्ष या उससे अधिक हेतु उपयोग किये जा सकते हैं, इसमें ब्रांड मूल्य, सद्भावना और बौद्धिक संपदा जैसे ट्रेडमार्क, पेटेंट तथा कॉपीराइट आदि शामिल हैं।
  • मूर्त संपत्ति एक ऐसी संपत्ति है जिसमें भौतिक पदार्थ होता है। इसके उदाहरणों में सूची, भवन, रोलिंग स्टॉक, निर्माण उपकरण या मशीनरी और कार्यालय फर्नीचर शामिल हैं।

अत: विकल्प c सही है।

स्रोत…

49. निम्नलिखित पर विचार कीजिये:

  1. जनांकिकीय निष्पादन
  2. वन और पारिस्थितिकी
  3. शासन सुधार       
  4. स्थिर सरकार
  5. कर एवं राजकोषीय प्रयास

समस्तर कर-अवक्रमण के लिये पंद्रहवें वित्त आयोग ने उपर्युक्त में से कितने को जनसंख्या क्षेत्रफल और आय के अंतर के अलावा निकष के रूप में प्रयुक्त किया?

(a) केवल दो                           
(b) 
केवल तीन
(c) केवल चार                        
(d) 
सभी पाँच

उत्तर:(b)

व्याख्या:

समस्तर कर-अवक्रमण हेतु पंद्रहवें वित्त आयोग ने निकष के रूप में निम्नलिखित का उपयोग किया:

  • जनसंख्या
  • क्षेत्रफल
  • वन और पारिस्थितिकी
  • आय अंतराल
  • कर एवं राजकोषीय प्रयास
  • जनांकिकीय निष्पादन

अतः उत्तर B सही है।

50. निम्नलिखित आधारिक संरचना क्षेत्रकों पर विचार कीजिये:

  1. किफायती आवास
  2. सर्वसाधारण द्रुत परिवहन (मास रैपिड ट्रांसपोर्ट)
  3. स्वास्थ्य देखभाल
  4. पुनर्नवीकरणीय ऊर्जा

उपर्युक्त में से कितनों पर यू.एन.ओ.पी.एस. आधारिक संरचना और नवाचार में धारणीय निवेश पहल [सस्टेनेबल इन्वेस्टमेंट्स इन इन्फ्रास्ट्रक्चर ऐंड इनोवेशन (S3i)] अपने निवेशों के लिये ध्यान केन्द्रित करता है।

(a) केवल एक                        
(b) 
केवल दो
(c) केवल तीन                       
(d) 
सभी चार

उत्तर:(c)

उत्तर:

  • S3i ने राजस्थान में स्थित 250 मेगावाट सौर ऊर्जा संयंत्र में निवेश करने हेतु महत्त्वपूर्ण शेयरधारक समझौते पर हस्ताक्षर किये हैं। S3i का लक्ष्य रणनीतिक और संस्थागत निवेशकों के साथ सहयोग करते हुए विकासशील देशों में बड़े पैमाने पर बुनियादी ढाँचा परियोजनाओं में निवेश करना है, जो किफायती आवास, नवीकरणीय ऊर्जा एवं स्वास्थ्य के अपने अनिवार्य क्षेत्रों पर ध्यान केंद्रित कर रहा है। अतः विकल्प (c) सही है।

स्रोत…

51. होम गार्ड के संदर्भ में, निम्नलिखित कथनों पर विचार कीजिये:

  1. होम गार्ड को होम गार्ड अधिनियम और केंद्र सरकार के नियमों के अधीन समुत्थापित किया जाता है।
  2. होम गार्ड की भूमिका आंतरिक सुरक्षा बनाए रखने में पुलिस को सहायक बल के रूप में सेवा देने की है।
  3. अंतर्राष्ट्रीय सीमा/तटीय क्षेत्रों में घुसपैठ रोकने के लिये कुछ राज्यों में सीमा विंग होम गार्ड बटालियनें समुत्थापित की गई हैं।

उपर्युक्त में से कितने कथन सही हैं?

(a) केवल एक                        
(b) 
केवल दो
(c)  सभी तीन 
(d)  कोई भी नहीं

 उत्तर: (b)

व्याख्या:

  • 'होम गार्ड (गृहरक्षक दल) एक स्वैच्छिक बल है, जिसे भारत में सर्वप्रथम दिसंबर 1946 में नागरिक अशांति और सांप्रदायिक दंगों को नियंत्रित करने में पुलिस की सहायता के लिये स्थापित किया गया था।
  • इसके बाद स्वैच्छिक नागरिक बल की अवधारणा को कई राज्यों द्वारा अपनाया गया था। वर्ष 1962 में चीन के आक्रमण के मद्देनज़र, केंद्र ने राज्यों एवं केंद्रशासित प्रदेशों को सलाह दी कि वे अपने मौजूदा स्वैच्छिक संगठन को एक समान स्वैच्छिक बल में विलय करें जिसे होम गार्ड्स के रूप में जाना जाता है।
  • होम गार्ड्स की स्थापना होम गार्ड अधिनियम तथा राज्यों/केंद्रशासित प्रदेशों के नियमों के तहत की जाती है, न कि केंद्र सरकार के नियमों के तहत। अतः कथन 1 सही नहीं है।
  • होम गार्ड्स की भूमिका आंतरिक सुरक्षा स्थितियों के प्रबंधन में पुलिस के लिये एक सहायक बल के रूप में सेवा करना, किसी भी प्रकार की आपात स्थिति जैसे हवाई हमले, आग लगना, चक्रवात, भूकंप, महामारी आदि में समुदाय की मदद करना, आवश्यक सेवाओं को जारी रखने में मदद करना, सांप्रदायिक सद्भाव को बढ़ावा देना तथा सुभेद्द्य/कमज़ोर वर्गों की रक्षा में प्रशासन की सहायता करना, सामाजिक-आर्थिक एवं कल्याणकारी गतिविधियों में भाग लेना और नागरिक सुरक्षा कर्तव्यों का पालन करना है। अतः कथन 2 सही है।
  • बॉर्डर विंग होम गार्ड्स (BWHG) की पंद्रह बटालियनों को सीमावर्त्ती राज्यों में स्थापित किया गया है। जिनमें से 6 बटालियन पंजाब में, 4 बटालियन राजस्थान में, 2 बटालियन गुजरात में तथा मेघालय, त्रिपुरा एवं पश्चिम बंगाल के लिये एक-एक बटालियन अंतर्राष्ट्रीय सीमा/तटीय क्षेत्रों में होने वाली घुसपैठ को रोकने के लिये सीमा सुरक्षा बल के सहायक के रूप में कार्य करेंगी। ये बाह्य आक्रमण के समय संवेदनशील क्षेत्र में VA/VPs एवं संचार लाइनों को सुरक्षा प्रदान करेंगी। अतः कथन 3 सही है

स्रोत…

52. भारत के संदर्भ में, निम्नलिखित युग्मों पर विचार कीजिये:

कृत्य

यह कृत्य जिस अधिनियम के अधीन आता है

1. अनधिकृत रूप से पुलिस या सेना की वर्दी को पहनना अधिनियम, 1923

शासकीय गुप्त बात

2. किसी पुलिस अधिकारी या सैनिक अधिकारी के ड्यूटी में तैनात होने के दौरान जान-बूझकर उन्हें गुमराह करना या अन्यथा उनके साथ हस्तक्षेप करना

भारतीय साक्ष्य अधिनियम, 1872

3. बंदूक/तोप से अनुष्ठानपरक गोली/गोला दागना जिससे अन्य लोगों की वैयक्तिक सुरक्षा के लिये जोखिम हो सकता हो

आयुध (संशोधन) अधिनियम, 2019

उपर्युक्त में से कितने युग्म सही सुमेलित हैं?

(a)  केवल एक                        
(b) 
केवल दो
(c)  सभी तीन                         
(d)  
कोई भी नहीं

उत्तर:(b)

व्याख्या:

  • आधिकारिक शासकीय गुप्त बात अधिनियम पहली बार वर्ष 1923 में अधिनियमित किया गया जिसे स्वतंत्रता के बाद भी इसे बरकरार रखा गया था। सरकारी कर्मचारियों तथा नागरिकों पर लागू होने वाला यह कानून जासूसी, देशद्रोह और राष्ट्र की अखंडता के लिये अन्य संभावित खतरों से निपटने हेतु रूपरेखा प्रदान करता है। कानून जासूसी, 'गुप्त' जानकारी साझा करने, वर्दी का अनधिकृत उपयोग, (धारा-6 के तहत) सूचना बाधित करने, निषिद्ध या प्रतिबंधित क्षेत्रों में सशस्त्र बलों के साथ हस्तक्षेप करने आदि को दंडनीय अपराध के रूप में वर्णित करता है। इसके अंतर्गत दोषी पाए जाने पर व्यक्ति को 14 वर्ष तक का कारावास, जुर्माना या दोनों हो सकता है। अतः युग्म 1 सही सुमेलित है।
  • आधिकारिक शासकीय गुप्त बात अधिनियम 1923 की धारा 7 के तहत किसी भी निषिद्ध स्थान के आसपास कोई भी व्यक्ति किसी भी पुलिस अधिकारी या केंद्र के सशस्त्र बलों के किसी भी सदस्य जो निषिद्ध स्थान के संबंध में संतरी, गश्ती या अन्य समान कर्तव्य को धारण करता हो, को जानबूझकर गुमराह या अन्यथा हस्तक्षेप या बाधा नहीं डालेगा। अतः युग्म 2 सही सुमेलित नहीं है।
  • शस्त्र (संशोधन) अधिनियम, 2019 के अंतर्गत कोई भी व्यक्ति शस्त्रों का उपयोग लापरवाही पूर्वक या जश्न मनाने के लिये करता है, जिससे मानव जीवन या दूसरों की व्यक्तिगत सुरक्षा प्रभावित होती है इससे उसे अल्पावधि के लिये दंडनीय कारावास दिया जा सकता है जिसे बढ़ाकर दो वर्ष तक किया जा सकता है इसके साथ ही एक लाख रुपए तक जुर्माना लगाया जा सकता है। अतः युग्म 3 सही सुमेलित है।

स्रोत:

53. निम्नलिखित युग्मों पर विचार कीजिये:

प्राय: समाचारों में उल्लिखित होने वाले क्षेत्र समाचारों में होने का कारण
1. उत्तरी किवू और इटुरी आमनिया और अज़रबैजान के बीच युद्ध
2. नागोर्नो कारबाख मोज़ाम्बीक में उपप्ल
3. खेरसॉन और जपोरिज़िया इजराइल और लेबनाम के बीच विवाद

उपर्युक्त में से कितने युग्म सही सुमेलित है?

(a) केवल एक                        
(b)  
केवल दो
(c)  सभी तीन                          
(d)  
कोई भी नहीं

उत्तर:(d)

व्याख्या:

  • किवु और इटुरी कांगो गणराज्य से संबंधित हैं। कांगो गणराज्य और रवांडा के बीच वर्ष 1994 में युद्ध शुरू हुआ, जिसमे 800,000 से अधिक रवांडा के तुत्सी एवं हुतस लोगों का नरसंहार कर दिया गया। अतः युग्म 1 सही सुमेलित नहीं है।
  • नागोर्नो-काराबाख दक्षिण-पश्चिमी अज़रबैजान का एक क्षेत्र है। इसको पूर्व अज़रबैजान सोवियत सोशलिस्ट रिपब्लिक (S.S.R.) के स्वायत्त ओब्लास्ट (प्रांत) एवं स्व-घोषित देश नागोर्नो-काराबाख गणराज्य नाम से जाना जाता है, जिसकी स्वतंत्रता अंतर्राष्ट्रीय स्तर पर मान्यता प्राप्त नहीं है। नागोर्नो-काराबाख के स्व-घोषित गणराज्य के सैनिक वर्तमान में लगभग 2,700 वर्ग मील (7,00 वर्ग किमी) क्षेत्र को नियंत्रित करते हैं, जबकि पूर्व स्वायत्त क्षेत्र में लगभग 1,700 वर्ग मील (4,400 वर्ग किमी) क्षेत्र शामिल था। अतः युग्म 2 सही सुमेलित नहीं है।
  • खेरसॉन और ज़ापोरिज़िया यूक्रेन से संबंधित हैं, जो यूक्रेन एवं रूस के बीच विवादित क्षेत्र है। अतः युग्म 3 सही सुमेलित नहीं है।

स्रोत…

54. निम्नलिखित कथनों पर विचार कीजिये:

कथन-I: इज़राइल ने कुछ अरब राज्यों के साथ राजनयिक संबंध स्थापित किये हैं।

कथन-II: ‘अरब शांति पहलसऊदी अरब की मध्यस्थता से इज़राइल और अरब लीग के बीच हस्ताक्षरित हुआ।

उपर्युक्त कथनों के बारे में, निम्नलिखित में से कौन-सा एक सही है?

(a)  कथन-I और कथन-II दोनों सही हैं तब कथन-II, कथन-1 की सही व्याख्या है।
(b)  कथन-I और कथन-II दोनों सही हैं तथा कथन-II, कथन-1 की सही व्याख्या नहीं है।
(c)  कथन-I सही है किन्तु कथन-II गलत है
(d)  कथन-1 गलत है किन्तु कथन-II सही है

उत्तर:(c)

व्याख्या:

  • मिस्र और जॉर्डन केवल दो अरब देश हैं जिनके इज़रायल के साथ औपचारिक राजनयिक संबंध हैं, हालाँकि सऊदी अरब और मोरक्को जैसे कुछ अन्य अरब देशों ने कथित तौर पर इसके साथ वर्षों से बैक-चैनल संचार किया है। अतः कथन 1 सही है।
  • 1970 के दशक से सऊदी अरब ने मध्य पूर्वी देशों और उनके तथा संयुक्त राज्य अमेरिका के बीच क्षेत्रीय समन्वय की नीति का प्रयोग किया था।
  • अरब-इजरायल मामलों में पिछले कुछ दशकों में सऊदी अरब ने जो विशेष स्थिति हासिल की है, वह क्षेत्रीय समन्वयक के रूप में सऊदी की भूमिका को दर्शाती है।
  • यह विश्लेषण 1980 के दशक में अरब दलों और इज़रायल के बीच शांति के लिये सऊदी पहल से संबंधित है- ( वर्ष 1981-1982 का "द फहद प्लान" जिसने "द फ़ेज़ रेज़ोल्यूशन" का मार्ग प्रशस्त हुआ) और इक्कीसवीं सदी की शुरुआत में "द अब्दुल्लाह" पहल" जिससे "अरब शांति पहल" (2002-2007) का मार्ग प्रशस्त हुआ। हालाँकि, इज़राइल ने आधिकारिक तौर पर इस पहल पर हस्ताक्षर नहीं किये हैं। अतः कथन 2 सही नहीं है।

कुछ अन्य पहलें इस प्रकार हैं:

अब्राहम एकॉर्ड :

  • ‘अब्राहम एकॉर्ड (Abraham Accord) इज़रायल और अरब देशों के बीच पिछले 26 वर्षों में पहला शांति समझौता है
  • गौरतलब है कि इससे पहले वर्ष 1994 में इज़रायल और जॉर्डन के बीच शांति समझौते पर हस्ताक्षर किये गए थे।
  • अमेरिकी राष्ट्रपति के अनुसार, यह समझौता पूरे अरब क्षेत्र में व्यापक शांति स्थापना के लिये एक नींव का काम करेगा, हालाँकि इस समझौते में इज़रायल-फिलिस्तीन संघर्ष के संदर्भ में कोई बात नहीं की गई है।
  • ध्यातव्य है कि 13 अगस्त, 2020 को इज़रायल-यूएई शांति समझौते की घोषणा के बाद 11 सितंबर को बहरीन-इज़रायल समझौते की घोषणा की गई थी।
  • अमेरिकी राष्ट्रपति ने अन्य अरब देशों के भी इस समझौते में शामिल होने के संकेत दिये हैं।

I2U2 पहल: समुद्री सुरक्षा, बुनियादी ढाँचे और परिवहन से संबंधित मुद्दों के समाधान के लिए I2U2 का गठन अक्टूबर, 2021 में इज़रायल और UAEके बीच अब्राहम समझौते के बाद किया गया था।

  • उस समय इसे 'इंटरनेशनल फोरम फॉर इकोनॉमिक कोऑपरेशन' कहा जाता था।
  • जिसे 'वेस्ट एशियन क्वाड' कहा जाता था।

स्रोत:

55. खेल पुरस्कारों के संदर्भ में, निम्नलिखित युग्मों पर विचार कीजिये:

1. मेजर ध्यानचंद खेल रत्न पुरस्कार किसी खिलाड़ी द्वारा पिछले चार वर्षों की अवधि के दौरान सबसे शानदार और उत्कृष्ट प्रदर्शन के लिये
2. अर्जुन पुरस्कार किसी खिलाड़ी द्वारा जीवन- काल की उपलब्धियों के लिये
3. द्रोणाचार्य पुरस्कार उन प्रतिष्ठित प्रशिक्षकों को सम्मानित करने के लिये जिन्होंने सफलतापूर्वक खिलाड़ियों या टीमों को प्रशिक्षित किया है
4. राष्ट्रीय खेल प्रोत्साहन पुरस्कार  खिलाड़ियों द्वारा रिटायर होने के बाद भी किये गए योगदान की सराहना करने के लिये

उपर्युक्त में से कितने युग्म सही सुमेलित हैं?

(a) केवल एक
(b) 
केवल दो
(c) केवल तीन
(d) 
सभी चार

उत्तर:(b)

व्याख्या:

  • राजीव गांधी खेल रत्न पुरस्कार का नाम बदलकर मेजर ध्यानचंद खेल रत्न पुरस्कार कर दिया गया है। परिवर्तित मेजर ध्यानचंद खेल रत्न पुरस्कार के अंतर्गत 25 लाख रुपए की नकद राशि पुरस्कार के रूप में दी जाती है। यह चार वर्ष की अवधि में एक खिलाड़ी द्वारा खेल के क्षेत्र में शानदार और उत्कृष्ट प्रदर्शन के लिये युवा मामले और खेल मंत्रालय द्वारा दिया जाने वाला सर्वोच्च खेल पुरस्कार है। अतः युग्म 1 सही सुमेलित है।
  • राष्ट्रीय खेल आयोजनों में उत्कृष्ट उपलब्धि को मान्यता देने हेतु भारत सरकार द्वारा वर्ष 1961 में अर्जुन पुरस्कार की स्थापना की गई थी। यह विगत चार वर्षों की अवधि में अच्छे प्रदर्शन और नेतृत्त्व, खेल कौशल एवं अनुशासन के लिये दिया जाता है। अतः युग्म 2 सही सुमेलित नहीं है।
  • खेल प्रक्षिक्षण में उत्कृष्टता को सम्मानित करने हेतु भारत सरकार द्वारा वर्ष 1985 में द्रोणाचार्य पुरस्कार की स्थापना की गई थी। यह प्रशिक्षकों को निरंतर आधार पर उत्कृष्ट एवं मेधावी कार्य करने तथा खिलाड़ियों को अंतर्राष्ट्रीय प्रतियोगिताओं में उत्कृष्ट प्रदर्शन करने में सक्षम बनाने के लिये दिया जाता है। अतः युग्म 3 सही सुमेलित है।
  • राष्ट्रीय खेल प्रोत्साहन पुरस्कार वर्ष 2009 में स्थापित किया गया था। यह कॉर्पोरेट संस्थाओं (निजी और सार्वजनिक दोनों क्षेत्रों में), खेल नियंत्रण बोर्डों, राज्य और राष्ट्रीय स्तर पर खेल निकायों सहित गैर सरकारी संगठनों को दिया जाता है जिन्होंने खेल संवर्द्धन एवं विकास के क्षेत्र में एक महत्त्वपूर्ण भूमिका निभाई है। अतः युग्म 4 सही सुमेलित नहीं है।

स्रोत…

56. 44वें शतरंज ओलिम्पियाड, 2022 के बारे में, निम्नलिखित कथनों पर विचार कीजिये:

  1. यह पहली बार था, जब शतरंज ओलिम्पियाड भारत में आयोजित किया गया।
  2. इसके अधिकृत शुभंकर को ‘तंबिनाम दिया गया था।
  3. ओपेन सेक्शन में जीतने वाली टीम के लिये ट्रॉफी, वेरा मेंचिक कप होती है।
  4. महिला विभाग (सेक्शन) में जीतने वाली टीम के लिये ट्रॉफी, हैमिल्टन-रसेल कप होती है।

उपर्युक्त में से कितने कथन सही हैं?

(a) केवल एक                        
(b) 
केवल दो
(c) केवल तीन                       
(d) 
सभी चार

उत्तर:(b)

व्याख्या:

  • वर्ष 1927 में वेस्टमिंस्टर सेंट्रल हॉल में लंदन में आयोजित पहला आधिकारिक ओलंपियाड 'टूर्नामेंट ऑफ नेशंस' के रूप में जाना जाता था। शतरंज ओलंपियाड का आयोजन पहली बार शतरंज की जन्मस्थली अर्थात् भारत में किया जा रहा है। यह 3 दशकों के बाद पहली बार एशिया में आयोजित हो रहा है। इसमें भाग लेने वाले देशों एवं टीमों की संख्या सबसे अधिक है। साथ ही इसमें महिला वर्ग में सबसे अधिक महिला खिलाड़ी भाग ले रही हैं। अतः कथन 1 सही है।
  • 44वें शतरंज ओलंपियाड का आधिकारिक शुभंकर 'थम्बी (Thambi)' है। तमिल भाषा में 'थम्बी' शब्द का अर्थ है - छोटा भाई। अतः कथन 2 सही है।
  • ओपन सेक्शन में विजेता टीम को हैमिल्टन-रसेल कप प्रदान किया जाता है, जिसे इंग्लिश मैग्नेट फ्रेडरिक हैमिल्टन-रसेल ने प्रथम ओलंपियाड (लंदन 1927) के लिये पुरस्कार के रूप में पेश किया था। अतः कथन 3 सही नहीं है।
  • पहली महिला विश्व शतरंज चैंपियन के सम्मान में विजेता महिला टीम की ट्रॉफी को वेरा मेनचिक कप के रूप में जाना जाता है। अतः कथन 4 सही नहीं है।

स्रोत:

57. निम्नलिखित युग्मों पर विचार कीजिये:

समाचारों में उल्लिखित जिस देश में अवस्थित है

  संघर्ष का क्षेत्र

  1. डोनबास    :  सीरिया
  2. काचिन     :  इथियोपिया
  3. टिग्रे         :  उत्तरी यमन

उपर्युक्त में से कितने युग्म सही सुमेलित हैं?

(a)  केवल एक
(b)  केवल दो
(c)  सभी तीन
(d)  कोई भी नहीं

उत्तर:(d)

व्याख्या:

  • डोनबास यूक्रेन के डोनेट्स बेसिन में स्थित है। यह दक्षिणपूर्वी यूरोप का एक बड़ा खनन और औद्योगिक क्षेत्र है, जो अपने कोयले के वृहद् भंडार हेतु प्रसिद्ध है। डोनबास के औद्योगिक क्षेत्र में डोनेट्स्क एवं लुहांस्क के अधिकांश यूक्रेनी ओब्लास्टी (प्रांत) शामिल हैं। अतः युग्म 1 सही सुमेलित नहीं है।
  • "काचिन" जिंगपॉ शब्द "गाख्येन" से लिया गया है, जिसका अर्थ है ‘लाल पृथ्वी (Red Earth)’, यह ऊपरी इरावदी नदी की दो शाखाओं की घाटी क्षेत्र में प्रभावशाली पारंपरिक शासकों का सबसे बड़ा संकेद्रण है। काचिन लोग ज़्यादातर म्याँमार (बर्मा) के काचिन राज्य में पाए जाते हैं, साथ ही उत्तरी शान राज्य के कुछ क्षेत्रों में, चीन के युन्नान के दक्षिण-पश्चिम में एवं भारत के पूर्वोत्तर राज्यों (असम व अरुणाचल प्रदेश) में पाए जाते हैं। अतः युग्म 2 सही सुमेलित नहीं है।
  • Tigray/टाइग्रे/टिग्रे उत्तरी इथियोपिया का ऐतिहासिक क्षेत्र है। अतः युग्म 3 सही सुमेलित नहीं है।

स्रोत..

58. हाल के वर्षों में, चाड, गिनी, माली और सूडान ने निम्नलिखित में से किस एक कारण से अंतर्राष्ट्रीय ध्यान आकर्षित किया, जो इन सभी देशों से संबंधित है?

(a) दुर्लभ मृदा तत्त्वों से समृद्ध निक्षेपों की खोज
(b) चीनी सैन्य बेस का स्थापित होना
(c) सहारा मरुस्थल का दक्षिणाभिमुखी प्रसार
(d) सफल तख्ता-पलट

उत्तर:(d)

व्याख्या:

  • अफ्रीकियों ने पिछले कुछ वर्षों में सैन्य तख्तापलट की शृंखला का अनुभव किया है। हाल के अधिकांश सैन्य अधिग्रहण पश्चिम अफ्रीका में हुए हैं, हालाँकि विश्व के अन्य क्षेत्रों में भी इस प्रवृत्ति के व्यापक प्रसार का भय बना हुआ है।
  • पश्चिम अफ्रीका में तख्तापलट की प्रवृत्ति चाड से शुरू हुई। वहीं माली ने 18 महीनों के भीतर दो सैन्य तख्तापलट का अनुभव किया है। आखिरी तख्तापलट अगस्त 2020 में हुआ था।
  • सूडान की सेना ने नागरिक प्रशासन के साथ अपने तनावपूर्ण सह-अस्तित्त्व को समाप्त करने हेतु वर्ष 2021 में तख्तापलट किया।
  • वर्ष 2021 में गिनी की सेना द्वारा राष्ट्रपति को पद से हटाना निरंकुश अतिक्रमण, आर्थिक कुप्रबंधन और लोकतांत्रिक मानदंडों के पतन का परिणाम था, जो क्षेत्रीय संगठनों और अंतर्राष्ट्रीय सहयोगियों की उभरती हुई तख्तापलट प्रक्रिया का समाधान और संबोधित करने की अपर्याप्तता को उजागर करता है।
  • यही कारण है कि ये देश अपने सफल तख्तापलट हेतु खबरों में हैं।
  • अतः विकल्प (d) सही है।

स्रोत…

59. निम्नलिखित भारी उद्योगों पर विचार कीजिये:

  1. उर्वरक संयंत्र
  2. तेलशोधक कारखाने
  3. इस्पात संयंत्र

उपर्युक्त में से कितने उद्योगों के विकार्बनन में हरित हाइड्रोजन की महत्त्वपूर्ण भूमिका होने की अपेक्षा है?

(a) केवल एक                        
(b) 
केवल दो
(c) सभी तीन                         
(d) 
कोई भी नहीं

उत्तर:(c)

व्याख्या:

हरित हाइड्रोजन:

  • हाइड्रोजन प्रमुख औद्योगिक ईंधन है जिसके अमोनिया (एक प्रमुख उर्वरक), स्टील, रिफाइनरियों और विद्युत उत्पादन सहित विभिन्न प्रकार के अनुप्रयोग हैं।
  • हालाँकि इस प्रकार निर्मित सभी हाइड्रोजन को तथाकथित 'ब्लैक या ब्राउन' हाइड्रोजन कहा जाता है क्योंकि वे कोयले से उत्पन्न होते हैं।
  • लेकिन जब विद्युत धारा जल से गुज़रती है, तो यह इलेक्ट्रोलिसिस के माध्यम से इसे मूल ऑक्सीजन और हाइड्रोजन में खंडित करती है। यदि इस प्रक्रिया के लिये उपयोग की जाने वाली विद्युत का स्रोत, पवन या सौर ऊर्जा जैसे नवीकरणीय स्रोत हैं तो इस प्रकार उत्पादित हाइड्रोजन को हरित हाइड्रोजन कहा जाता है।

हरित हाइड्रोजन के उत्पादन की आवश्यकता:

  • हरित हाइड्रोजन विशेष रूप से शून्य उत्सर्जन के साथ ऊर्जा के सबसे स्वच्छ स्रोतों में से एक है। इसका उपयोग कारों के लिये ईंधन सेल के रूप में या उर्वरक और इस्पात निर्माण जैसे अत्यधिक ऊर्जा खपत वाले उद्योगों में किया जा सकता है।
  • हरित हाइड्रोजन वातावरण में CO2 के उत्पादन के बिना कच्चे तेल के डीसल्फराइज़ेशन में सहायता कर सकता है, इसलिये यह एक स्वच्छ, ऑन-साइट हरित हाइड्रोजन आपूर्ति प्रदान कर सकता है जो रिफाइनिंग प्रक्रिया को विकार्बनित (decarbonise) करेगा और उत्सर्जन को कम करेगा।

अतः विकल्प (c) सही है।

स्रोत:

60. G-20 के बारे में, निम्नलिखित कथनों पर विचार कीजिये:

  1. G-20 समूह की मूल रूप से स्थापना वित्त मंत्रियों और केंद्रीय बैंक के गवर्नरों द्वारा अंतर्राष्ट्रीय आर्थिक और वित्तीय मुद्दों पर चर्चा के मंच के रूप में की गई थी।
  2. डिजिटल सार्वजनिक बुनियादी ढाँचा भारत की G-20 प्राथमिकताओं में से एक है।

उपर्युक्त कथनों में से कौन-सा/से सही है/हैं?

(a) केवल 1                            
(b) 
केवल 2
(c) 1 और 2 दोनों               
(d) 
न तो 1 और न ही 2

उत्तर:(c)

व्याख्या:

G20:

  • इसकी स्थापना वर्ष 1999 में एशियाई वित्तीय संकट के बाद वैश्विक आर्थिक और वित्तीय मुद्दों पर चर्चा करने के लिए वित्त मंत्रियों और केंद्रीय बैंक के गवर्नरों के लिये एक मंच के रूप में की गई थी। अतः कथन 1 सही है।
  • वर्ष 2007 एवं 2009 के वैश्विक आर्थिक और वित्तीय संकट के कारण इसमें राज्य/सरकार के प्रमुखों को शामिल करने के रूप में अपग्रेड किया गया था।
  • G20 शिखर सम्मेलन प्रतिवर्ष आयोजित किया जाता है।
  • यह शुरुआत में बड़े पैमाने पर व्यापक आर्थिक मुद्दों पर केंद्रित था लेकिन इसके बाद इसमें व्यापार, सतत् विकास, स्वास्थ्य, कृषि, ऊर्जा, पर्यावरण, जलवायु परिवर्तन के साथ भ्रष्टाचार विरोधी पहलों को शामिल किया गया था।

डिजिटल सार्वजनिक बुनियादी ढाँचा (DPI):

  • यह कई साध्यों हेतु एक साझा साधन है। यह डिजिटल परिवर्तन का एक महत्वपूर्ण प्रवर्तक है और बड़े पैमाने पर सार्वजनिक सेवा वितरण में सुधार करने में सहायक रहा है।
  • अच्छी तरह से डिज़ाइन और कार्यान्वित होने के साथ यह देशों को उनकी राष्ट्रीय प्राथमिकताओं को प्राप्त करने और सतत् विकास लक्ष्यों को गति देने में मदद कर सकता है।
  • इलेक्ट्रॉनिक्स और सूचना प्रौद्योगिकी मंत्रालय (MeitY), भारत सरकार ने भारत की G20 अध्यक्षता के दौरान डिजिटल सार्वजनिक बुनियादी ढाँचे पर सामूहिक कार्रवाई करने के लिए संयुक्त राष्ट्र विकास कार्यक्रम (UNDP) के साथ भागीदारी की है। अतः कथन 2 सही है।

स्रोत:

61. भारतीय इतिहास के संदर्भ में, अलेक्जेंडर री., ए.एच. लॉन्गहर्स्ट, रॉबर्ट स्वेल, जेम्स बर्गेस और किस गतिविधि से जुड़े थे?

(a)  पुरातात्त्विक उत्खनन
(b)  औपनिवेशिक भारत में अंग्रेजी प्रेस की स्थापना
(c)  देशी रजवाड़ों में गिरजाघरों की स्थापना
(d)  औपनिवेशिक भारत में रेल का निर्माण

उत्तर:(a)

व्याख्या:

  • भारत में कार्य करने वाले ब्रिटिश पुरातत्त्वविद् अलेक्जेंडर री को वर्ष 1903-04 में आदिचनल्लूर में 9,000 से अधिक वस्तुओं के खजाने का पता लगाने हेतु जाना जाता था।
  • ए. एच. लॉन्गहर्स्ट चंद्रकेतुगढ़ बंगाल में खुदाई से जुड़े पुरातत्त्वविद् थे।
  • रॉबर्ट सेवेल: रॉबर्ट सेवेल (1845 - 1925) औपनिवेशिक भारत में मद्रास प्रेसीडेंसी में कलेक्टर और मजिस्ट्रेट थे। वे इतिहास के विद्वान थे। साथ ही तत्कालीन पुरातत्त्व विभाग के प्रभारी भी थे।
  • जेम्स बर्गेस: जेम्स बर्गेस वर्ष 1872 में द इंडियन एंटीकरी के संस्थापक और 19वीं शताब्दी में भारत के महत्त्वपूर्ण पुरातत्त्वविद् थे।
  • वाल्टर इलियट: वह भारत में स्कॉटिश सिविल सेवक, पुरातत्त्वविद् , मुद्राशास्त्री एवं कलेक्टर थे। उन्होंने वर्ष 1845 में अमरावती में खुदाई की। उनके कुछ खुदाई से प्राप्त स्रोत आज भी सरकारी संग्रहालय मद्रास (चेन्नई) में मौजूद हैं।

स्रोत…

62. निम्नलिखित युग्मों पर विचार कीजिये:

 स्थल             जिसके लिये जाना जाता है

  1. बेसनगर         :  शैव गुफा मंदिर
  2. भाजा           : बौद्ध गुफा मंदिर
  3. सित्तनवासल    : जैन गुफा मंदिर

उपर्युक्त में से कितने युग्म सही सुमेलित हैं?

(a) केवल एक                       
(b) 
केवल दो
(c) सभी तीन                         
(d) 
कोई भी नहीं

उत्तर: (b)

व्याख्या:

  • हेलियोडोरस स्तंभ एक प्रस्तर स्तंभ है जिसे लगभग 113 ईसा पूर्व मध्य भारत के बेसनगर (विदिशा, मध्य प्रदेश के पास) में स्थापित किया गया था। वैष्णव मत से प्रेरित होकर इस स्तंभ को हेलियोडोरस द्वारा गरुड़-स्तंभ के रूप में संदर्भित किया गया था। अत: 1 सही नहीं है।
  • भाजा गुफाएँ दूसरी शताब्दी ईसा पूर्व में निर्मित भारत के पुणे शहर में स्थित 22 रॉक-कट गुफाओं का एक समूह है। यह गुफाएँ भाजा गाँव के समीप 400 फीट ऊँचीई पर हैं। यह अरब सागर की ओर से आने वाले महत्त्वपूर्ण प्राचीन व्यापार मार्ग पर स्थित हैं।
  • भारतीय पुरातत्व सर्वेक्षण की अधिसूचना संख्या 2407-A द्वारा इस शिलालेख और गुफा मंदिर को राष्ट्रीय महत्त्व के स्मारक के रूप में संरक्षित किया गया है। यह महाराष्ट्र के प्रारंभिक बौद्ध शिक्षा केंद्रों से संबंधित है। इन गुफाओं में बौद्ध धर्म की विशेषताओं से संबंधित कई स्तूप हैं। अत: 2 सही है।
  • सित्तनवासल गुफाएँ (अरिवर कोइल): यह तमिलनाडु में पुदुक्कोट्टई शहर से 16 किमी. उत्तर पश्चिम में स्थित हैं। चट्टानों को काटकर बनाई गई ये प्रसिद्ध गुफाएँ जैन मंदिरों के रूप में प्रसिद्ध हैं। अत: 3 सही है।

Reference:

63. निम्नलिखित कथनों पर विचार कीजिये:

कथन-1: 7 अगस्त को राष्ट्रीय हथकरघा दिवस के रूप में घोषित किया गया है।

कथन-II: 1905 में, इसी दिन स्वदेशी आंदोलन शुरू किया गया था।

उपर्युक्त कथनों के बारे में, निम्नलिखित में से कौन-सा एक सही है?

(a)  कथन-I और कथन-II दोनों सही हैं तथा कथन-II, कथन-I की सही व्याख्या है
(b)  कथन- I और कथन-II दोनों सही हैं तथा कथन-II, कथन-I की सही व्याख्या नहीं है
(c)  कथन- I सही है किन्तु कथन-II गलत है
(d)  कथन- I गलत है किन्तु कथन-II सही है

उत्तर:(a)

व्याख्या:

  • हथकरघा बुनकरों को सम्मानित करने एवं देश के सामाजिक विकास में हथकरघा उद्योग के महत्त्व के बारे में जागरूकता विकसित करने तथा हथकरघा उद्योग को बढ़ावा देने के उद्देश्य से प्रति वर्ष 7 अगस्त को राष्ट्रीय हथकरघा दिवस मनाया जाता है। अतः कथन 1 सही है।
  • 7 अगस्त को 'स्वदेशी' आंदोलन (1905) की स्मृति में राष्ट्रीय हथकरघा दिवस को नामित किया गया था। इसलिये राष्ट्रीय हथकरघा दिवस एवं स्वदेशी आंदोलन के बीच गहरा संबंध है। अतः कथन 2 सही है।

स्रोत:

64. भारत की ध्वज-संहिता, 2002 के अनुसार, भारत के राष्ट्रीय ध्वज के बारे में निम्नलिखित कथनों पर विचार कीजिये:

कथन-I: भारत के राष्ट्रीय ध्वज का एक मानक आमाप 600 मि०मी० *400 मि०मी० है।

कथन-II: ध्वज की लंबाई से ऊँचाई (चौड़ाई) का अनुपात 3 : 2 होगा।

उपर्युक्त कथनों के बारे में, निम्नलिखित में से कौन-सा एक सही है?

(a)  कथन-I और कथन-II दोनों सही हैं तथा कथन-II, कथन-I की सही व्याख्या है।
(b)  कथन-I और कथन-II दोनों सही हैं तथा कथन-II, कथन-I की सही व्याख्या नहीं है
(c)  कथन-I सही है किन्तु कथन-II गलत है
(d)  कथन-I गलत है किन्तु कथन-II सही है

उत्तर: (d)

व्याख्या:

  • राष्ट्रीय ध्वज का मानक आमाप निम्नानुसार है:

1

6300x4200

2

3600x2400

3

2700x1800

4

1800x1200

5

1350x900

6

900x600

7

450x300

8

225x150

9

150x100

  • अतः कथन 1 सही नहीं है।
  • राष्ट्रीय ध्वज का आकार आयताकार है। ध्वज की लंबाई एवं ऊँचाई (चौड़ाई) का अनुपात 3:2 होगा। अतः कथन 2 सही है।

स्रोत…

65. संविधान दिवस के बारे में, निम्नलिखित कथनों पर कीजिये:

कथन-I: नागरिकों के बीच सांविधानिक मूल्यों को संवर्द्धित करने के लिये संविधान दिवस प्रतिवर्ष 26 नवंबर को मनाया जाता है।

कथन-II: 26 नवंबर, 1949 को, भारत की संविधान सभा में भारत के संविधान का प्रारूप तैयार करने के लिये डॉ० बी० आर० अम्बेडकर की अध्यक्षता में प्रारूपण समिति बनाई।

उपर्युक्त कथनों के बारे में, निम्नलिखित में से कौन-सा एक सही है?

(a)  कथन-I और कथन-II दोनों सही हैं तथा कथन-II, कथन-I की सही व्याख्या है।
(b)  कथन-I और कथन-II दोनों सही है तथा कथन-II, कथन-1 की सही व्याख्या नहीं है।
(c)  कथन-I सही है किन्तु कथन-II गलत है।
(d)  कथन-I गलत है किन्तु कथन-II सही है।

उत्तर:(c)

व्याख्या:

  • 26 नवंबर को संविधान दिवस के रूप में मनाया जाता है। इसी दिन वर्ष 1949 में भारतीय संविधान सभा ने औपचारिक रूप से भारत के संविधान को अपनाया, जो आगे चलकर 26 जनवरी, 1950 को लागू हुआ। अतः कथन 1 सही है।
  • 29 अगस्त, 1947 को संविधान सभा ने डॉ. बी.आर.अंबेडकर की अध्यक्षता में प्रारूप समिति का गठन किया। जिसने भारतीय संविधान के प्रारूप को तैयार किया। अतः कथन 2 सही नहीं है।
  • 13 दिसंबर, 1946 को संविधान सभा ने औपचारिक रूप से भारतीय संविधान के निर्माण की शुरुआत की। जवाहरलाल नेहरू ने उद्देश्य प्रस्ताव पेश किया, जिसका उद्देश्य भारत को एक स्वतंत्र संप्रभु गणराज्य के रूप में घोषित करना तथा इसके संचालन हेतु एक संविधान का निर्माण करना था। इस प्रस्ताव ने संविधान सभा के कार्य का मार्गदर्शन करने हेतु सामान्य सिद्धांतों की स्थापना की। 22 जनवरी, 1947 को संविधान सभा ने इस प्रस्ताव को अपनाया।

स्रोत…

66. निम्नलिखित कथनों पर विचार कीजिये:

कथन-I: स्विट्ज़रलैंड, मूल्य के आधार पर, स्वर्ण के अग्रणी निर्यातकों में से एक है।

कथन-II: स्विट्ज़रलैंड के पास विश्व का दूसरा विशालतम स्वर्ण निचय (रिज़र्व) है।

उपर्युक्त कथनों के बारे में, निम्नलिखित में से कौन-सा एक सही है?

(a)  कथन-I और कथन-II दोनों सही है तथा कथन-II, कथन-I की सही व्याख्या है
(b)  कथन-I और कथन-II दोनों सही है तथा कथन-II, कथन-1 की सही व्याख्या नहीं है।
(c)  कथन-I सही है किन्तु कथन-II गलत है
(d)  कथन-I गलत है किन्तु कथन-II सही है

उत्तर: (c)

व्याख्या:

  • स्विट्ज़रलैंड मूल्य के आधार पर, स्वर्ण के अग्रणी निर्यातकों में से एक है। अतः कथन 1 सही है।
  • वर्ष 2021 में स्विट्ज़रलैंड का स्वर्ण निर्यात लगभग 87 बिलियन अमेरिकी डॉलर के मूल्य के बराबर था। स्विट्ज़रलैंड लगातार मूल्य के आधार पर विश्व का अग्रणी स्वर्ण निर्यातक देश है।
  • मूल्य के आधार पर स्वर्ण का सबसे बड़ा निर्यातक होने के अतिरिक्त, स्विट्ज़रलैंड विश्व में स्वर्ण  का लगातार सबसे बड़ा आयातक भी रहा है। इसे इस तथ्य से समझाया जा सकता है कि स्विट्ज़रलैंड विश्व के लगभग 70 प्रतिशत स्वर्ण को परिष्कृत करता है: स्विट्ज़रलैंड अपरिष्कृत स्वर्ण का आयात करता है एवं  इसे अपने परिष्कृत रूप में निर्यात करता है।
  • 1,040 टन स्वर्ण के साथ, स्विस नेशनल बैंक (SNB) के पास विश्व का सातवाँ सबसे बड़ा स्वर्ण का भंडार है। अतः कथन 2 सही नहीं है।

स्रोत…

67. निम्नलिखित कथनों पर विचार कीजिये:

कथन-I: हाल ही में, संयुक्त राज्य अमरीका (यू.एस.ए.) और यूरोपीय संघ (ई.यू.) ने ‘व्यापार और प्रौद्योगिकी परिषद् प्रारंभ की है।

कथन-II: संयुक्त राज्य अमरीका और यूरोपीय संघ यह दावा करते हैं कि वे इसके माध्यम से तकनीकी प्रगति और भौतिक उत्पादकता को अपने नियंत्रण में लाने का प्रयास कर रहे हैं।

उपर्युक्त कथनों के बारे में, निम्नलिखित में से कौन-सा एक सही है?

(a)  कथन-I और कथन-II दोनों सही है तथा कथन-II, कथन-I की सही व्याख्या है
(b)  कथन-I और कथन-II दोनों सही है तथा कथन-II, कथन-I की सही व्याख्या नहीं है
(c)  कथन-I सही है किन्तु कथन-II गलत है
(d)  कथन-I गलत है किन्तु कथन-II सही है

उत्तर:(c )

व्याख्या:

  • हाल ही में, संयुक्त राज्य अमेरिका (USA) और यूरोपीय संघ (EU) ने 'व्यापार एवं प्रौद्योगिकी परिषद्' शुरू की है।
  • यूरोपीय संघ-अमेरिका व्यापार और प्रौद्योगिकी परिषद् संयुक्त राज्य अमेरिका तथा यूरोपीय संघ के लिये प्रमुख वैश्विक व्यापार, आर्थिक एवं प्रौद्योगिकी मुद्दों के दृष्टिकोण का समन्वय करने और साझा मूल्यों के आधार पर ट्रान्स-अटलांटिक व्यापार एवं आर्थिक संबंधों को मज़बूत करने के लिये एक मंच के रूप में कार्य करता है। इसकी स्थापना 15 जून, 2021 को ब्रुसेल्स में यूरोपीय संघ-अमेरिका शिखर सम्मेलन के दौरान की गई थी। अतः कथन 1 सही है।
  • अमेरिका और यूरोपीय संघ ने कृत्रिम बुद्धिमत्ता, अर्द्धचालक और सूचना तथा संचार प्रौद्योगिकी सेवाओं जैसे मुद्दों पर ध्यान देने के साथ TTC के कार्यों पर चर्चा की।
  • परिषद् के माध्यम से यूरोपीय संघ और अमेरिका एक साथ मिलकर कार्य कर रहे हैं:

1. व्यापार एवं प्रौद्योगिकी के सामान्य मूल्यों को बनाए रखते हुए समाज और अर्थव्यवस्था की सेवा सुनिश्चित करना
2. तकनीकी एवं औद्योगिक नेतृत्त्व को मज़बूत करना
  i. द्विपक्षीय व्यापार और निवेश का विस्तार करना

अतः कथन 2 सही नहीं है।

संदर्भ:

68. निम्नलिखित कथनों पर विचार कीजिये

कथन-I: वस्तुओं के वैश्विक निर्यात में भारत का निर्यात 3.2% है।

कथन-II: भारत में कार्यरत अनेक स्थानीय कंपनियों एवं भारत में कार्यरत कुछ विदेशी कंपनियों ने भारत की ‘उत्पादन-आधारित प्रोत्साहन (प्रोडक्शन-लिंक्ड इंसेंटिव)’  योजना का लाभ उठाया है।

उपर्युक्त कथनों के बारे में, निम्नलिखित में से कौन-सा एक सही है?

(a)  कथन-I और कथन-II दोनों सही हैं तथा कथन-II, कथन-I की सही व्याख्या है।
(b)  कथन-I और कथन-II दोनों सही हैं तथा कथन-II, कथन-I की सही व्याख्या नहीं है
(c)  कथन-I सही है किन्तु कथन-II गलत है
(d)  कथन-I गलत है किन्तु कथन-II सही है

उत्तर:(d )

व्याख्या:

  • हाल ही में विश्व व्यापार संगठन की वैश्विक व्यापार आउटलुक एवं स्टेटिक्स रिपोर्ट के अनुसार, भारत वस्तुओं के वैश्विक निर्यात में 1.8 % हिस्सेदारी रखता है। अतः कथन 1 गलत है।
  • वर्ष 2024 के लिये अनुमानित 3.2% के लक्ष्य तक पहुँचने से पूर्व वर्ष 2023 में विश्व वाणिज्यिक वस्तु व्यापार की मात्रा में 1.7% वृद्धि का अनुमान है।
  • उत्पादन आधारित प्रोत्साहन (प्रोडक्शन लिंक्ड इनिशिएटिव) योजना कंपनियों को भारत में निर्मित उत्पादों के वृद्धिशील विक्रय पर प्रोत्साहन प्रदान करती है। इसका उद्देश्य विदेशी कंपनियों को भारत में इकाइयाँ स्थापित करने के लिये आकर्षित करना है, जबकि स्थानीय कंपनियों को अपनी विनिर्माण इकाइयों का विस्तार करने एवं अधिक रोजगार सृजन करने तथा आयात पर देश की निर्भरता को कम करने के लिये प्रोत्साहित करना है। अतः कथन 2 सही है।

स्रोत…

69. निम्नलिखित कथनों पर विचार कीजिये:

यूरोपीय संघ का ‘स्थिरता एवं संवृद्धि समझौता (स्टेबिलिटी एंड ग्रोथ पैक्ट)’ ऐसी संधि है, जो

  1. यूरोपीय संघ के देशों के बजटीय घाटे के स्तर को सीमित करती है
  2. यूरोपीय संघ के देशों के लिये अपनी आधारिक संरचना सुविधाओं को आपस में बाँटना सुकर बनाती है
  3. यूरोपीय संघ के देशों के लिये अपनी प्रौद्योगिकियों को आपस में बाँटना सुकर बनाती है

उपर्युक्त में से कितने कथन सही है?

(a)  केवल एक                        
(b)  
केवल दो
(c)  सभी तीन                         
(d)  
कोई भी नहीं

उत्तर:(a)

व्याख्या: 

  • स्थिरता एवं समृद्धि समझौता एक राजनीतिक समझौता है जो यूरोपीय मौद्रिक संघ (EMU) के सदस्य राज्यों के राजकोषीय घाटे तथा सार्वजनिक ऋण की सीमा निर्धारित करता है। अतः कथन 1 सही है।
  • इन दिशानिर्देशों का उद्देश्य किसी सदस्य राज्य की गैर-जिम्मेदार बजटीय नीतियों द्वारा पूरे यूरो क्षेत्र की आर्थिक स्थिरता को असंतुलित होने से रोकने तथा EMU के अंतर्गत सार्वजनिक वित्त के प्रबंधन को सुनिश्चित करना है।
  • यूरोपीय संघ स्थिरता एवं समृद्धि समझौता आधारिक संरचना सुविधाओं और प्रौद्योगिकियों को साझा करने से संबंधित कोई प्रावधान नहीं करता है। अतः कथन 2 और 3 सही नहीं हैं।

स्रोत…

70. निम्नलिखित कथनों पर विचार कीजिये:

  1. हाल ही में, संयुक्त राष्ट्र के सभी देशों ने अंतर्राष्ट्रीय प्रवास के लिये अभी तक की पहली संविदा, ‘सुरक्षित, व्यवस्थित और नियमित प्रवास के लिये वैश्विक संविदा [ग्लोबल कॉम्पैक्ट पर सेफ, ऑर्डरली ड रेगुलर माइग्रेशन (जी.सी.एम.)]’ अपनाई है।
  2. जी.सी.एम. में अधिकथित उद्देश्य और प्रतिबद्धताएँ यू.एन. सदस्य देशों पर बाध्यकारी है।
  3. जी.सी.एम. अपने उद्देश्यों और प्रतिबद्धताओं के अंतर्गत आंतरिक प्रवास या आंतरिक रूप से विस्थापित लोगों के लिये भी कार्य करती है।

उपर्युक्त में से कितने कथन सही है?

(a) केवल एक                        
(b) 
केवल दो
(c) सभी तीन                         
(d) 
कोई भी नहीं

उत्तर: (d)

व्याख्या:

  • सितंबर 2016 में अफ्रीका और पश्चिम एशिया के काफी प्रवासी यूरोप पहुँच गए जिसके साथ ही संयुक्त राष्ट्र के सभी 193 सदस्य देशों ने मानव गतिशीलता हेतु व्यापक दृष्टिकोण तथा वैश्विक स्तर पर सहयोग बढ़ाने के लिये शरणार्थियों और प्रवासियों से संबंधित न्यूयॉर्क घोषणा को अपनाया, जिसके परिणामस्वरूप वर्ष 2018 में ग्लोबल कॉम्पैक्ट को अपनाया गया था।
  • न्यूयॉर्क घोषणा में शरणार्थियों और प्रवासियों के प्रति प्रतिबद्धताओं के साथ ग्लोबल कॉम्पैक्ट ऑन रिफ्यूजी तथा ग्लोबल कॉम्पैक्ट फॉर सेफ,ऑर्डरली एंड रेगुलर माइग्रेशन को अपनाने से संबंधित घटकों को शामिल किया गया था।
  • हाल ही में ऑस्ट्रिया ने ग्लोबल कॉम्पेक्ट के प्रवास-समर्थक दृष्टिकोण की आलोचना करते हुए यह घोषणा की कि वह ऐसे किसी प्रवासन समझौते पर हस्ताक्षर नहीं करेगा जो ऑस्ट्रिया की राष्ट्रीय सुरक्षा के लिये एक खतरे का प्रतिनिधित्व करता हो। अतः कथन 1 सही नहीं है।
  • ग्लोबल कॉम्पैक्ट फॉर सेफ ऑर्डरली एंड रेगुलर माइग्रेशन, गैर-कानूनी रूप से बाध्यकारी है। अतः कथन 2 सही नहीं है।
  • सुरक्षित, सुव्यवस्थित और नियमित प्रवासन के लिये ग्लोबल कॉम्पैक्ट विश्व का पहला, अंतर-सरकारी समझौता है, जिसमें अंतर्राष्ट्रीय प्रवास के सभी आयामों को समग्र एवं व्यापक रूप से शामिल किया गया है। अतः कथन 3 सही नहीं है।

संदर्भ:

71. निम्नलिखित देशों पर विचार कीजिये:

  1. बुल्गारिया
  2. चेक रिपब्लिक
  3. हंगरी
  4. लातविया
  5. लिथुआनिया
  6. रोमानिया

उपर्युक्त में से कितने देशों की सीमाएँ यूक्रेन की सीमा के साथ साझी हैं?

(a) केवल दो                           
(b) 
केवल तीन
(c) केवल चार                        
(d) 
केवल पाँच

उत्तर:(a)

व्याख्या:

  • दिये गए मानचित्र के अनुसार, हंगरी और रोमानिया यूक्रेन के साथ अपनी भूमि सीमाएँ साझा करते हैं।

  • अतः विकल्प A सही है।

स्रोत:

72. पृथ्वी के वायुमंडल के संदर्भ में, निम्नलिखित कथनों में से कौन-सा एक सही है?

(a)  भूमध्यरेखा पर प्राप्त होने वाले सूर्यताप की कुल मात्रा, ध्रुवों पर प्राप्त होने वाले सूर्यताप की कुल मात्रा की लगभग 10 गुनी है।
(b)  अवरक्त किरणें सूर्याताप का लगभग दो-तिहाई भाग हैं।
(c)  अवरक्त तरंगें निचले वायुमंडल में संकेंद्रित जलवाष्प द्वारा वृहद् रूप से अवशोषित होती हैं।
(d)  अवरक्त तरंगें सौर विकिरण की विद्युतचुम्बकीय तरंगों के दृश्यमान स्पेक्ट्रम के भाग हैं।

उत्तर:(C)

व्याख्या:

  • पृथ्वी के पृष्ठ पर प्राप्त होने वाली ऊर्जा का अधिकतम अंश लघु तरंगदैर्ध्य के रूप में आता है। पृथ्वी को प्राप्त होने वाली ऊर्जा को ‘आगमी सौर विकिरण’ या छोटे रूप में ‘सूर्यातप’ कहते हैं।
  • लघु तरंगदैर्ध्य वाले सौर-विकिरण के लिए वायुमंडल अधिकांशतः पारदर्शी होता है। पृथ्वी की सतह पर पहुँचने से पहले सूर्य की किरणें वायुमंडल से होकर गुजरती हैं। क्षोभमंडल में मौजूद जलवाष्प, ओज़ोन तथा अन्य किरणें अवरक्त विकिरण को अवशोषित कर लेती हैं।

अतः विकल्प C सही है।

स्रोत:

73. निम्नलिखित कथनों पर विचार कीजिये:

कथन-I: उष्णकटिबंधीय वर्षावनों की मृदा पोषक तत्त्वों से भरपूर होती है।

कथन-II: उष्णकटिबंधीय वर्षावनों के उच्च ताप और आर्द्रता के कारण मृदा में विद्यमान मृत जैव पदार्थ का द्रुत अपघटन होता है।

उपर्युक्त कथनों के बारे में, निम्नलिखित में से कौन-सा एक सही है?

(a)  कथन-I और कथन-II दोनों सही हैं तथा कथन-II, कथन-I की सही व्याख्या है
(b)  कथन-I और कथन-II दोनों सही हैं तथा कथन-II, कथन-I की सही व्याख्या नहीं है
(c)  कथन-I सही है किन्तु कथन-II गलत है।
(d)  कथन-I गलत है किन्तु कथन-II सही है

उत्तर:(d)

व्याख्या:

  • सामान्यतः माना जाता है कि उष्णकटिबंधीय वर्षावन की मृदा ऊष्मा और नमी के कारण बहुत समृद्ध होगी, जबकि इसके विपरीत उष्णकटिबंधीय वर्षावनों में मृदा समृद्ध नहीं होती है। अतः कथन 1 सही नहीं है।
  • उष्णकटिबंधीय वर्षावनों के उच्च ताप और नमी के कारण मृदा में मृत कार्बनिक पदार्थ अन्य जलवायु की तुलना में अधिक तेज़ी से विघटित होते हैं, इस प्रकार इसके पोषक तत्व तेज़ी से अपघटित हो जाते हैं।
  • उष्णकटिबंधीय वर्षावनों में वर्षा की उच्च मात्रा अन्य जलवायु की तुलना में मृदा से पोषक तत्वों को अधिक तेज़ी से निक्षालित कर देती है। अतः कथन 2 सही है।

West Texas A&M University:

74. निम्नलिखित कथनों पर विचार कीजिये:

कथन-I: ग्रीष्म ऋतु में महाद्वीपों और महासागरों के बीच तापमान विपर्यास शीत ऋतु की अपेक्षा अधिक होता है।

कथन-II: जल की विशिष्ट ऊष्मा, भूपृष्ठ की विशिष्ट ऊष्मा की अपेक्षा अधिक होती है।

उपर्युक्त कथनों के बारे में, निम्नलिखित में से कौन-सा एक सही है?

(a)  कथन-I और कथन-II दोनों सही है तथा कथन-II, कथन-I की सही व्याख्या है।
(b)  कथन-I और कथन-II दोनों सही हैं तथा कथन-II, कथन-I की सही व्याख्या नहीं है।
(c)  कथन-I सही है किन्तु कथन-II गलत है
(d)  कथन-I गलत है किन्तु कथन-II सही है

उत्तर:(a)

व्याख्या:

  • विशिष्ट उष्मा किसी पदार्थ का वह गुण है जो किसी पदार्थ द्वारा दी गई ऊष्मा के अवशोषित (या अस्वीकृत) होने पर पदार्थ के तापमान में परिवर्तन (कोई चरण परिवर्तन नहीं होता है) को निर्धारित करता है।
  • जल में भूपृष्ठ की तुलना में लगभग चार गुना अधिक ऊष्मा क्षमता होती है, यानी जल के तापमान को बढ़ाने हेतु बहुत अधिक ऊर्जा की आवश्यकता होती है, यही कारण है कि जल का तापमान परिवर्तन धीमी गति से होता है। अतः कथन 2 सही है।
  • भारत और उत्तरी यूरोप जैसे अपेक्षाकृत अच्छी तरह से परिभाषित मौसम वाले स्थानों हेतु ऐसी स्थितियों से तात्पर्य वसंत ऋतु के अंत/शुरुआती गर्मियों और शरद ऋतु के अंत/शुरुआती सर्दियों के दौरान जल एवं भूपृष्ठ के तापमान में बड़े अंतर से है। अतः कथन 2 सही है।
  • अतः कथन I और कथन II दोनों सही हैं और कथन II, कथन I की सही व्याख्या है।

स्रोत…

75. निम्नलिखित कथनों पर विचार कीजिये:

  1. भूकम्प-लेखी (सीस्मोग्राफ) में S तरंगों से पूर्व P तरंगें अभिलिखित की जाती है।
  2. P तरंगों में, अलग-अलग कण तरंग प्रसार की दिशा में आगे-पीछे कंपन करते हैं, जबकि तरंगों में कण तरंग प्रसार की दिशा के समकोणीय ऊपर-नीचे कंपन करते हैं।

उपर्युक्त कथनों में से कौन-सा/से सही है/है?

(a) केवल 1                            
(b) 
केवल 2
(c) 1 और 2 दोनों               
(d) 
न तो 1 और न ही 2

उत्तर:(c)

व्याख्या:

  • भूकंप के दौरान दो मुख्य प्रकार की कंपन तरंगें पृथ्वी से होकर गुजरती हैं। प्राथमिक तरंगें P, सबसे तेज़ी से गमन करती हैं और सिस्मोग्राफ पर सबसे पहले दर्ज की जाती हैं। द्वितीयक तरंगें या S, P की तुलना में धीमी गति से गमन करती हैं। अतः कथन 1 सही है।
    • चूँकि S तरंगों का आयाम P तरंगों की तुलना में अधिक होता है इसलिये इन दोनों को सिस्मोग्राम पर आसानी से दर्ज किया जा सकता है।
  • भूकंपीय P तरंगों को संपीडित या अनुदैर्ध्य तरंगें भी कहा जाता है। ये ध्वनि तरंगों की तरह सतह को आगे और पीछे की ओर कंपित करते हुए गमन करती हैं।
    • इसमें कंपन से कण गति प्रसार (अनुदैर्ध्य) इसके गमन की दिशा के समानांतर होता है। इस तरह से तरंग गुजरने के बाद पदार्थ अपने मूल अवस्था में आ जाता है।
  • भूकंप में S तरंगें अनुप्रस्थ तरंगों के उदाहरण हैं।
    • अनुप्रस्थ तरंग में कंपन से पृथ्वी के कणों का विस्थापन तरंग प्रसार की दिशा के लंबवत होता है। जैसे ही तरंग गुजरती है यह ऊपर और नीचे की ओर दोलन करते हैं। अतः कथन 2 सही है।

स्रोत:

76. भारत में कोयला आधारित तापीय शक्ति संयंत्रों के संदर्भ में, निम्नलिखित कथनों पर विचार कीजिये:

  1. उनमें से किसी में भी समुद्र जल का उपयोग नहीं होता।
  2. उनमें से कोई भी जल संकट वाले ज़िले में स्थापित नहीं है।
  3. उनमें से कोई भी निजी स्वामित्व में नहीं है।

उपर्युक्त में से कितने कथन सही हैं?

(a) केवल एक                       
(b) 
केवल दो
(c) सभी तीन                         
(d) 
कोई भी नहीं

उत्तर:(d)

व्याख्या:

 

  • उपरोक्त चित्रों से पता चलता है कि कोयला आधारित विद्युत् संयंत्र सागरीय जल का उपयोग करते हैं। अतः कथन 1 सही नहीं है।
  • अतः कथन 2 सही नहीं है।
  • अडानी पॉवर लिमिटेड 13,650 मेगावाट की स्थापित क्षमता के साथ भारत में सबसे बड़ा निजी ताप शक्ति उत्पादक है। अतः कथन 3 सही नहीं है।

स्रोत:

77. ‘वोलबैचिया पद्धति’ का कभी-कभी निम्नलिखित में से किस एक के संदर्भ में उल्लेख होता है?

(a)  मच्छरों से होने वाले विषाणु रोगों के प्रसार को नियंत्रित करना
(b)  शेष शस्य (क्रॉप रेज़िड्यु) से संवेष्टन सामग्री (पैकिंग मटीरियल) बनाना
(c)  जैव निम्नीकरणीय प्लास्टिकों का उत्पादन करना
(d)  जैव मात्रा के ऊष्मरासायनिक रूपांतरण से बायोचार का उत्पादन करना

उत्तर: (a)

व्याख्या:

  • वोलबैचिया (कुछ मच्छरों सहित) लगभग 60% कीट प्रजातियों में मौजूद प्राकृतिक बैक्टीरिया है। वोलबैचिया बैक्टीरिया लोगों या जानवरों (उदाहरण के लिये, मछली, पक्षी, पालतू जानवर) को संक्रमित नहीं कर सकता है।
    • कई सालों से वैज्ञानिक वोलबैचिया का अध्ययन कर रहे हैं और मानव में संक्रमित वायरस फैलाने वाले मच्छरों को संभावित रूप से नियंत्रित करने के लिये इसका उपयोग करने के तरीकों की तलाश कर रहे हैं।
  • वोलबैचिया विधि से संबंधित विश्व मच्छर कार्यक्रम:
    • WMP's की फील्ड टीमें नर और मादा एडीज एजिप्टी मच्छरों को वोलबैचिया के साथ कई सप्ताह तक वातावरण में प्रसारित करती हैं। ये मच्छर तब प्राकृतिक रूप से उपलब्ध मच्छरों के साथ प्रजनन करते हैं। इसके साथ ही वोलबाचिया वाले मच्छरों का प्रतिशत उच्च बना रहता है। वोल्बाचिया वाले मच्छरों द्वारा लोगों में वायरस को प्रसारित करने की क्षमता कम होती है, जिससे ज़ीका, डेंगू और चिकनगुनिया का खतरा कम हो जाता है।

अतः विकल्प A सही है।

स्रोत:

78. निम्नलिखित गतिविधियों पर विचार कीजिये

  1. बड़े पैमाने पर बारीक पिसी हुई बेसाल्ट शैल खेतों में बिछाना
  2. चूना मिलाकर महासागरों की क्षारीयता बढ़ाना
  3. विभिन्न उद्योगों द्वारा निर्मुक्त कार्बन डाइऑक्साइड का अभिग्रहण कर उसे कार्बोनिटीकृत जल के रूप में परित्यक्त भूमिगत खानों के अंदर पंप करना

उपर्युक्त गतिविधियों में से कितनी कार्बन और अभिग्रहण और विविक्तीभवन (सिकेस्ट्रेशन) के लिये प्राय: विचार और चर्चा में लाई जाती है?

(a) केवल एक                      
(b) केवल दो
(c) सभी तीन                        
(d) 
कोई भी नहीं

उत्तर:(c)

व्याख्या:

  • कार्बन विविक्तीभवन (सीक्वेस्ट्रेशन) एक प्राकृतिक एवं कृत्रिम दोनों प्रक्रिया है जिसके द्वारा कार्बन डाइऑक्साइड को पृथ्वी के वायुमंडल से रिमूव कर दिया जाता है एवं पुनः तरल या ठोस रूप में संग्रहीत किया जाता है।
  • ग्राउंड बेसाल्ट (बारीक पिसी हुई बेसाल्ट शैल) का उपयोग शुरुआती तीस के दशक से खनिज उर्वरक के रूप में किया जाता रहा है। ग्राउंड बेसाल्ट वातावरण और मृदा के छिद्रों से CO2 को कैप्चर करता है जिससे मृदा का PH बढ़ता है तथा यह प्रक्रिया महासागरीय अम्लीकरण को कम करती है। अंतःस्पदंन दर के आधार पर एक टन बेसाल्ट 0.153–0.165 टन CO2 ग्रहण करता है।
    • कृषि उर्वरक में डाला गया ग्राउंड बेसाल्ट वायुमंडलीय कार्बन डाइऑक्साइड (CO2) को कैप्चर करता है, मृदा का PH बढ़ाता है, महासागरीय अम्लीकरण कम होता है तथा मैग्नीशियम, पोटेशियम, कैल्शियम, लौह एवं फास्फोरस जैसे महत्त्वपूर्ण पोषक तत्त्वों की आपूर्ति करता है। अत: 1 सही है।
  • महासागर प्रति वर्ष मानवीय गतिविधियों के माध्यम से उत्सर्जित कार्बन डाइऑक्साइड का लगभग 25% अवशोषित करते हैं।
    • जैसे-जैसे ध्रुवीय क्षेत्र अधिक कार्बन डाइऑक्साइड को अवशोषित करते जाएँगे एवं वर्ष 2100 तक अधिकांश वैश्विक महासागरों के कार्बन डाइऑक्साइड से निर्मित होने का अनुमान लगाया जा रहा है, जो संभवतः समुद्र की रसायनिकी को परिवर्तित कर इसे और अधिक अम्लीय बना देगा।
    • चूँकि कार्बन महासागरों को अम्लीय बनाता है, महासागरों की बढ़ती क्षारीयता कार्बन प्रच्छादन में मदद करेगी। अत: 2 सही है।
  • कार्बन अभिग्रहण और विविक्तिभवन (CCS) गतिविधियों में, कार्बन डाइऑक्साइड को पहले औद्योगिक उत्सर्जन में निहित अन्य गैसों से अलग किया जाता है।
    • इसके बाद इसे संपीड़ित किया जाता है एवं एक ऐसे स्थान पर ले जाया जाता है जो दीर्घकालिक भंडारण के लिये वातावरण से अलग होता है।
    • उपयुक्त भंडारण स्थानों में भूगर्भिक संरचनाएँ जैसे कि परित्यक्त भूमिगत खानें (तलछटी चट्टानें जिनके छिद्र स्थान घुलित लवणों की उच्च सांद्रता वाले जल से संतृप्त होते हैं), अनुपयोगी तेल एवं गैस कुँए या गहरे समुद्र आदि शामिल हो सकते हैं । अत: 3 सही है।

स्रोत:

79. ‘एरियल मेटाजिनोमिक्स’ निम्नलिखित में से किस एक स्थिति को सबसे सही निर्दिष्ट करता है?

(a)  किसी पर्यावास में वायु से डी.एन.ए. प्रतिदर्शों को एक बार में एकत्र करना
(b)  किसी पर्यावास की पक्षी जातियों की आनुवंशिक रचना को समझना
(c)  गतिशील प्राणियों से रुधिर प्रतिदर्श लेने के लिये वायुवाहित युक्तियों का प्रयोग करना
(d)  भूमितल और जल निकायों से पादप एवं जंतु प्रतिदर्श एकत्र करने के लिये अगम्य क्षेत्रों में ड्रोन भेजना

उत्तर:(a)

व्याख्या:

  • मेटाजिनोमिक्स का आशय सभी जीवों (आमतौर पर रोगाणुओं) से काफी बड़ी मात्रा में पृथक किये गए संपूर्ण न्यूक्लियोटाइड अनुक्रमों की संरचना और कार्य का अध्ययन करना है।
    • मेटाजिनोमिक्स का उपयोग अक्सर सूक्ष्मजीवों (मानव त्वचा, मृदा या जल में पाए जाने वाले) के विशिष्ट समुदाय का अध्ययन करने के लिये किया जाता है।
  • एरियल मेटाजेनोमिक्स एक ऐसा वैज्ञानिक क्षेत्र है जिसमें वायु में मौजूद आनुवंशिक पदार्थों (विशेष रूप से पर्यावास में निलंबित सूक्ष्मजीवों) के डीएनए और आरएनए का अध्ययन करना शामिल है।
    • यह आउटडोर वायु, इनडोर वायु और वायुजनित कणों जैसे विभिन्न वातावरणों से एकत्र किये गए वायु के नमूनों में पाए जाने वाले माइक्रोबियल जीवों या माइक्रोबायोम के विश्लेषण पर केंद्रित है।
  • अतः विकल्प A सही है।

स्रोत…

80. ‘माइक्रोसैटेलाइट डी.एन.ए.’ निम्नलिखित में से किस एक मामले में प्रयोग किया जाता है?

(a)  प्राणिजात की विभिन्न जातियों के बीच विकासपरक संबंधों का अध्ययन करना
(b)  ‘स्टेम कोशिकाओंको विविध प्रकार्यात्मक ऊतकों में रूपांतरित होने के लिये उद्दीप्त करना
(c)  उद्यान-कृषि के पादपों के क्लोनी प्रसार को संवर्धित करना
(d)  किसी समष्टि में शृंखलाबद्ध औषध परीक्षण कर औषधों की प्रभावकारिता का आकलन करना

उत्तर:(d)

व्याख्या:

  • माइक्रोसैटेलाइट, जीनोमिक्स से संबंधित है, जो कि DNA के एक छोटे खंड को संदर्भित करता है, आमतौर पर लंबाई में एक से छह या अधिक क्षार-युग्मों (बेस पेयर) , जिनका एक विशेष जीनोमिक स्थान पर कई बार अनुक्रमण होता है।
    • ये DNA अनुक्रम आमतौर पर गैर-कोडित होते हैं।
  • ये रोग की व्यापकता, दुर्लभ रोगों से संबंधित नए जीनों की खोज, उन्नत नैदानिक विधियों और इन बीमारियों के लिये नवीन औषधियों के विकास में मूल्यवान अंतर्दृष्टि प्रदान करने में उपयोगी हैं।
  • अतः विकल्प D सही है।

स्रोत:

81. जननी सुरक्षा योजना के संबंध में, निम्नलिखित कथनों पर विचार कीजिये

  1. यह राज्यों के स्वास्थ्य विभागों की एक सुरक्षित मातृत्व की योजना है।
  2. इसका उद्देश्य गरीब गर्भवती महिलाओं में मातृ तथा नवजात शिशु मृत्यु दर में कमी लाना है।
  3. इसका लक्ष्य गरीब गर्भवती महिलाओं के लिये स्वास्थ्य केंद्रों में प्रसव को बढ़ावा देना है।
  4. इसके उद्देश्यों में एक वर्ष की आयु तक के बीमार बच्चों को लोक स्वास्थ्य सुविधाएँ प्रदान करना शामिल है।

उपर्युक्त में से कितने कथन सही हैं?

(a) केवल एक                       
(b) 
केवल दो
(c) केवल तीन                      
(d) 
सभी चार

उत्तर:(b)

व्याख्या:

जननी सुरक्षा योजना:

  • यह योजना 12 अप्रैल, 2005 को शुरू हुई थी, जिसे सभी राज्यों एवं केंद्रशासित प्रदेशों में लागू की जा रही है, जिसमें कम प्रदर्शन करने वाले राज्यों पर विशेष ज़ोर दिया गया है।
  • यह राष्ट्रीय ग्रामीण स्वास्थ्य मिशन (National Rural Health Mission- NRHM) के तहत एक सुरक्षित मातृत्त्व योजना है। अतः कथन 1 सही नहीं है।
  • इसे गरीब गर्भवती महिलाओं के बीच संस्थागत प्रसव को बढ़ावा देकर मातृ एवं नवजात मृत्यु दर को कम करने के उद्देश्य से लागू किया गया था। अतः कथन 2 और 3 सही हैं।
  • एक वर्ष तक के बीमार शिशुओं को लोक स्वास्थ्य सुविधाएँ उपलब्ध कराना योजना का उद्देश्य नहीं है। अतः कथन 4 सही नहीं है।

स्रोत…

82. एनीमिया मुक्त भारत रणनीति के अंतर्गत की जा रही व्यवस्थाओं के संदर्भ में, निम्नलिखित कथनों पर विचार कीजिये:

  1. इसमें स्कूल जाने से पूर्व के (प्री-स्कूल) बच्चों, किशोरों और गर्भवती महिलाओं के लिये रोगनिरोधक कैल्सियम पूरकता प्रदान की जाती है।
  2. इसमें शिशु जन्म के समय देरी से रज्जु बंद करने के लिये अभियान चलाया जाता है।
  3. इसमें बच्चों और किशोरों की निर्धारित अवधियों पर कृमि-मुक्ति की जाती है।
  4. इसमें मलेरिया, हीमोग्लोबिनोपैथी और फ्लुओरोसिस पर विशेष ध्यान देने के साथ स्थानिक बस्तियों में एनीमिया के गैर-पोषण कारणों की ओर ध्यान दिलाना शामिल है।

उपर्युक्त में से कितने कथन सही हैं?

(a)  केवल एक                       
(b) 
केवल दो
(c)  केवल तीन                       
(d)  
सभी चार

उत्तर:(c)

व्याख्या:

एनीमिया (रक्ताल्पता) मुक्त भारत के हस्तक्षेप:

  • इसमें रोगनिरोधी कैल्शियम पूरकता नहीं बल्कि प्रोफिलैक्टिक आयरन और फोलिक एसिड पूरकता बच्चों, किशोरों एवं प्रजनन आयु की महिलाओं तथा गर्भवती महिलाओं को एनीमिया के बावजूद प्रदान की जाती है। अतः कथन 1 सही नहीं है।
  • शिशु और छोटे बच्चों का उपयुक्त आहार (IYCF) 6 महीने और उससे अधिक उम्र के बच्चों हेतु पर्याप्त एवं आयु-उपयुक्त पूरक खाद्य पदार्थ प्रदान करने पर ध्यान केंद्रित करता है।
  • अपने आहार में विविधता लाकर भोजन की मात्रा एवं आवृत्ति में वृद्धि करना, साथ ही खाद्य पदार्थों में आयरन युक्त, प्रोटीन युक्त तथा विटामिन C युक्त खाद्य पदार्थों का सेवन बढ़ाना।
  • सभी स्वास्थ्य सुविधाओं के जन्मों में विलंबित कॉर्ड क्लैम्पिंग (रक्त के प्रवाह को रोकना) (कम से कम 3 मिनट या रज्‍जु स्पंदन बंद होने तक) को बढ़ावा देना, इसके बाद प्रसव के 1 घंटे के भीतर प्रारंभिक स्तनपान कराने पर ज़ोर देना। अतः कथन 2 सही है।
  • राष्ट्रीय कृमि मुक्ति दिवस (National Deworming Day- NDD) कार्यक्रम के तहत प्रत्येक वर्ष 1-19 वर्ष के आयु वर्ग के बच्चों हेतु द्वि-वार्षिक सामूहिक कृमि नियंत्रण किया जाता है। अतः कथन 3 सही है।
    • NDD योजना के हिस्से के रूप में एनीमिया मुक्त भारत में गर्भवती महिलाओं और प्रजनन आयु की महिलाओं हेतु कृमिनाशक दवा भी शामिल है।
  • मलेरिया, हीमोग्लोबिनोपैथी और फ्लोरोसिस पर विशेष ध्यान देने के साथ स्थानिक क्षेत्रों में एनीमिया के गैर-पोषण संबंधी कारणों को उजागर करना। अतः कथन 4 सही है।

स्रोत…

83. निम्नलिखित कथनों पर विचार कीजिये:

  1. ऑटोमोबाइल और एयरक्राफ्ट में प्रयुक्त होने वाले अवयवों के विनिर्माण में कार्बन तंतुओं का इस्तेमाल होता है।
  2. कार्बन तंतु एक बार प्रयुक्त होने पर पुनर्चक्रित किये जा सकते।

उपर्युक्त कथनों में से कौन-सा/से सही है/हैं?

(a)  केवल 1                             
(b)  
केवल 2
(c)  1 और 2 दोनों                
(d)  
न तो 1 और न ही 2

उत्तर:(a)

व्याख्या:

कार्बन फाइबर:

  • कार्बन फाइबर दीर्घ शृंखला बनाने हेतु एक-साथ बँधे कार्बन परमाणुओं से बना होता है। ये फाइबर बेहद कठोर, मज़बूत एवं हल्के होते हैं जिनका उत्कृष्ट निर्माण सामग्री बनाने के लिये कई प्रक्रियाओं में उपयोग किया जाता है।

  • अतः कथन 1 सही है।
  • कार्बन फाइबर पर्यावरण के अनुकूल है और इसका दीर्घ जीवन चक्र होता है। हालाँकि स्टील की तुलना में कार्बन फाइबर इसके निर्माण में लगभग 14 गुना अधिक ऊर्जा की खपत करता है। अतः इस महत्त्वपूर्ण ऊर्जा-गहनता के कारण ग्रीनहाउस गैसों का भारी उत्सर्जन होता है।
  • परिणामस्वरूप वैश्विक स्तर पर औद्योगिक अनुप्रयोगों में इस सामग्री की मांग की आपूर्ति करते हुए इस पर्यावरणीय प्रभाव को कम करने हेतु पुनर्चक्रण सबसे बड़े समाधानों में से एक हो सकता है।
  • वर्तमान में कार्बन फाइबर अपशिष्ट या अन्य फाइबर कंपोज़िट को चार प्रकार की तकनीकों का उपयोग करके पुनर्नवीनीकरण किया जा सकता है। अतः कथन 2 सही नहीं है।

स्रोत…

84. निम्नलिखित क्रियाओं पर विचार कीजिये:

  1. कार क्रैश/टक्कर का, जिससे एयरबैग लगभग तुरंत फैल जाते हैं, पता लगाना
  2. लैपटॉप अचानक धरातल पर गिर पड़ने का, जिससे हार्ड ड्राइव तुरंत बंद हो जाता है, पता लगाना
  3. स्मार्टफोन के झुकाव का, जिससे पोर्ट्रेट और लैंडस्केप मोड के बीच प्रदर्शन (डिस्प्ले) घूम जाता है, पता लगाना

उपर्युक्त में से कितनी क्रियाओं में, त्वरणमापी (ऐक्सेलरोमीटर) के प्रकार्य की आवश्यकता है?

(a) केवल एक                        
(b) 
केवल दो
(c) सभी तीन                         
(d) 
किसी में भी नहीं

उत्तर:(c)

व्याख्या:

त्वरणमापी (Accelerometer) के कार्य:

  • त्वरणमापी उन संकेतों का पता लगाता है जिनका उपयोग वाहन दुर्घटना/टक्कर की पहचान करने हेतु किया जाता है। अतः कथन 1 सही है।
  • त्वरणमापी का अनुप्रयोग अकादमिक और उपभोक्ता-चालित दोनों तरह के कई क्षेत्रों में किया जाता है।
    • उदाहरण के लिये लैपटॉप में त्वरणमापी हार्ड ड्राइव को क्षतिग्रस्त होने से बचाता है। यदि उपयोग के दौरान लैपटॉप अचानक गिर जाता है, तो त्वरणमापी अचानक फ्री फॉल का पता लगा लेगा एवं हार्ड ड्राइव प्लैटर में रीडिंग हेड्स से टकराने से बचने हेतु तत्काल हार्ड ड्राइव को बंद कर देगा। अतः कथन 2 सही है।
  • त्वरणमापी उपयोगकर्त्ता को किसी वस्तु के परिवेश को बेहतर ढंग से समझने की अनुमति प्रदान करता है।
    • उदाहरण के लिये स्मार्टफोन अपने डिस्प्ले को पोर्ट्रेट और लैंडस्केप मोड में व्यवस्थित करता है, जो इस बात पर निर्भर करता है कि उपयोगकर्त्ता फोन को किस प्रकार झुकाता है। अतः कथन 3 सही है।

स्रोत…

85. पुनर्संचरणशील ऐक्वाकल्चर प्रणाली में जैव निस्यंदकों (बायोफिल्टर) की भूमिका के संदर्भ में, निम्नलिखित कथनों पर विचार कीजिये:

  1. जैव निस्यंदक, बिना खाए हुए मत्स्य चारे को हटाकर, अपशिष्ट उपचार प्रदान करते हैं।
  2. जैव निस्यंदक, मत्स्य अपशिष्ट में विद्यमान अमोनिया को नाइट्रेट में बदल देते हैं।
  3. जैव निस्यंदक, जल में मत्स्य के लिये पोषक तत्त्व के रूप में फॉस्फोरस को बढ़ाते हैं।

उपर्युक्त में से कितने कथन सही है?

(a) केवल एक                        
(b) 
केवल दो
(c)  सभी तीन                          
(a) 
कोई भी नहीं

उत्तर:(b)

व्याख्या:

पुनर्संचरणशील एक्वाकल्चर प्रणाली (RAS) में जैव निस्यंदकों (बायोफिल्टर) की भूमिका:

  • RAS बायोफिल्टर मत्स्य प्रोटीन अपचय और ऑक्सीकरण प्रक्रियाओं द्वारा उत्पन्न नाइट्रोजनयुक्त अपशिष्ट उपोत्पादों को हटाने का कार्य करते हैं। अतः कथन 1 सही है।
  • RAS सामान्यतः मत्स्य प्रोटीन अपचय के उपोत्पाद के रूप में उत्पादित अमोनिया के स्तर को नियंत्रित करने हेतु बायोफिल्टर का उपयोग करता है।
    • बायोफिल्टर के उपयोग के माध्यम से एक्वैरियम प्रणाली से अमोनिया को हटा दिया जाता है। बायोफिल्टर एक अधःस्तर (Substrate) प्रदान करता है जिस पर नाइट्रिफाइंग बैक्टीरिया वृद्धि करते हैं। ये नाइट्रिफाइंग बैक्टीरिया अमोनिया का सेवन करते हैं एवं नाइट्राइट का उत्पादन करते हैं, जो मत्स्य के लिये भी विषैला होता है।
    • बायोफिल्टर में अन्य नाइट्रिफाइंग बैक्टीरिया नाइट्राइट का उपभोग करते हैं और नाइट्रेट का उत्पादन करते हैं। अतः कथन 2 सही है।
  • बायोफिल्टर सिस्टम संश्लिष्ट जल से नाइट्राइट, नाइट्रेट, फास्फोरस और अमोनियम आयनों की सांद्रता को कम करता है। अतः कथन 3 सही नहीं है।

स्रोत…

86. निम्नलिखित युग्मों पर विचार कीजिये:

  अंतरिक्ष में पिंड            वर्णन

  1. सेफीड       : अंतरिक्ष में धूल और गैस के विशाल बादल
  2. निहारिकाएँ   : तारे जो आवर्ती रूप से जलते-बुझते हैं
  3. पल्सर        : न्यूट्रॉन तारे जो तब बनते हैं जब विशाल तारों का ईंधन खत्म हो जाता है और उनका निपात हो जाता है

उपर्युक्त युग्मों में से कितने सही सुमेलित हैं?

(a)  केवल एक                        
(b)  
केवल दो
(c)  सभी तीन                         
(d)  
कोई भी नहीं

उत्तर: (d)

व्याख्या:

  • सेफीड्स, जिन्हें सेफीड वेरिएबल्स भी कहा जाता है, ऐसे तारे होते हैं जो समय-समय पर तीव्र एवं मंद रूप से प्रकाशित होते हैं। इस विशेषता के कारण इन्हें लाखों प्रकाश-वर्ष दूर ब्रह्मांडीय मानदंड (cosmic yardsticks) के रूप में उपयोग किया जाता है। अतः युग्म 1 सही सुमेलित नहीं है।

  • निहारीकाएँ, अंतरिक्ष में धूल एवं गैस का विशाल बादल हैं। कुछ निहारीकाएँ (एक से अधिक नेबुला) एक मृत तारे के विस्फोट से निकली गैस और धूल के माध्यम बनते हैं, जैसे कि सुपरनोवा। अतः युग्म 2 सही सुमेलित नहीं है।
  • पल्सर, घूर्णन कर रहे न्यूट्रॉन तारे होते हैं जिनमें नियमित अंतराल पर विकिरण के स्पंदन पाए जाते हैं जो सामान्यतः मिलीसेकंड से सेकंड तक होते हैं। अतः युग्म 3 सुमेलित नहीं है।

स्रोत:

87. निम्नलिखित देशों में से किस एक के पास अपनी उपग्रह मार्गनिर्देशन (नैविगेशन) प्रणाली है?

(a) ऑस्ट्रेलिया                        
(b) 
कनाडा
(c)  इज़रायल                           
(d)  
जापान

उत्तर:(d)

व्याख्या:

विश्व में संचालित मार्गनिर्देशन (नेविगेशन) प्रणालियाँ:

  • अमेरिका की GPS
  • रूस की GLONASS
  • यूरोपीय संघ की Galileo
  • चीन की BeiDou
  • भारत की NavIC
  • जापान की QZSS

अतः विकल्प (d) सही है।

स्रोत:

88. निम्नलिखित कथनों पर विचार कीजिये:

  1. बैलिस्टिक मिसाइल अपनी पूरी उड़ान में अवध्वनिक चाल पर प्रधार-नोदित होती हैं, जबकि क्रूज़ मिसाइल केवल उड़ान के आरंभिक चरण में रॉकेट संचालित होती हैं।
  2. अग्नि-V मध्यम दूरी की पराध्वनिक क्रूज़ मिसाइल है, जबकि ब्रह्मोस ठोस ईंधन चालित अंतरमहाद्वीपीय बैलिस्टिक मिसाइल है।

उपर्युक्त कथनों में से कौन-सा/से सही है/हैं?

(a)  केवल 1                             
(b)  
केवल 2
(c)  1 और 2 दोनों                
(d)  
न तो 1 और न ही 2

उत्तर:(d)

व्याख्या:

  • क्रूज़ मिसाइलें अपनी पूरी उड़ान में अवध्वनिक चाल पर प्रधार-नोदित होती हैं जबकि बैलिस्टिक मिसाइल केवल उड़ान के प्रारंभिक चरण में रॉकेट-संचालित होती हैं जिसके बाद ये लक्ष्य के लिये एक आर्किंग प्रक्षेपवक्र का अनुसरण करती हैं। अतः कथन 1 सही नहीं है।
  • अग्नि-V भारत की लंबी दूरी की सतह से सतह पर मार करने वाली बैलिस्टिक मिसाइल है जो 5,000 किमी. दूर के लक्ष्य को सटीकता से भेद सकती है। ब्रह्मोस एक पराध्वनिक क्रूज़ मिसाइल है जिसे संयुक्त रूसी-भारतीय ब्रह्मोस एयरोस्पेस कंपनी द्वारा विकसित किया गया है। अतः कथन 2 सही नहीं है।

स्रोत:

89. पारा प्रदूषण के बारे में, निम्नलिखित कथनों पर विचार कीजिये:

  1. स्वर्ण खनन गतिविधि विश्व में पारा प्रदूषण का स्रोत है।
  2. कोयला-आधारित ऊष्मीय शक्ति संयंत्र (थर्मल पावर प्लांट) से पारा प्रदूषण होता है।
  3. पारा के संपर्क में आने का कोई ज्ञात सुरक्षा स्तर नहीं है।

उपर्युक्त में से कितने कथन सही है?

(a) केवल एक                        
(b) 
केवल दो
(c) सभी तीन                         
(d) 
कोई भी नहीं

उत्तर:(c)

व्याख्या:

  • स्वर्ण खनन से आस-पास के जल संसाधनों पर विनाशकारी प्रभाव पड़ सकते हैं। स्वर्ण खनन के विषाक्त अवशेषों में तीन दर्जन से अधिक खतरनाक रसायन शामिल होते हैं:
    • आर्सेनिक
    • लैड
    • पारा
    • पेट्रोलियम उपोत्पाद
    • अम्ल
    • सायनाइड
    • अतः कथन 1 सही है।
  • अधिकांश पारा प्रदूषण कोयले से चलने वाले ताप विद्युत संयंत्रों और अन्य औद्योगिक प्रक्रियाओं द्वारा उत्पन्न होता है। अतः कथन 2 सही है।
  • पारा उत्सर्जन के लिये कोयला आधारित ऊष्मीय शक्ति संयंत्रों का योगदान कोयले के दहन से होने वाले कुल उत्सर्जन का 70.7% है।
  • तीव्र या जीर्ण पारा जोखिम विकास की किसी भी अवधि के दौरान प्रतिकूल प्रभाव उत्पन्न कर सकता है। पारा एक अत्यधिक विषैला तत्व है; पारे के संपर्क में आने का कोई ज्ञात सुरक्षा-स्तर नहीं है। अतः कथन 3 सही है।

स्रोत:

90. हरित हाइड्रोजन के संदर्भ में, निम्नलिखित कथनों पर विचार कीजिये:

  1. इसे आंतरिक दहन के लिये ईंधन के रूप में सीधे इस्तेमाल किया जा सकता है।
  2. इसे प्राकृतिक गैस के साथ मिलाकर ताप या शक्ति जनन के लिये ईंधन के रूप में इस्तेमाल किया जा सकता है।
  3. इसे वाहन चालन के लिये हाइड्रोजन ईंधन प्रकोष्ठ में इस्तेमाल किया जा सकता है।

उपर्युक्त में से कितने कथन सही हैं?

(a)  केवल एक                        
(b)  
केवल दो
(c)  सभी तीन                         
(d)  
कोई भी नहीं

उत्तर:(b)

हरित हाइड्रोजन: 

  • हरित हाइड्रोजन को अक्षय ऊर्जा स्रोतों जैसे सौर या पवन ऊर्जा का उपयोग करके जल के इलेक्ट्रोलिसिस के माध्यम से उत्पन्न किया जाता है।
  • आंतरिक दहन के लिये हरित हाइड्रोजन का सीधे उपयोग नहीं किया जा सकता है। हालाँकि इसका उपयोग आंतरिक दहन इंजन में कुछ परिवर्तनों के साथ किया जा सकता है। अतः कथन 1 सही नहीं है।
  • स्वच्छ माध्यमों से उत्पादित हाइड्रोजन को प्राकृतिक गैस पाइपलाइनों में इंजेक्ट किया जा सकता है और परिणामी मिश्रणों का उपयोग केवल प्राकृतिक गैस का उपयोग करने से होने वाले उत्सर्जन की तुलना में कम उत्सर्जन के द्वारा ऊष्मा और विद्युत् उत्पन्न करने के लिए किया जा सकता है। अतः कथन 2 सही है।
  • हाइड्रोजन का उपयोग दो प्रकार के वाहनों में किया जा सकता है: आंतरिक दहन इंजन (ICEs) और ईंधन सेल में। उदाहरण के लिए जर्मनी की हाइड्रोजन से चलने वाली पैसेंजर ट्रेन। अतः कथन 3 सही है।

Source:

91. भारत के संदर्भ में, निम्नलिखित कथनों पर विचार कीजिये :

  1. ‘सूक्ष्म, लघु और मध्यम उद्यम विकास (एम.एस.एम.ई.डी.) अधिनियम 2006’ के अनुसार, ‘जिनके संयंत्र और मशीन में निवेश 15 करोड़ रुपये से 25 करोड़ रुपये के बीच हैं, वे मध्यम उद्यम हैं
  2. सूक्ष्म, लघु और मध्यम उद्यमों को दिये गए सभी बैंक ऋण प्राथमिकता क्षेत्रक के अधीन अर्ह हैं।

उपर्युक्त कथनों में से कौन-सा/से सही है/है?

(a) केवल 1                             
(b) 
केवल 2
(c) 1 और 2 दोनों                
(d) 
न तो 1 और न ही 2

उत्तर:(b)

व्याख्या:

  • सूक्ष्म,लघु और मध्यम उद्यमों (MSMEs) की नई परिभाषा और मानदंड 1 जुलाई, 2020 से लागू होंगे।
    • सूक्ष्म विनिर्माण एवं सेवा इकाईयों की परिभाषा में परिवर्तन कर निवेश की सीमा बढ़ाकर 1 करोड़ रु एवं टर्नओवर की सीमा 5 करोड़ रु. कर दी गई है ।
    • लघु इकाई की निवेश की सीमा बढ़ाकर 10 करोड़ रु एवं  टर्नओवर की सीमा 50 करोड़ कर दी गई है।
    • जबकि मध्यम उद्यम इकाइयों के लिये अब यह सीमा निवेश के लिये 50 करोड़ रु और टर्नओवर के लिये  250 करोड़ रु. कर दी गई है।अतः कथन 1 सही नहीं है।

  • प्राथमिकता क्षेत्रक को ऋण दिशानिर्देश: दिनांक 4 सितंबर, 2020 के 'प्राथमिकता प्राप्त क्षेत्र को ऋण (PSL) - लक्ष्य और वर्गीकरण' पर प्रमुख निर्देशों के अनुसार, निर्धारित शर्तों के अनुरूप MSME को दिये गए सभी बैंक ऋण प्राथमिकता क्षेत्रक के अधीन अहर्ता रखते हैं।
    • MSMED अधिनियम, 2006 के तहत उपकरणों में निवेश के संदर्भ में परिभाषित सूक्ष्म, लघु और मध्यम उद्यमों (विनिर्माण और सेवा दोनों क्षेत्रों के लिये) को बैंक ऋण, ऋण सीमा के बावजूद, ऋण प्राथमिकता क्षेत्रक के अधीन अहर्ता रखते हैं। जो कि 1 मार्च, 2018. से प्रभावी हैं अतः कथन 2 सही है।

स्रोत:

92. केंद्रीय बैंक डिजिटल मुद्राओं के संदर्भ में, निम्नलिखित कथनों पर विचार कीजिये:

  1. यू.एस. डॉलर या एस.डब्ल्यू.आई.एफ.टी. प्रणाली का प्रयोग किये बिना डिजिटल मुद्रा में भुगतान करना संभव है।
  2. कोई डिजिटल मुद्रा इसके अंदर प्रोग्रामिंग प्रतिबंध, जैसे कि इसके व्यय के समय-ढाँचे के साथ वितरित की जा सकती है।

उपर्यक्त कथनों में से कौन-सा /से सही है/हैं?

(a) केवल 1                             
(b) 
केवल 2
(c) 1 और 2 दोनों                
(d) 
न तो 1 और न ही 2

उत्तर:(c)

व्याख्या:

  • केंद्रीय बैंक डिजिटल मुद्रा (CBDC) एक केंद्रीय बैंक द्वारा जारी मुद्रा नोटों का एक डिजिटल रूप है। इसके अंतर्गत डिजिटल मुद्रा में भुगतान US  डॉलर या स्विफ्ट प्रणाली का उपयोग किये बिना होता है। अतः कथन 1 सही है।
  • CBDC को दो व्यापक प्रकारों में वर्गीकृत किया जा सकता है। सामान्य प्रयोजन या खुदरा (CBDC-R) और थोक (CBDC-W)।
    • खुदरा CBDC संभावित रूप से सभी के उपयोग के लिये, जैसे कि निजी क्षेत्र, गैर-वित्तीय उपभोक्ता एवं व्यवसाय आदि, उपलब्ध होता है जबकि थोक CBDC को कुछ चयनित  वित्तीय संस्थानों तक सीमित पहुँच के लिये डिज़ाइन किया गया है।
    • थोक CBDC इंटरबैंक ट्रांसफर और संबंधित थोक लेनदेन के निपटान के लिये अभिप्रेत करता है, जबकि खुदरा CBDC मुख्य रूप से खुदरा लेनदेन के लिये नकदी का एक इलेक्ट्रॉनिक रूप है।
  • प्रोग्रामयोग्यता: CBDC का एक महत्त्वपूर्ण अनुप्रयोग प्रोग्रामयोग्यता की तकनीकी संभावना है। CBDC के पास अंतिम उपयोग को संदर्भित कर मुद्रा की प्रोग्रामिंग करने की संभावना होती है। उदाहरण के लिये, बैंकों द्वारा कृषि ऋण को यह सुनिश्चित करने के लिये प्रोग्राम किया जा सकता है कि इनका उपयोग केवल कृषि आवश्यक सामग्रियों की दुकानों पर ही किया जाए।
    • हालाँकि, किसी मुद्रा की आवश्यक विशेषताओं को बनाए रखने के लिये CBDC की प्रोग्राम योग्यता सुविधा का सावधानीपूर्वक परिक्षण करने की आवश्यकता है। मौद्रिक नीति प्रसारण के लिये इसके अन्य निहितार्थ भी हो सकते हैं क्योंकि टोकन की समाप्ति तिथि हो सकती है,जो कि उन्हें खर्च करने की आवश्यकता को निर्धारित करेगा एवं इस प्रकार खपत सुनिश्चित हो सकेगी। अतः कथन 2 सही है।
    • निम्नलिखित का उपयोग करके टोकन की प्रोग्राम योग्यता प्राप्त की जा सकती है:
      • स्मार्ट अनुबंध: व्यावसायिक नियमों को संहिता के रूप में संग्रहीत किया जाता है जो लेन-देन के दौरान निष्पादित किया जाता है ताकि यह सत्यापित किया जा सके कि टोकन का सही उपयोग किया जा रहा है।
      • टोकन संस्करण: टोकन संस्करण को तकनीकी कोड वर्ग से संबद्ध किया जा सकता है। इसमें विकल्प यह है कि संस्करण को टोकन डेटा फ़ील्ड के रूप में संग्रहीत किया जाता है।

स्रोत:

93. वित्त के संदर्भ में, शब्द ‘बीटा’ किसे निर्दिष्ट करता है?

(a)  अलग-अलग प्लेटफॉर्मों से, साथ-साथ किसी परिसंपत्ति को खरीदने और बेचने की प्रक्रिया
(b)  किसी पोर्टफोलियो प्रबंधक की, जोखिम और प्रतिफल के बीच संतुलन लाने की, निवेश कार्यनीति
(c)  एक प्रकार का व्यवस्थागत जोखिम, जो वहाँ उत्पन्न होता है जहाँ पूर्ण प्रतिरक्षा संभव नहीं है।
(d)  एक संख्यात्मक मान, जो पूरे स्टॉक बाज़ार में होने वाले परिवर्तनों के प्रति किसी स्टॉक के विचलनों को मापता है।

उत्तर:(d)

व्याख्या:

वित्त के संदर्भ में 'बीटा' शब्द का आशय ऐसी स्थिति से है जिसमें शेयर बाज़ार के समग्र रूप से बढ़ने या घटने पर किसी व्यक्तिगत संपत्ति के उतार-चढ़ाव को मापा जाता है। इसका उपयोग जोखिम की माप के रूप में किया जाता है और यह कैपिटल एसेट प्राइसिंग मॉडल (CAPM) का एक अभिन्न अंग है। उच्च बीटा वाली कंपनी में अधिक जोखिम होता है और अधिक अपेक्षित रिटर्न भी होता है। बीटा गुणांक की व्याख्या इस प्रकार की जा सकती है:

β =1 बिल्कुल बाज़ार की तरह अस्थिर

β >1 बाज़ार से अधिक अस्थिर

β <1>0 बाज़ार से कम अस्थिर

β =0 बाज़ार से असंबद्ध

β <0 बाज़ार से नकारात्मक रूप से सहसंबद्ध

बीटा गुणांक की गणना प्रतिभूति के प्रतिफल के सहप्रसरण के गुणनफल और एक निर्दिष्ट अवधि में बाज़ार के प्रतिफल के प्रसरण द्वारा होने वाले प्रतिफल को विभाजित करके की जा सकती है।

स्रोत:

  • एक प्रकार का ऐसा प्रणालीगत जोखिम जो तब होता है जब परफेक्ट हेजिंग संभव नहीं होती है, आधार जोखिम कहा जाता है।
  • जोखिम बनाम लाभ को संतुलित करने हेतु किसी पोर्टफोलियो मैनेजर की निवेश रणनीति को एसेट एलोकेशन कहा जाता है।
  • कीमतों में होने वाले अंतर से लाभ प्राप्त करने हेतु विभिन्न प्लेटफॉर्मों, एक्सचेंजों या स्थानों से एक साथ संपत्ति की खरीद और बिक्री की प्रक्रिया को आर्बिट्रेज कहा जाता है।
  • ऐसा संख्यात्मक मान जिससे किसी स्टॉक के उतार-चढ़ाव को समग्र स्टॉक मार्केट में होने वाले परिवर्तन के संदर्भ में मापा जाता है, बीटा कहलाता है।

94. निम्नलिखित कथनों पर विचार कीजिये:

  1. स्वयं सहायता समूह [सेल्फ-हेल्प ग्रुप (एस.एच.जी.)] कार्यक्रम मूलत: भारतीय स्टेट बैंक द्वारा वित्तीय रूप से वंचितों को लघु ऋण प्रदान कर प्रारंभ किया गया था।
  2. किसी एस.एच.जी. में, समूह के सभी सदस्य उस ऋण के लिये उत्तरदायित्व लेते हैं, जो ऋण कोई अकेला सदस्य लेता है।
  3. क्षेत्रीय ग्रामीण बैंक और अनुसूचित वाणिज्यिक बैंक एस.एच. जी. को समर्थन देते हैं।

उपर्युक्त में से कितने कथन सही हैं?

(a) केवल एक                        
(b) 
केवल दो
(c) सभी तीन                         
(d) 
कोई भी नहीं

उत्तर:(b)

व्याख्या:

  • स्वयं सहायता समूह या SHG लगभग दो दशक एक प्रसिद्ध अवधारणा बन गई है। देश के आर्थिक विकास को गति देने में स्वयं सहायता समूहों की अपनी भूमिका है। स्वयं सहायता समूह अब एक आंदोलन के रूप में विकसित हुआ है। हम बांग्लादेश में डॉ. महमूद यूनुस द्वारा स्थापित स्वयं सहायता समूहों की अवधारणा की उत्पत्ति का पता लगा सकते हैं। भारत ने बांग्लादेश के मॉडल को संशोधित रूप में अपनाया है।
    • वर्ष 1970 में अहमदाबाद में 'सेवा' (स्व-नियोजित महिला संघ) संगठन की संस्थापक सदस्य ईलाबेन भट ने 'महिला एवं सूक्ष्म-वित्त' की अवधारणा विकसित की थी। महाराष्ट्र में अन्नपूर्णा महिला मंडल' और तमिलनाडु में 'कामकाजी महिला मंच' तथा कई राष्ट्रीय कृषि एवं ग्रामीण विकास बैंक (नाबार्ड) प्रायोजित समूहों ने 'सेवा' द्वारा निर्धारित पथ का अनुसरण किया है।
    • वर्ष 1991-92 में नाबार्ड ने बड़े पैमाने पर स्वयं सहायता समूहों को बढ़ावा देना शुरू किया, जो ‘SHG आंदोलन' के लिये वास्तविक अनुकरण बिंदु था। वर्ष 1993 में भारतीय रिज़र्व बैंक ने भी स्वयं सहायता समूहों को बैंकों में बचत खाते खोलने की अनुमति दी। अतः कथन 1 सही नहीं है।
  • ऐसे समूह उन सदस्यों के लिये सामूहिक सुनिश्चितता प्रणाली के रूप में कार्य करते हैं जो संगठित स्रोतों से ऋण लेने का प्रस्ताव रखते हैं। निर्धन लोग अपनी बचत पूंजी को बैंकों में जमा करते हैं। जिसके बदले में उन्हें अपनी सूक्ष्म इकाई उद्यम शुरू करने के लिये न्यूनतम ब्याज दर पर आसानी से ऋण प्राप्त होता है। अतः कथन 2 सही है।

अतः कथन 3 सही है।

स्रोत:

95. निम्नलिखित कथनों पर विचार कीजिये :

कथन-I : भारत की सार्वजनिक क्षेत्रक स्वास्थ्य देखभाल प्रणाली सीमित निरोधक, वर्धक और पुन:स्थापक देखभाल के साथ मुख्यत: रोगनाशक व्यवस्था पर ध्यान केन्द्रित करती है।

कथन-II : भारत के स्वास्थ्य देखभाल प्रदान करने के विकेंद्रीकृत उपागम के अंतर्गत, राज्य प्राथमिक रूप से स्वास्थ्य सेवाओं को जुटाने के लिये उत्तरदायी हैं।

उपर्युक्त कथनों के बारे में, निम्नलिखित में से कौन-सा एक सही है?

(a)  कथन-I और कथन-II दोनों सही हैं तथा कथन-II, कथन-I की सही व्याख्या है।
(b)  कथन-I और कथन-II दोनों सही हैं तथा कथन-II, कथन-I की सही व्याख्या नहीं है
(c)  कथन-I सही है किन्तु कथन-II गलत है।
(d)  कथन-I गलत है किन्तु कथन-II सही है।

उत्तर:(d)

व्याख्या:

  • आयुष्मान भारत - स्वास्थ्य और कल्याण केंद्र (AB-HWC) को आयुष्मान भारत कार्यक्रम के तहत सभी उम्र के लोगों के लिये निवारक, प्रोत्साहक, उपचारात्मक, पुनर्वास एवं उपशामक देखभाल में एक व्यापक श्रेणी की सेवाओं के चयनात्मक स्वास्थ्य देखभाल में होने वाली कमी हेतु शुरू किया गया था। अतः कथन 1 सही नहीं है।
  • आयुष्मान भारत में उप स्वास्थ्य केंद्र (SHC) स्तर पर स्वास्थ्य और कल्याण केंद्र को बहुउद्देश्यीय कार्यकर्त्ता (पुरुष एवं महिला) एवं आशा तथा प्राथमिक स्वास्थ्य केंद्र / शहरी प्राथमिक स्वास्थ्य केंद्र का आयोजन  राज्य सरकार द्वारा किया जाता है,
  • लेकिन तृतीयक स्वास्थ्य सेवाओं के मामले में केंद्र सरकार भी एक प्रमुख हितधारक है। अतः कथन 2 सही है।

96. निम्नलिखित कथनों पर विचार कीजिये:

कथन-I: संयुक्त राष्ट्रसंघ की ‘विश्व जल विकास रिपोर्ट, 2022’ के अनुसार, भारत प्रतिवर्ष विश्व के भौम जल के एक-चौथाई से अधिक जल का निष्कर्षण करता है।

कथन-II: भारत को अपने राज्यक्षेत्र में रहने वाली विश्व की लगभग 18% जनसंख्या के पेय जल और स्वच्छता आवश्यकताओं को पूरा करने के लिये प्रतिवर्ष विश्व में भौम जल के एक-चौथाई से अधिक जल का निष्कर्षण करने की ज़रूरत है।

उपर्युक्त कथनों के बारे में, निम्नलिखित में से कौन-सा एक सही है?

(a)  कथन-1 और कथन-II दोनों सही है कथन-II, कथन-I की सही व्याख्या है।
(b)  कथन-I और कथन-II दोनों सही है तथा कथन-II, कथन-I की सही व्याख्या नहीं है।
(c)  कथन-I सही है किन्तु कथन-II गलत है।
(d)  कथन-I गलत है किन्तु कथन-II सही है।

उत्तर:(c)

व्याख्या:

  • भारत विश्व में भूजल का सबसे बड़ा उपयोगकर्त्ता है। विश्व बैंक के अनुसार यह प्रति वर्ष अनुमानित 230 क्यूबिक किलोमीटर वैश्विक कुल के एक चौथाई से अधिक भूजल का उपयोग करता है।
  • 60% से अधिक सिंचित कृषि और 85% पेयजल आपूर्ति, भूजल पर निर्भर है। अविश्वसनीय एवं अपर्याप्त सार्वजनिक जल आपूर्ति के कारण शहरी निवासी भूजल पर निर्भर हैं (क्योंकि 89% भूजल का उपयोग सिंचाई के लिये किया जाता है)। अतः कथन 2 सही नहीं है।

उपरोक्त आँकड़ों से भारत के वैश्विक भूजल का लगभग आधा भाग उत्सर्जित होता है। अतः कथन 1 सही है।

स्रोत:

97. निम्नलिखित कथनों पर विचार कीजिये:

  1. भारत के संविधान के अनुसार, केंद्र सरकार का यह एक दायित्व है कि वह राज्यों को आंतरिक विक्षोभों से बचाए।
  2. भारत का संविधान राज्यों को, निवारक निरोध में रखे जा रहे किसी व्यक्ति को विधिक काउंसेल उपलब्ध कराने से छूट प्रदान करता है।
  3. आतंकवाद निवारण अधिनियम, 2002 के अनुसार, पुलिस के समक्ष अभियुक्त की संस्वीकृति को साक्ष्य के रूप में प्रयुक्त नहीं किया जा सकता।

उपर्युक्त में से कितने कथन सही हैं?

(a)  केवल एक                        
(b)  
केवल दो
(c)  सभी तीन                         
(d)  
कोई भी नहीं

उत्तर:(b)

व्याख्या:

  • अनुच्छेद 355 के अनुसार, संघ का यह कर्त्तव्य होगा कि वह प्रत्येक राज्य को बाह्य आक्रमण और आंतरिक असुरक्षा से बचाए और यह सुनिश्चित करे कि प्रत्येक राज्य की सरकार इस संविधान के प्रावधानों के अनुसार चले। अतः कथन 1 सही है।
  • उदाहरण के लिये संविधान का अनुच्छेद 22 (1) कानूनी परामर्श के अधिकारों को सुनिश्चित करता है, लेकिन अनुच्छेद 22 (3) (B) निवारक निरोध कानून के तहत हिरासत में लिये गए व्यक्तियों से यह अधिकार छीन लेता है। इन प्रावधानों के तहत सर्वोच्च न्यायालय ने ए.के. रॉय बनाम भारत संघ के मामले में यह फैसला सुनाया कि बंदियों को सलाहकार बोर्ड की सुनवाई में कानूनी प्रतिनिधित्त्व का अधिकार नहीं है। अतः कथन 2 सही है।
  • आतंकवादी और विघटनकारी गतिविधियाँ (रोकथाम) अधिनियम, 1987 और आतंकवाद निवारण अधिनियम, 2002 (क्रमशः टाडा और पोटा के रूप में जाना जाता है) ने पुलिस अधिकारियों के समक्ष अभियुक्तों द्वारा दिए गए बयानों को स्वीकार करने का प्रावधान किया था। अतः कथन 3 सही नहीं है।
    • भारतीय साक्ष्य अधिनियम, 18723 में यह प्रावधान है कि पुलिस प्राधिकरण के समक्ष या पुलिस हिरासत के तहत की गई स्वीकारोक्ति अस्वीकार्य है।

स्रोत:

98. निम्नलिखित में से कौन-सा एक देश, दशकों से गृहकलह और खाद्य संकट से त्रस्त रहा है तथा हाल के पिछले दिनों में अति गंभीर दुर्भिक्ष के लिये समाचारों में था?

(a) अंगोला
(b) 
कोस्ता रीका
(c) इकाडोर 
(d) 
सोमालिया

उत्तर:(d)

व्याख्या:

  • सोमालिया दशकों से गृह युद्ध और खाद्य संकट से जूझ रहा है, हाल के पिछले दिनों में यह अति गंभीर और दुर्भिक्ष के लिये समाचारों में था।
    • सोमालिया लंबे समय तक सूखे, संघर्ष, विस्थापन और असुरक्षा से प्रभावित रहा है, जिसके परिणामस्वरूप लाखों लोगों को गंभीर खाद्य असुरक्षा और कुपोषण का सामना करना पड़ा है।
    • वर्ष 2021 में सोमालिया को पिछले 25 वर्षों की तुलना में गंभीर रूप से रेगिस्तानी टिड्डे के प्रकोप का सामना करना पड़ा, जिसने खाद्य सुरक्षा और आजीविका को और संकट में डाल दिया।
  • रिपोर्ट में खाद्य संकट का सामना कर रहे अन्य देशों का भी उल्लेख किया गया है, जैसे कि इथियोपिया, दक्षिण सूडान, यमन, अफगानिस्तान और सीरिया आदि।

स्रोत:

99. निम्नलिखित कथनों पर विचार कीजिये:

  1. भारत में जैव विविधता प्रबंधन समितियाँ नागोया प्रोटोकॉल के उद्देश्यों को हासिल करने के लिये प्रमुख कुंजी हैं।
  2. जैव विविधता प्रबंधन समितियों के, अपने क्षेत्राधिकार के अंतर्गत, जैविक संसाधनों तक पहुँच के लिये संग्रह शुल्क लगाने की शक्ति सहित, पहुँच और लाभ सहभागिता निर्धारित करने के लिए, महत्त्वपूर्ण प्रकार्य हैं।

उपर्युक्त कथनों में से कौन-सा/से सही है/हैं?

(a)  केवल 1                             
(b)  
केवल 2
(c)  1 और 2 दोनों                
(d)  
न तो 1 और न ही 2

उत्तर: (c)

व्याख्या:

भारत में जैव विविधता शासन: भारत का जैविक विविधता अधिनियम, 2002 (BD अधिनियम), नागोया प्रोटोकॉल से निकटतम रूप से संबंधित है, इसका उद्देश्य जैविक विविधता अभिसमय (CBD) के प्रावधानों को लागू करना है।

नागोया प्रोटोकॉल ने आनुवंशिक संसाधनों में वाणिज्यिक एवं अनुसंधान के उपयोग को सुनिश्चित करने हेतु सरकार के ऐसे संसाधनों का संरक्षण करने वाले समुदाय के साथ लाभों को साझा करने की मांग की। जैविक विविधता अधिनियम, 2002 की धारा 41(1) के तहत राज्य में प्रत्येक स्थानीय निकाय अपने अधिकार क्षेत्र के भीतर एक जैव विविधता प्रबंधन समिति का गठन करेगा। अतः कथन 1 सही है।

  • BMC का मुख्य कार्य स्थानीय लोगों के परामर्श से जन जैव विविधता रजिस्टर (PBR) तैयार करना है। BMC, PBR में दर्ज सूचनाओं की सुरक्षा सुनिश्चित करने, विशेष रूप से बाहरी व्यक्तियों और एजेंसियों तक इसकी पहुँच को विनियमित करने के लिये उत्तरदायी होगा।
  • जन जैव विविधता रजिस्टर (PBR) तैयार करने के अतिरिक्त, BMC अपने संबंधित क्षेत्राधिकार में निम्नलिखित के लिये भी ज़िम्मेदार होगा: -
    • जैविक संसाधनों का संरक्षण, सतत् उपयोग एवं पहुँच तथा लाभ को साझा करना।
    • वाणिज्यिक एवं अनुसंधान उद्देश्यों हेतु जैविक संसाधनों और/या संबद्ध पारंपरिक ज्ञान तक पहुँच का विनियमन।
    • BMC जैविक संसाधनों तक पहुँच और प्रदान किये गए पारंपरिक ज्ञान के विवरण, संग्रह शुल्क का विवरण, प्राप्त लाभों का विवरण और अपने अधिकार क्षेत्र के अंतर्गत उनके साझाकरण के तरीके के बारे में जानकारी देने वाला एक रजिस्टर भी बनाए रखेगा। अतः कथन 2 सही है।

स्रोत:

100. भारत के राष्ट्रपति के निर्वाचन के बारे में, निम्निलिखित कथनों पर विचार कीजिये:

  1. संसद के दोनों में से किसी भी सदन या राज्यों की विधान-सभाओं में नामनिर्दिष्ट किये गए सदस्य निर्वाचक मंडल में शामिल किये जाने के लिये भी अर्ह है।
  2. निर्वाच्य विधान-सभा सीटें जितनी अधिक होती हैं, उस राज्य के प्रत्येक एम.एल.ए. के वोट का मान भी उतना ही अधिक होता है।
  3. मध्य प्रदेश के प्रत्येक एम.एल.ए. के वोट का मान, केरल के प्रत्येक एम.एल.ए. के वोट के मान से अधिक है।
  4. पुदुच्चेरी के प्रत्येक एम.एल.ए. के वोट का मान, अरुणाचल प्रदेश के प्रत्येक एम.एल.ए. के वोट के मान से अधिक है, क्योंकि अरुणाचल प्रदेश की तुलना में पुदुच्चेरी में कुल जनसंख्या का निर्वाच्य सीटों की कुल संख्या से अनुपात अधिक है।

उपर्युक्त में से कितने कथन सही है?

(a)  केवल एक
(b)  केवल दो
(c)  केवल तीन
(d)  सभी चार

उत्तर:(a)

व्याख्या:

  • विधायकों के वोट का मान अलग-अलग राज्यों में अलग-अलग होगा क्योंकि ऐसे प्रत्येक वोट के मान की गणना नीचे बताई गई प्रक्रिया द्वारा की जाती है। हालाँकि, सभी सांसदों के वोट का मान एक समान होता है।

स्रोत:

  • कथन 1 सही नहीं है।
  • कथन 2 और 3 सही नहीं हैं।
  • पुदुच्चेरी में कुल निर्वाच्य सीटों की कुल जनसंख्या का अनुपात =471707/30=15,723.56।
    • अरुणाचल प्रदेश में कुल निर्वाचित सीटों की कुल जनसंख्या का अनुपात = 467511/60 = 7,791.85।
    • पुदुच्चेरी के प्रत्येक एम.एल.ए. के वोट का मान अरुणाचल प्रदेश की तुलना में अधिक है क्योंकि पुदुच्चेरी में कुल निर्वाच्य सीटों की कुल संख्या का अनुपात अरुणाचल प्रदेश की तुलना में अधिक है। अतः कथन 4 सही है।
और सटीक उत्तरमाला।


















close
एसएमएस अलर्ट
Share Page
images-2
images-2